12.05.2013 Views

Exámenes resueltos.

Exámenes resueltos.

Exámenes resueltos.

SHOW MORE
SHOW LESS

Create successful ePaper yourself

Turn your PDF publications into a flip-book with our unique Google optimized e-Paper software.

AMPLIACIÓN DE CÁLCULO (Curso 2001/2002) Examen Final de Febrero 31.01.02<br />

Solución del<br />

PROBLEMA 2<br />

(3 puntos)<br />

Calcular el flujo del campo vectorial F (x, y, z) = z 2 cos(x 2 + y 2 ) k a través de la semiesfera<br />

según la normal exterior a la esfera.<br />

Respuesta:<br />

x 2 + y 2 + z 2 = a 2 , z ≥ 0,<br />

Utilizando coordenadas esféricas, una parametrización de la semiesfera viene dada por las ecuaciones:<br />

Σ := Σ(θ, φ) = (x(θ, φ), y(θ, φ), z(θ, φ)),<br />

donde,<br />

x(θ, φ) = a cos θ sen φ, y(θ, φ) = a sen θ sen φ, z(θ, φ) = a cos φ,<br />

con 0 < θ < 2π y 0 < φ < π/2.<br />

Un vector en la dirección de la normal a la semiesfera es entonces<br />

n := ∂ Σ<br />

∂φ × ∂ Σ<br />

∂θ = a 2 cos θ (sen φ) 2 , a 2 (sen φ) 2 sen θ, a 2 cos φ sen φ <br />

que, además, tiene el sentido de la normal exterior.<br />

Por otra parte, el valor que toma el campo vectorial sobre la semiesfera es:<br />

F (x(θ, φ), y(θ, φ), z(θ, φ)) = (0, 0, a 2 (cos φ) 2 cos(a 2 (sen φ) 2 )).<br />

Por lo tanto, el flujo pedido es, por definición:<br />

Φ :=<br />

<br />

Σ<br />

F · ds =<br />

= 2πa 4<br />

π/2<br />

0<br />

2π<br />

0<br />

π/2<br />

dθ<br />

0<br />

(cos φ) 3 cos(a 2 (sen φ) 2 ) sen φ dφ<br />

Para resolver esta integral efectuamos el cambio de variable<br />

del que se obtiene:<br />

que es el resultado pedido.<br />

a 2 (cos φ) 2 cos(a 2 (sen φ) 2 ) a 2 cos φ sen φ dφ<br />

u = a 2 (sen φ) 2 =⇒ du = 2a 2 cos φ sen φ<br />

a2 Φ := π (a<br />

0<br />

2 − u) cos u du = π(1 − cos a 2 )


AMPLIACIÓN DE CÁLCULO (Curso 2001/2002) Examen Final de Junio 24.06.02<br />

Solución del<br />

PROBLEMA 2<br />

(3 puntos)<br />

Una loxodroma sobre un cilindro es aquella curva que corta las generatrices del cilindro formando<br />

con cada una de ellas un mismo ángulo α. Se considera el cilindro de ecuación implícita<br />

x 2 + y 2 = r 2 .<br />

1) Determínense las funciones f para las cuales la curva<br />

x = r cos t<br />

y = r sent<br />

z = f(t)<br />

⎫<br />

⎬<br />

⎭<br />

t ∈ R ,<br />

es una loxodroma de ángulo α. ¿Qué nombre reciben estas curvas ?<br />

2) Considérense las loxodromas del apartado anterior que pasan por los puntos (r, 0, 0) y (r, 0, 2π)<br />

y, para cada una de ellas, calcúlese su longitud entre dichos puntos.<br />

Respuesta:<br />

1) El ángulo que forman dos curvas que se cortan en un punto es el menor ángulo α que forman sus<br />

tangentes en ese punto (α ∈ [0, π]).<br />

Las generatrices del cilindro dado son paralelas al eje OZ, por tanto un vector unitario en la<br />

dirección de la tangente a cualquiera de ellas es k = (0, 0, 1). En cuanto a la curva dada, un vector<br />

unitario en la dirección de la tangente es<br />

t =<br />

(x ′ (t), y ′ (t), z ′ (t))<br />

[(x ′ (t)) 2 + (y ′ (t)) 2 + (z ′ (t)) 2 ] 1/2 = (− r sen t, r cos t, f ′ (t))<br />

[r2 + (f ′ (t)) 2 ] 1/2 .<br />

El producto escalar entre ambos vectores unitarios da lugar a la expresión:<br />

t · k =<br />

Por otra parte, su producto vectorial es:<br />

f ′ (t)<br />

[r2 + (f ′ (t)) 2 = cos α . (1)<br />

] 1/2<br />

t × k =<br />

(r sen t, − r cos t, 0)<br />

[r 2 + (f ′ (t)) 2 ] 1/2<br />

Por tanto el módulo de este producto vectorial da lugar a la relación:<br />

|t × k| =<br />

Dividiendo miembro a miembro (1) entre (2) resulta:<br />

r<br />

[r2 + (f ′ (t)) 2 = sen α . (2)<br />

] 1/2<br />

f ′ (t) = r ctg α .<br />

Puesto que r = 0, esta operación siempre puede realizarse ya que, en este caso, se tiene sen α = 0.<br />

Integrando miembro a miembro en la última ecuación se obtiene el valor de la función f(t)<br />

buscado, que es:<br />

f(t) = (r ctg α) t + C


siendo C una constante arbitraria.<br />

Así pues, las ecuaciones paramétricas de la curva buscada son:<br />

x = r cos t<br />

⎫<br />

⎬<br />

y<br />

z<br />

=<br />

=<br />

r sent<br />

⎭<br />

(r ctg α) t + C<br />

t ∈ R ,<br />

que corresponden a una familia de hélices.<br />

2) Se trata de determinar la familia de hélices que pasa por los puntos dados.<br />

Si la hélice pasa por (r, 0, 0), entonces debe existir un t0 ∈ R tal que:<br />

De ambas expresiones se deduce que<br />

Además,<br />

x(t0) = r cos t0 = r =⇒ cos t0 = 1 ,<br />

y(t0) = r sent0 = 0 =⇒ sen t0 = 0 .<br />

t0 = 2nπ , n ∈ Z .<br />

z(t0) = (r ctg α) t0 + C = 0 =⇒ C = − (r ctg α) 2nπ .<br />

Por otra parte, si la hélice pasa por (r, 0, 2π), entonces debe existir un t1 ∈ R (t1 = t0) tal que:<br />

con lo cual,<br />

x(t1) = r cos t1 = r =⇒ cos t1 = 1 ,<br />

y(t1) = r sent1 = 0 =⇒ sen t1 = 0 ,<br />

t1 = 2mπ , m ∈ Z y m = n .<br />

Además,<br />

z(t1) = (r ctg α) t1 − (r ctg α) 2nπ = 2π =⇒ r ctg α =<br />

1<br />

m − n .<br />

Es decir, la hélice que pasa por los dos puntos dados es:<br />

x = r cos t<br />

y<br />

z<br />

=<br />

=<br />

r sent<br />

⎫<br />

⎬<br />

<br />

1<br />

2nπ t − m−n m−n<br />

⎭<br />

t ∈ R y n, m ∈ Z (n = m).<br />

La longitud de arco pedida viene dada por:<br />

<br />

<br />

<br />

<br />

2mπ<br />

<br />

r<br />

2nπ<br />

2 <br />

2 <br />

1 <br />

+<br />

dt <br />

m − n <br />

=<br />

⎛<br />

⎝ r2 +<br />

⎞<br />

2 1<br />

⎠ |m − n| 2 π .<br />

m − n<br />

Si se considera n = 0 y m = 1 entonces la longitud de arco es √ r 2 + 1 2 π.


AMPLIACIÓN DE CÁLCULO (Curso 2001/2002) Examen Final de Junio 24.06.02<br />

Solución del<br />

PROBLEMA 3<br />

(3 puntos)<br />

Sea U(r) (r = xi + yj + zk) un campo escalar de clase dos y armónico en un dominio M ⊂ R 3 . Sea<br />

Ω ⊂ M un subconjunto medible - Jordan compacto, cuya frontera S está formada por un número<br />

finito de superficies regulares.<br />

Se pide:<br />

1) Demostrar la igualdad<br />

<br />

Ω<br />

gradU 2 dxdydz =<br />

<br />

en donde la superficie S está orientada según la normal saliente.<br />

Indicación: div(V F) = V divF + F · gradV .<br />

S<br />

UgradUdσ (∗)<br />

2) Comprobar la fórmula anterior (calculando explícitamente las dos integrales de la igualdad (∗) )<br />

para el campo escalar armónico<br />

U(r) = 1<br />

r<br />

y el recinto Ω ⊂ R 3 limitado por las superficies:<br />

Respuesta:<br />

S1 : esfera de centro el origen y radio 1.<br />

S2 : superficie cónica de revolución alrededor del eje z de vértice el origen<br />

y semiángulo en el vértice.<br />

S3 : plano z = 1/2.<br />

1) Sea F = UgradU que es un campo de clase uno en M. Mediante el uso del teorema de Gauss,<br />

el flujo de F sobre la superficie S puede ser expresado, de la siguiente forma<br />

<br />

<br />

<br />

UgradUds = div(UgradU)dxdydz = (Udiv(gradU) + gradU · gradU) dxdydz =<br />

S<br />

=<br />

Ω<br />

Ω<br />

<br />

2<br />

U∆U + gradU <br />

dxdydz = gradU 2 dxdydz<br />

Ω<br />

donde se ha utilizado que ∆U = 0 en Ω por definición de campo armónico.<br />

2) El conjunto Ω se muestra en la figura.<br />

Ω<br />

Σ3<br />

Σ1<br />

Σ2<br />

Ω<br />

1.4<br />

1.2<br />

1<br />

0.8<br />

0.6<br />

0.4<br />

0.2<br />

-1 -0.5 0.5 1


Ω grad( 1<br />

r )<br />

<br />

<br />

2<br />

<br />

dxdydz y J = S<br />

Sea r := r = x2 + y2 + z2 . En primer lugar,<br />

<br />

Sean I =<br />

1 1 grad( r r )ds.<br />

grad( 1 −grad r<br />

) =<br />

r r2 = −r<br />

r 3<br />

donde se ha utilizado que grad r = − r.<br />

Es inmediato comprobar, aunque no lo pide el enunciado, que<br />

r<br />

1<br />

r es armónico en R3 <br />

<br />

−{0}. Ahora, grad( 1<br />

r )<br />

<br />

<br />

2<br />

A) Cálculo de I.<br />

<br />

I =<br />

1<br />

2 dxdydz<br />

= 1<br />

r4 1 =<br />

(x2 +y2 +z2 ) 2 y 1 1 −r<br />

grad( ) = r r r4 = −(x,y,z)<br />

(x2 +y2 +z2 ) 2 .<br />

Ω (x2 + y2 + z2 )<br />

Por la geometría del recinto y la forma de la función subintegral, lo más adecuado es hacer un cambio<br />

a esféricas.<br />

x = r cos θsenφ ; y = rsenθsenφ ; z = r cos φ<br />

Obviamente, el ángulo θ variará entre 0 y 2π y el ángulo φ entre 0 y π/4. En cuanto a r,<br />

éste variará entre r0 y 1 donde r0 se puede calcular expresando la ecuación del plano z = 1/2 en<br />

coordenadas esféricas<br />

r0 cos φ = 1<br />

2 ⇒ r0 = 1<br />

2 cos φ<br />

Por ello,<br />

I =<br />

<br />

1<br />

2 dxdydz =<br />

2π<br />

Ω (x2 + y2 + z2 =<br />

) 0<br />

π 1<br />

π<br />

4<br />

1<br />

4<br />

2π dφsenφ dr = 2π<br />

1<br />

0<br />

r2 0<br />

2 cos φ<br />

π<br />

π<br />

= 2π<br />

4<br />

senφ (2 cos φ − 1) dφ = 4π<br />

4<br />

=<br />

0<br />

2π<br />

0<br />

1<br />

2 sen2φ | π/4<br />

0 −2π cos φ | 0 π/4= π<br />

π 1<br />

4<br />

dθ dφ<br />

1<br />

0<br />

2 cos φ<br />

dφsenφ<br />

√ <br />

2 − 1<br />

<br />

− 1<br />

r<br />

1<br />

1<br />

2 cos φ<br />

senφ cos φdφ − 2π<br />

1<br />

r 4 r2 senφdr =<br />

=<br />

π<br />

4<br />

0<br />

senφdφ =<br />

B) Cálculo de J. La integral de superficie se calcula descomponiéndola en las integrales correspondientes<br />

a Σ1, Σ2 y Σ3 (ver figura).<br />

<br />

J = −<br />

S<br />

<br />

r<br />

ds = −<br />

r4 S<br />

<br />

r<br />

· nds = −<br />

r4 Σ1<br />

<br />

r<br />

· nds+<br />

r4 Σ2<br />

donde n es el vector unitario normal saliente a la superficie.<br />

B.1. Como en la superficie esférica se tiene n = r y r = 1 entonces<br />

<br />

Σ1<br />

<br />

r<br />

· nds=<br />

r4 Σ1<br />

1 1<br />

ds =<br />

r2 12 <br />

ds =<br />

Σ1<br />

Área(Σ1)<br />

<br />

=<br />

<br />

r<br />

· nds+<br />

r4 Σ3<br />

G<br />

<br />

<br />

<br />

∂Φ<br />

∂φ<br />

× ∂Φ<br />

∂θ<br />

<br />

r<br />

· nds<br />

r4 <br />

<br />

<br />

dφdθ<br />

donde (x, y, z) = Φ(u, v), (u, v) ∈ G es una parametrización de Σ1. Como parametrización de Σ1 se<br />

puede tomar<br />

x = cos θsenφ ; y = senθsenφ ; z = cos φ<br />

<br />

θ ∈ [0, 2π] ; φ ∈ 0, π<br />

<br />

4


para la cual ∂Φ<br />

∂φ<br />

× ∂Φ<br />

∂θ<br />

<br />

Σ1<br />

<br />

<br />

= senφ luego<br />

2π<br />

r<br />

· nds = dθ<br />

r4 0<br />

π/4<br />

π/4<br />

senφdφ = 2π senφdφ = π(2 − √ 2)<br />

0<br />

B.2. En Σ2 los vectores normales a la superficie son perpendiculares al campo r<br />

r 4 (nótese que es<br />

un campo central) con lo que <br />

B.3. En Σ3, n =(0, 0, −1) luego<br />

<br />

<br />

r<br />

· nds = −<br />

r4 y ahora, parametrizando Σ3 en la forma<br />

<br />

<br />

se tiene, al ser ∂Φ<br />

<br />

Σ3<br />

∂φ<br />

× ∂Φ<br />

∂θ<br />

<br />

<br />

= 1,<br />

Σ3<br />

<br />

r<br />

· nds = −<br />

r4 Σ2<br />

= − 1<br />

<br />

2<br />

1/2<br />

= −<br />

0<br />

Por ello, se tiene finalmente<br />

Σ2<br />

r<br />

· nds = 0<br />

r4 0<br />

z<br />

Σ2 (x2 + y2 + z2 )<br />

x = x ; y = y ; z = 1<br />

2 ; x2 + y 2 ≤ 1<br />

4<br />

J = −<br />

z<br />

(x 2 + y 2 + z 2 )<br />

x 2 +y 2 ≤ 1<br />

4<br />

ρ<br />

ρ 2 + 1<br />

4<br />

<br />

2 ds = −<br />

x 2 +y 2 ≤ 1<br />

4<br />

1<br />

<br />

x2 + y2 1 2 dxdy = −<br />

+ 4<br />

1<br />

2<br />

2 dρ = −π<br />

con lo que se compueba que, efectivamente, I = J.<br />

2 ds<br />

1<br />

2<br />

<br />

x2 + y2 1 2 dxdy =<br />

+<br />

2π<br />

<br />

π(2 − √ <br />

2) + 0 − π = π( √ 2 − 1)<br />

0<br />

dθ<br />

4<br />

1/2<br />

0<br />

ρ<br />

<br />

ρ2 1 2 dρ =<br />

+ 4


AMPLIACIÓN DE CÁLCULO (Curso 2001/2002) Examen Final de Septiembre 9.9.02<br />

Solución del<br />

PROBLEMA 1<br />

(4 puntos)<br />

Sea f : (0, ∞) → R una función de clase uno, sea a ∈R 3 un vector fijo no nulo y sea el campo<br />

F :R 3 \ {0} → R 3 definido por<br />

F(r) =f(r) a × r<br />

donde r = xi + yj + zk y r = x 2 + y 2 + z 2 .<br />

Se pide:<br />

a) Calcular la divergencia del campo F y simplificar el resultado.<br />

b) Sea Γ la curva definida por las ecuaciones<br />

Calcular<br />

x 2 + y 2 + z 2 = 1<br />

<br />

I =<br />

Γ<br />

x − z = 0<br />

a × r<br />

dr<br />

r2 donde la orientación de Γ es tal que su proyección sobre el plano xy está orientada en sentido contrario<br />

a las agujas del reloj.<br />

c) Se considera el tetraedro T de vértices A ≡ (0, 0, 1/2), B ≡ (0, 1, 0), C ≡ (1, 0, 0) y V ≡ (1, 1, 1).<br />

Calcular la integral<br />

<br />

J = r 2 (a × r) dσ<br />

Σ<br />

siendo a = (1, −1, 0) y Σ la superficie formada por las tres caras de T que pasan por el punto V ,<br />

orientada según la normal saliente a T .<br />

Respuesta:<br />

a) En primer lugar calculamos<br />

a × r = (a2z − a3y, a3x − a1z, a1y − a2x)<br />

y ahora, utilizando que si U es un campo escalar y V un campo vectorial se cumple div (UV) = grad<br />

U · V + Udiv V, se tiene<br />

pues:<br />

div (f(r)a × r) = grad (f(r)) · (a × r) + f(r) div (a × r) =<br />

= f ′ (r) gradr · (a × r) + f(r) div (a × r) =<br />

= f ′ (r) r<br />

· (a × r) + f(r) div (a × r) = 0<br />

r<br />

- gradr = r<br />

r<br />

- div (a × r) = 0 al ser su componente i-ésima independiente de la i-ésima variable, i = 1, 2, 3.<br />

- el producto mixto r · (a × r) es nulo al ser los vectores linealmente dependientes.<br />

Por ello, para cualquier f el campo F es solenoidal en su dominio de definición.<br />

b) Se puede proceder de dos formas:


.1) Calculando drectamente la integral curvilínea. Para parametrizar Γ calculamos su<br />

proyección sobre el plano xy eliminando la z entre las dos ecuaciones que definen la curva. Esta<br />

proyección es 2x 2 + y 2 = 1 y entonces la curva se puede escribir<br />

Parametrizamos la proyección en la forma<br />

2x 2 + y 2 = 1<br />

z = x<br />

x = 1<br />

√ 2 cos t ; y = sen t , t ∈ [0, 2π]<br />

(nótese que lleva la orientación que indica el enunciado), y entonces una posible parametrización<br />

para Γ viene dada por<br />

x = 1<br />

√ 2 cos t<br />

y = sen t<br />

z = 1<br />

√ 2 cos t con t ∈ [0, 2π]<br />

Ahora, como r 2 = x 2 + y 2 + z 2 vale uno en todos los puntos de Γ, la expresión de I se puede<br />

simplificar. En efecto,<br />

<br />

<br />

a × r<br />

I = dr =<br />

r2 0<br />

Γ<br />

Γ<br />

<br />

(a × r) · dr = (a2z − a3y)dx + (a3x − a1z)dy + (a1y − a2x) dz<br />

Γ<br />

y entrando con la parametrión anterior se tiene<br />

2π <br />

a2 cos t<br />

sen t<br />

I = √ − a3sen t) − √ + (<br />

2 2<br />

a3 cos t<br />

√ −<br />

2 a1 cos t<br />

√ ) cos t + (a1sen t −<br />

2 a2<br />

<br />

cos t sen t<br />

√ − √ dt<br />

2 2<br />

Por simetría, la integral 2π<br />

0 cos tsentdt vale cero mientras que 2π<br />

0 cos2 dt = 2π<br />

0 sen2tdt = 4 π/2<br />

cos 0<br />

2 dt<br />

por lo que se puede escribir<br />

<br />

2a3<br />

I = 4 √2 − 2a1<br />

π/2<br />

√2 cos 2 dt = √ 2π (a3 − a1)<br />

0<br />

b.2) Aplicando el teorema de Stokes. Como rot(a × r) = 2a tiene una expresión muy simple<br />

y la curva Γ es la frontera de la superficie (también muy sencilla) S correspondiente a la porción del<br />

plano x = z contenido en el interior de la esfera x 2 + y 2 + z 2 = 1 parece razonable aplicar el teorema<br />

de Stokes para calcular I. Así,<br />

<br />

I =<br />

Γ<br />

<br />

<br />

a × r<br />

dr = (a × r) · dr = 2a · dσ<br />

r2 Γ<br />

S<br />

donde S está orientada por el vector normal unitario n = 1<br />

√ 2 (−1, 0, 1). De esta forma<br />

<br />

I = 2 a · dσ =2 a ·<br />

S<br />

S<br />

1<br />

√ (−1, 0, 1) dσ =<br />

2 √ <br />

2(a3 − a1) dσ =<br />

S<br />

√ 2(a3 − a1) Área(S)<br />

y como S es un círculo de radio 1 se tiene Área(S) = π y se llega al resultado final I = √ 2π (a3 − a1).<br />

c) El calcular J integrando sobre Σ parece complicado debido a la complejidad de dicha superficie,<br />

por lo que se busca otra alternativa para su cálculo. Puesto que el campo r 2 (a × r) tiene la forma<br />

del primer apartado, es solenoidal en R 3 . De esta forma, y mediante el uso del teorema de Gauss, J


es igual a la integral del campo en cuestión sobre cualquier superficie que se apoye sobre el borde de<br />

Σ y esté orientada coherentemente con dicho borde. Por ejemplo, podemos elegir por ejemplo la cara<br />

S del tetraedro T correspondiente a los vértices A, B y C. Esta cara está situada sobre el plano<br />

x + y + 2z = 1<br />

y se debe considerar la orientación correspondiente al vector normal unitario n = 1<br />

√ 6 (1, 1, 2).<br />

Entonces<br />

<br />

J = r<br />

Σ<br />

2 <br />

(a × r)dσ = r<br />

Σ<br />

2 (a × r) · ndσ =<br />

= 1<br />

<br />

2 2 2 √ x + y + z<br />

6 Σ<br />

=<br />

((1, −1, 0)×(x, y, z)) · (1, 1, 2)dσ =<br />

2<br />

<br />

2 2 2 √ x + y + z<br />

6<br />

(x + y − z)dσ<br />

Σ<br />

Ahora parametrizamos S en la forma estandar, es decir,<br />

Entonces<br />

y se tiene<br />

x = x<br />

Φ : y = y<br />

(1 − x − y)<br />

J = 2<br />

=<br />

<br />

√<br />

6<br />

<br />

=<br />

<br />

z = h(x, y) = 1<br />

2<br />

M<br />

M<br />

<br />

<br />

<br />

∂Φ<br />

∂x<br />

× ∂Φ<br />

∂y<br />

<br />

x 2 + y 2 + 1<br />

4<br />

<br />

<br />

<br />

=<br />

<br />

1 +<br />

; (x, y) ∈ M = {(x, y) : 0 ≤ x ≤ 1 ; y = 1 − x}<br />

<br />

2 ∂h<br />

+<br />

∂x<br />

2 ∂h<br />

=<br />

∂y<br />

√ 6<br />

(x + y − 1<br />

√<br />

6<br />

(1 − x − y) ) dxdy =<br />

2 2<br />

(1 − x − y)2<br />

M<br />

x 2 + y 2 + 1<br />

<br />

(1 − x − y)2 (x + y −<br />

4 1<br />

(1 − x − y) )dxdy = · · · =<br />

2<br />

<br />

15<br />

8 x3 + 15<br />

8 y3 + 21<br />

8 xy2 + 21<br />

8 x2y − 11<br />

8 x2 − 11<br />

8 y2 − 7<br />

<br />

5 5 1<br />

xy + x + y − dxdy<br />

4 8 8 8<br />

<br />

Ahora hay que evaluar 10 integrales del tipo<br />

<br />

al ser M simétrico respecto de la recta y = x, entonces<br />

<br />

J =<br />

M<br />

15<br />

4 x3 + 21<br />

4 x2 y − 11<br />

2<br />

M xpyqdx. En primer lugar podemos utilizar que,<br />

M xpyq <br />

dx = M xqypdx (*) con lo que<br />

4 x2 − 7<br />

4<br />

<br />

5 1<br />

xy + x − dxdy<br />

4 8<br />

y hay que calcular 6 integrales. Estas se pueden calcular de forma conjunta evaluando, para p y q<br />

genéricos, la integral<br />

<br />

x<br />

M<br />

p y q dxdy =<br />

=<br />

1<br />

0<br />

x p dx<br />

1−x<br />

0<br />

y q dy =<br />

1<br />

1<br />

1<br />

β(p + 1, q + 2) =<br />

q + 1 q + 1<br />

0<br />

p 1<br />

x<br />

q + 1 (1 − x)q+1dx = 1<br />

q + 1<br />

Γ(p + 1)Γ(q + 2)<br />

Γ(p + q + 3)<br />

1<br />

x<br />

0<br />

p (1 − x) q+1 dx =<br />

1 p!(q + 1)!<br />

=<br />

q + 1 (p + q + 2)! =<br />

p!q!<br />

(p + q + 2)!<br />

Nótese que esta fórmula demuestra la la validez de la igualdad (*) que se estableció anteriormente<br />

por un argumento de simetría.


Así,<br />

y entonces<br />

<br />

<br />

M<br />

<br />

M<br />

M<br />

x 3 dxdy = 3!<br />

5!<br />

x 2 dxdy = 2!<br />

4!<br />

xdxdy = 1<br />

3!<br />

J = 15<br />

4<br />

1<br />

20<br />

+ 21<br />

4<br />

= 1<br />

20 ;<br />

<br />

1<br />

=<br />

12 ;<br />

<br />

1<br />

=<br />

6 ;<br />

<br />

1<br />

60<br />

y operando se llega al resultado final J = 19<br />

160 .<br />

− 11<br />

4<br />

M<br />

M<br />

M<br />

x 2 ydxdy = 2<br />

5!<br />

xydxdy = 1<br />

4!<br />

dxdy = 1<br />

2<br />

1 7 1 5 1 1 1<br />

− + −<br />

12 4 24 4 6 8 2<br />

= 1<br />

60<br />

= 1<br />

24


AMPLIACIÓN DE CÁLCULO (Curso 2002/2003) Examen Final de Junio 23.06.03<br />

Solución del<br />

PROBLEMA 1<br />

(4 puntos)<br />

Sean φ : R 3 → R un campo escalar de clase C 2 y Ω ⊂ R 3 un sólido limitado por una superficie S<br />

regular y orientada según la normal saliente n.<br />

Se pide:<br />

1) Calcular div (φ grad φ).<br />

2) Aplicando el teorema de Gauss, transformar la integral de superficie<br />

<br />

S<br />

φ ∂φ<br />

∂n dσ<br />

en una integral triple extendida a Ω. (Se recuerda que ∂φ<br />

∂n<br />

= grad φ · n).<br />

3) Determinar la expresión en Ω de todos los campos vectoriales F : R 3 → R 3 , F ∈ C 1 , que sean<br />

irrotacionales, solenoidales y con componente normal sobre S igual a cero en todos los puntos.<br />

4) Sea S una superficie regular orientable que encierra un volumen acotado Ω ⊂ R 3 ¿Queda unívocamente<br />

determinado en Ω un campo vectorial G : R 3 → R 3 , G ∈ C 1 , si se conocen rot G y div G<br />

en R 3 y la componente normal de G sobre todos los puntos de S? Razonar la respuesta.<br />

Respuesta:<br />

1) Utilizando las definiciones de gradiente de un campo escalar y de divergencia de un campo vectorial,<br />

se tiene:<br />

div (φ grad φ) = ∂<br />

<br />

φ<br />

∂x<br />

∂φ<br />

<br />

+<br />

∂x<br />

∂<br />

<br />

φ<br />

∂y<br />

∂φ<br />

<br />

+<br />

∂y<br />

∂<br />

<br />

φ<br />

∂z<br />

∂φ<br />

<br />

∂z<br />

<br />

∂φ ∂φ<br />

=<br />

+ φ<br />

∂x ∂x<br />

∂2 <br />

φ ∂φ ∂φ<br />

+<br />

+ φ<br />

∂x2 ∂y ∂y<br />

∂2 <br />

φ ∂φ ∂φ<br />

+<br />

∂y2 ∂z ∂z<br />

2 2 2 2 ∂φ ∂φ ∂φ ∂ φ<br />

= + + + φ<br />

∂x ∂y ∂z ∂x2 + ∂2φ ∂y2 + ∂2φ ∂z2 <br />

= grad φ 2 + φ ∆ φ ,<br />

donde . representa la norma dos y ∆ es el operador laplaciano.<br />

+ φ ∂2 φ<br />

∂z 2<br />

2) En las condiciones establecidas en el enunciado es posible aplicar el teorema de Gauss a la integral<br />

de superficie dada. Para ello basta tener en cuenta que<br />

<br />

S<br />

φ ∂φ<br />

dσ =<br />

∂n<br />

<br />

S<br />

φ grad φ · n dσ .<br />

La aplicación del teorema de Gauss a esta integral da lugar a la igualdad<br />

<br />

φ ∂φ<br />

<br />

dσ =<br />

∂n<br />

div [φ grad φ] dV .<br />

S<br />


siendo Ω el volumen limitado por la superficie cerrada S. Finalmente, teniendo en cuenta el resultado<br />

obtenido en el apartado anterior, se obtiene la igualdad:<br />

<br />

φ ∂φ<br />

<br />

dσ = grad φ<br />

∂n 2 <br />

dV + φ ∆ φ dV .<br />

S<br />

Ω<br />

3) Que el campo F sea irrotacional en R 3 (que es un dominio estrellado) significa que es el gradiente<br />

de un potencial escalar. Es decir, existe un campo escalar φ : R 3 → R tal que<br />

F = grad φ (en R 3 ) .<br />

Además, F es solenoidal (es decir, tiene divergencia nula), por lo tanto:<br />

div (F) = div (grad φ) = ∆ φ = 0 (en R 3 ) ,<br />

donde ∆ vuelve a representar el operador laplaciano.<br />

Podemos concluir, por tanto, que todo campo irrotacional y solenoidal en R3 es el gradiente de<br />

un campo escalar armónico (que satisface la ecuación de Laplace ∆ φ = 0 en R3 ).<br />

Por otra parte, puesto que F = grad φ, su componente normal sobre la superficie cerrada S viene<br />

dada por grad φ · n = (∂φ/∂n), siendo n un vector dirigido según la normal saliente a S en cada<br />

punto. Ahora bien, esta componente normal es cero y, en consecuencia, la integral de superficie<br />

<br />

S<br />

φ F · n dσ =<br />

<br />

S<br />

φ ∂φ<br />

∂n dσ<br />

debe también anularse.<br />

El resultado final obtenido en el apartado 2) anterior nos permite entonces concluir que<br />

<br />

φ ∂φ<br />

<br />

dσ = grad φ<br />

∂n 2 <br />

dV + φ ∆ φ dV = 0 .<br />

S<br />

Ω<br />

De aquí, teniendo en cuenta que (como se acaba de demostrar) φ es un campo escalar armónico<br />

(∆φ = 0), resulta <br />

Ω<br />

grad φ 2 dV = 0<br />

y como grad φ 2 es una función continua no negativa, obtenemos finalmente<br />

grad φ = 0 en Ω ⇐⇒ F = grad φ = 0 en Ω .<br />

Así pues, el único campo vectorial que satisface todas las condiciones establecidas en este tercer<br />

apartado es el campo nulo en Ω, que deriva de un potencial escalar constante.<br />

4) Se trata de determinar si en el volumen Ω limitado por la superficie cerrada S, el conocimiento del<br />

rotacional y la divergencia de un campo vectorial G, junto con los valores de su componente normal<br />

sobre S, determina dicho campo de forma única.<br />

Para ello supongamos que existe otro campo vectorial H tal que su rotacional, su divergencia y<br />

los valores de su componente normal sobre S, coinciden con los de G.<br />

En estas condiciones, la diferencia entre ambos campos vectoriales, F = G − H, es a su vez un<br />

campo vectorial con las siguientes propiedades:<br />

rotF = rotG − rotH = 0, divF = divG − divH = 0 .<br />

Además, la componente normal de F sobre S es también nula.<br />

Según se ha aprobado en el apartado anterior, el único campo en Ω que cumple estas condiciones<br />

es el idénticamente nulo. Es decir , G y H tienen que ser iguales, de lo que concluimos que, en efecto,<br />

las condiciones dadas sobre G en este apartado 4) lo determinan unívocamente en Ω.<br />

Ω<br />


AMPLIACIÓN DE CÁLCULO (Curso 2002/2003) Examen Final de Junio 23.06.03<br />

Solución del<br />

PROBLEMA 2<br />

(3 puntos)<br />

Se considera el astroide de ecuación cartesiana:<br />

Se pide:<br />

1) Calcular su longitud.<br />

2) Calcular el área que encierra.<br />

x 2/3 + y 2/3 = a 2/3 , (a > 0).<br />

Respuesta:<br />

1) La curva Γ en cuestión es cerrada y presenta simetría respecto de los dos<br />

ejes de coordenadas. Su aspecto es el que muestra la figura.<br />

Para calcular la longitud de la curva se debe encontrar una parametrización<br />

de la misma. Para ello, se puede utilizar el hecho de que sen 2 t + cos 2 t = 1,<br />

obteniendo así la parametrización r(t) = (x(t), y(t)) dada por<br />

Se tiene entonces<br />

L =<br />

2π<br />

= 3a<br />

= 3a<br />

0<br />

2π<br />

r ′ (t) dt =<br />

0<br />

2π<br />

0<br />

x = acos 3 t<br />

y = asen 3 t, t ∈ [0, 2π]<br />

2π<br />

0<br />

<br />

2π<br />

x ′ (t) 2 + y ′ (t) 2dt = 3a<br />

0<br />

2π<br />

sen 2 t cos 2 t(sen 2 t + cos 2 t)dt = 3a<br />

π<br />

2<br />

|sent| |cos t| dt = 12a<br />

0<br />

0<br />

√ sen 2 t cos 4 t + sen 4 t cos 2 tdt =<br />

√ sen 2 t cos 2 tdt =<br />

sent cos tdt = 6asen 2 t | π<br />

2<br />

0 = 6a<br />

donde en la antepenúltima igualdad se ha utilizado la periodicidad de la función |sent| |cos t| (nótese<br />

que si se comete el error de escribir que √ sen2t cos2 t es igual a sent cos t, se obtiene que la longitud<br />

es nula, lo cual, evidentemente, no tiene sentido).<br />

2) La parametrización anterior puede utilizarse para el cálculo del área mediante una integración<br />

curvilínea haciendo uso de la siguiente expresión<br />

A = 1<br />

2<br />

= 1<br />

2<br />

= 3<br />

2 a2<br />

<br />

xdy − ydx =<br />

Γ<br />

1<br />

2π<br />

[x(t)y<br />

2 0<br />

′ (t) − y(t)x ′ (t)] dt =<br />

2π<br />

[x(t)y<br />

0<br />

′ (t) − y(t)x ′ (t)] dt = 3<br />

2 a2<br />

2π<br />

0<br />

2π<br />

sen 2 t cos 2 tdt = 6a 2<br />

π<br />

2<br />

sen 2 t cos 2 2 1<br />

tdt = 6a<br />

0<br />

0<br />

sen 2 t cos 4 t + sen 4 t cos 2 t dt =<br />

2 β(3<br />

2<br />

3 3<br />

, ) = · · · =<br />

2 8 πa2<br />

Nótese que a debe tener dimensiones de longitud para que x e y representen una posición. Se<br />

comprueba que los resultados obtenidos son dimensionalmente correctos, puesto que L y A tienen<br />

dimensiones de longitud y área respectivamente.


AMPLIACIÓN DE CÁLCULO (Curso 2002/2003) Examen Final de Septiembre 8.09.03<br />

PROBLEMA 1 (4 puntos) Contestar razonadamente a cada una de las siguientes cuestiones: (las<br />

cuestiones 1, 2 y 3 no son actual del programa)<br />

4) Sea Ω = R 2 \ {(±1, 0)} y sean P, Q : Ω → R dos funciones de clase C 1 tales que<br />

y <br />

Γ1<br />

∂P<br />

∂y<br />

∂Q<br />

(x, y) = (x, y) , (x, y) ∈ Ω<br />

∂x<br />

P dx + Qdy = 4 ,<br />

<br />

Γ2<br />

P dx + Qdy = 7 ,<br />

en donde Γ1 es el cuadrado de vértices (±3, ±3) con orientación positiva y Γ2 es la circunferencia<br />

(x − 1) 2 + y2 = 1 con orientación positiva.<br />

Se pide calcular P dx+Qdy siendo Γ cada una de las siguientes curvas, con orientación positiva:<br />

Γ<br />

a) Circunferencia (x + 1) 2 + y 2 = 1.<br />

b) Elipse 4x 2 + y 2 = 1.<br />

c) Cuadrado de vértices (0, ±1), (2, ±1).<br />

Respuesta:<br />

Aplicaremos el siguiente resultado (consecuencia del teorema de Green):<br />

Sea F = (P, Q) un campo vectorial de clase 1 en un dominio Ω (múltiplemente conexo) de R2 y<br />

tal que ∂Q ∂P<br />

= . Sea R una región cerrada en Ω cuya frontera está formada por un número finito<br />

∂x ∂y<br />

de curvas de Jordan Γ, γ1, . . . , γm de forma que Γ contiene a γ1, . . . , γm en su interior. Entonces<br />

<br />

Γ<br />

P dx + Qdy =<br />

recorridas las curvas con la misma orientación.<br />

m<br />

<br />

k=1<br />

γk<br />

P dx + Qdy .


Llamemos C a la circunferencia del apartado a), E a la elipse de<br />

b) y G al cuadrado de c) (véase la figura); apliquemos el resultado<br />

enunciado:<br />

a) Consideramos la región cuya frontera exterior es el cuadrado<br />

Γ1 y cuyas fronteras interiores son las circunferencias Γ2 y C,<br />

se tiene:<br />

<br />

Γ1<br />

<br />

=<br />

Γ2<br />

<br />

+ ⇒ P dx + Qdy = 4 − 7 = −3 .<br />

C C<br />

b) En la región encerrada por la elipse el campo es conservativo,<br />

entonces P dx + Qdy = 0.<br />

E<br />

c) Consideramos la región cuya frontera exterior es G y cuya<br />

frontera interior es Γ2, entonces<br />

<br />

G<br />

<br />

P dx + Qdy =<br />

Γ2<br />

P dx + Qdy = 7 .<br />

Γ1<br />

4<br />

3<br />

2<br />

1<br />

-4 -3 -2 -1 1 2 3 4<br />

Γ2<br />

-1<br />

-2<br />

-3<br />

-4<br />

E<br />

C G


AMPLIACIÓN DE CÁLCULO (Curso 2002/2003) Examen Final de Septiembre 8.09.03<br />

Solución del<br />

PROBLEMA 2<br />

(3 puntos)<br />

Se considera un elipsoide en el que las longitudes de los semiejes son a, b y c. A cada punto de la<br />

superficie del elipsoide se le asocia la distancia, d(x, y, z), del centro del elipsoide al plano tangente al<br />

elipsoide en ese punto. Calcúlese la integral de la función 1/d 2 (x, y, z) sobre la superficie del elipsoide.<br />

Respuesta: Puesto que la solución del problema no depende del sistema de referencia elegido,<br />

tomaremos aquel en el que el centro del elipsoide está en el origen y sus ejes coinciden con los ejes<br />

cartesianos. Así, la ecuación del elipsoide es<br />

x 2<br />

a<br />

b<br />

y2 z2<br />

+ + 2 2<br />

= 1<br />

c2 En primer lugar calcularemos el plano tangente al elipsoide en un punto genérico (x, y, z) de su<br />

superficie. Puesto que la ecuación del elipsoide es F (x, y, z) = 0 con F (x, y, z) = x2<br />

a2 + y2<br />

b2 + z2<br />

c2 − 1,<br />

un vector normal al plano viene dado por gradF (x, y, x) = 2x<br />

a2 , 2y<br />

b2 , 2z<br />

c2 <br />

, o dividiendo por 2, por<br />

. De esta forma la ecuación de los puntos (X, Y, Z) del plano es<br />

x<br />

a 2 , y<br />

b 2 , z<br />

c 2<br />

x<br />

y<br />

z<br />

(X − x) + (Y − y) + (Z − z) = 0<br />

a2 b2 c2 es decir,<br />

x y z x2 y2 z2<br />

X + Y + Z = + +<br />

a2 b2 c2 a2 b2 c2 y teniendo en cuenta que (x, y, z) pertenece al elipsoide, se tiene finalmente<br />

x y z<br />

X + Y + Z = 1<br />

a2 b2 c2 La distancia de un punto (x0, y0, z0) a un plano AX + BY + CZ + D = 0 es<br />

d = |Ax0 + By0 + Cz0 + D|<br />

√ A 2 + B 2 + C 2<br />

por lo que al ser (x0, y0, z0) = (0, 0, 0), la función distancia pedida viene dada por<br />

d(x, y, z) =<br />

|−1|<br />

√ A 2 + B 2 + C 2 =<br />

De esta forma, la integral pedida es<br />

<br />

1<br />

I =<br />

d2 <br />

dσ =<br />

(x, y, z)<br />

S<br />

S<br />

x 2<br />

a<br />

1<br />

<br />

x2 a4 + y2<br />

b4 + z2<br />

c4 b<br />

c 4<br />

y2 z2<br />

+ + 4 4<br />

Utilizando la paridad de la función subintegral respecto de x, y y z y la simetría del recinto<br />

respecto de los tres planos coordenados se tiene<br />

2 x y2 z2<br />

I = 8 + +<br />

a4 b4 c4 <br />

dσ<br />

donde S ′ es la porción de elipsoide comprendida en el primer octante.<br />

S ′<br />

.<br />

<br />


Procederemos parametrizando S ′ en la forma<br />

(x, y, z) = Φ(θ, φ), (θ, φ) ∈ Ω<br />

y entonces la integral se calcula como una integral doble en la forma<br />

<br />

1<br />

I = 8<br />

(d(Φ(θ, φ))) 2<br />

<br />

<br />

<br />

<br />

∂Φ ∂Φ <br />

(θ, φ) × (θ, φ) <br />

∂θ ∂φ dθdφ<br />

La parametrización más cómoda para S ′ es<br />

Ω<br />

x = a cos θsenφ<br />

y = bsenθsenφ<br />

z = c cos φ, θ ∈ [0, π<br />

<br />

], φ ∈ 0,<br />

2 π<br />

<br />

2<br />

Operando se obtiene<br />

<br />

<br />

<br />

<br />

∂Φ ∂Φ <br />

(θ, φ) × (θ, φ) <br />

∂θ ∂φ = b2c2 cos2 θsen4φ + a2c2sen2θsen4φ + a2b2 cos2 φsen2 =<br />

φ =<br />

<br />

cos2 θsen2φ abc |senφ|<br />

a2 + sen2θsen2φ b2 + cos2 φ<br />

c2 Por otro lado,<br />

y por ello,<br />

<br />

I = 8<br />

S ′<br />

= 8abc<br />

x 2<br />

a<br />

y2 z2<br />

+ + 4 b4 c4 = cos2 θsen2φ a2 + sen2 θsen 2 φ<br />

b 2<br />

+ cos2 φ<br />

c 2<br />

2 x y2 z2<br />

+ +<br />

a4 b4 c4 π π <br />

2<br />

2 2 2 cos θsen φ<br />

dσ = 8abc dθ<br />

0 0 a2 + sen2θsen2φ b2 + cos2 φ<br />

c2 3<br />

2<br />

|senφ| dφ =<br />

π π <br />

2<br />

2 2 2 cos θsen φ<br />

dθ<br />

0 0 a2 + sen2θsen2φ b2 + cos2 φ<br />

c2 3<br />

2<br />

senφdφ<br />

donde se ha usado que el seno es no negativo en 0, π<br />

<br />

. Las integrales obtenidas no pueden resolverse<br />

2<br />

por los métodos habituales de integración, por lo que el resultado se deja indicado.


AMPLIACIÓN DE CÁLCULO (Curso 2003/2004) Examen Final de Febrero 5.02.04<br />

Solución del<br />

PROBLEMA 1<br />

(4 puntos)<br />

1) Sea Ω un dominio acotado de R3 limitado por una superficie regular Σ orientada según la<br />

normal saliente. Sea F :R3 → R3 un campo vectorial regular del que se sabe que admite tanto un<br />

potencial escalar como un<br />

<br />

potencial<br />

<br />

vector y sea G un potencial vector cualquiera de F. Encontrar<br />

una expresión para I =<br />

Σ<br />

recinto Ω.<br />

2) Calcular las constantes α, β,γ y δ para que el campo<br />

F × G dσ que dependa únicamente del valor del módulo de F en el<br />

F(r) =(x + αz)i+(βx + y)j+(γy + δz)k, r =xi+yj+zk<br />

definido en R 3 , admita tanto un potencial escalar como un potencial vector. Para los valores de α, β,γ<br />

y δ obtenidos, calcular las constantes A, B, C, D y E para que el campo<br />

G(r) =(yz + Azx, Bxz + Cxy, Dxy + Eyz)<br />

sea un potencial vector de F.<br />

3) Comprobar la validez de la fórmula obtenida en el apartado 1 en el caso en que Σ es la esfera<br />

unidad orientada según la normal saliente y F y G son los campos calculados en el apartado 2.<br />

4) Sea Γ la curva de ecuaciones cartesianas<br />

x 2 + y 2 − 1 = 0<br />

x + y + z − 1 = 0<br />

orientada de forma que su proyección sobre el plano xy se recorre en sentido positivo. Se pide:<br />

4.1. Parametrizar Γ y calcular la circulación del campo G hallado en el apartado 2 sobre Γ.<br />

4.2. Comprobar el resultado aplicando de forma adecuada el teorema de Stokes.<br />

Respuesta: Se entregará esta hoja y, a lo sumo, una adicional.<br />

1. Puesto que se pide relacionar una integral de superficie de un campo vectorial con una integral<br />

triple parece razonable aplicar el teorema de Gauss. La regularidad de la superficie Σ y de los campos<br />

F y G involucrados hacen que se verifiquen las condiciones suficientes para su aplicabilidad. Así,<br />

<br />

<br />

F × Gdσ = div(F × G)dv,<br />

Σ<br />

Ahora se utilizará que div(F × G) = G·rotF − F·rotG, que al admitir F un potencial escalar en<br />

R 3 se sigue rotF = 0 en R 3 y, por definición de potencial vector, rotG = F obteniéndose<br />

<br />

Σ<br />

<br />

F × Gdσ =<br />

Ω<br />

Ω<br />

<br />

<br />

−F · Fdv = −<br />

Ω<br />

F 2 dv<br />

que es el resultado pedido.<br />

2. Para que F admita un potencial escalar en R 3 , es decir, para que F sea un campo conservativo<br />

en R 3 , es condición necesaria que se cumpla rotF = 0 en R 3 mientras que para que F admita un<br />

potencial vector es condición necesaria que se cumpla divF = 0 en R 3 (y ello es independiente<br />

del hecho de que R 3 sea un dominio estrellado). Imponiendo las condiciones anteriores se tiene<br />

rotF = (γ, α, β) = (0, 0, 0) de donde se sigue α = β = γ = 0 y divF = 1 + 1 + δ = 0 de donde<br />

se sigue δ = −2, es decir F = xi + yj − 2zk. Ahora bien, como R 3 es estrellado las condiciones


necesarias anteriores también son suficientes y por ello se puede afirmar que el campo F anterior<br />

verdaderamente admite un potencial escalar y un potencial vector.<br />

Forzando a que G sea un potencial vector de F se tiene<br />

∀(x, y, z) ∈ R 3 , rotG = (Dx + Ez − Bx, y + Ax − Dy, Bz + Cy − z) = (x, y, −2z)<br />

Como dicha igualdad es válida para todo (x, y, z) ∈ R 3 , G es un potencial vector de F si y sólo si se<br />

verifican las igualdades<br />

que se cumplen si y sólo si<br />

D − B = 1, E = 0, 1 − D = 1<br />

A = 0, B − 1 = −2, C = 0<br />

A = C = D = E = 0, B = −1<br />

con lo que el campo G pedido es G(r) = yzi − xzj.<br />

3) El enunciado del problema pide que se evalúen las integrales <br />

<br />

F × Gdσ y − Σ Ω F2 dv<br />

siendo F y G los campos del apartado 2 y Σ la esfera unidad con la orientación saliente, comprobando<br />

que en ambos casos se obtiene el mismo resultado.<br />

En primer lugar, F × G = − (2xz2 , 2yz2 , x2z + y2z). Utilizando que al ser Σ la esfera unidad su<br />

normal unitaria saliente es n = (x, y, z) se tiene<br />

<br />

<br />

<br />

2 2 2 2<br />

I = F × Gdσ = (F × G) ·ndσ = − 2xz , 2yz , x z + y z (x, y, z)dσ=<br />

Σ<br />

Σ<br />

Σ<br />

2 2 2 2 2 2 2 2<br />

= − 2x z + 2y z + x z + y z <br />

2 2 2 2<br />

dσ = − 3x z + 3y z dσ<br />

Σ<br />

<br />

Ahora se puede utilizar que, al ser la esfera invariante ante permutaciones de los ejes coordenados,<br />

Σ x2z2dσ = <br />

Σ y2z 2dσ con lo que I = −6 <br />

Σ x2z2dσ. Utilizando la paridad de la función subintegral<br />

y la simetría de Σ respecto de los tres planos coordenados se tiene finalmente I = −48 <br />

Σ ′ x2z2dσ donde Σ ′ es la porción de Σ contenida en el primer octante. Parametrizando Σ ′ en la forma<br />

(x, y, z) = Φ(θ, ϕ) = (cos θsenϕ, senθsenϕ, cos ϕ), θ ∈ [0, π/2], ϕ ∈ [0, π/2]<br />

<br />

<br />

y utilizando que ∂Φ<br />

<br />

<br />

(θ, ϕ) = senϕ se tiene<br />

Ahora,<br />

π<br />

2<br />

0<br />

∂θ<br />

(θ, ϕ) × ∂Φ<br />

∂ϕ<br />

<br />

I = −48<br />

π<br />

2<br />

0<br />

= −48<br />

Σ ′<br />

π<br />

2<br />

0<br />

cos 2 θdθ = 1<br />

2<br />

sen 3 ϕ cos 2 ϕdϕ = 1<br />

2<br />

x 2 z 2 π<br />

2<br />

dσ = −48<br />

cos 2 θdθ<br />

π<br />

2<br />

0<br />

sen 3 ϕ cos 2 ϕdϕ<br />

π<br />

2<br />

0<br />

β(2, 3<br />

2<br />

0<br />

Σ<br />

dθ<br />

π<br />

2<br />

0<br />

(cos θsenϕ) 2 (cos ϕ) 2 senϕdϕ =<br />

(1 + cos 2θ)dθ = 1<br />

π<br />

2<br />

dθ +<br />

2 0<br />

1<br />

2<br />

1 Γ(2)Γ(3/2)<br />

) =<br />

2 Γ(7/2)<br />

= · · · = 2<br />

15<br />

π<br />

2<br />

0<br />

cos 2θdθ = π π<br />

+ 0 =<br />

4 4<br />

y se obtiene finalmente −48 π 2 8π = − 4 15 5 .<br />

Por otro lado, utilizando argumentos de simetría y paridad análogos a los anteriores se tiene<br />

<br />

− F 2 <br />

dv = − (x 2 + y 2 + 4z 2 <br />

)dv = −6 z 2 <br />

dv = −48 z 2 dv<br />

Ω<br />

Ω<br />

Ω<br />

Ω ′


donde Ω ′ es la porción de la esfera unidad contenida en el primer octante. Haciendo un cambio a<br />

esféricas<br />

(x, y, z) = (r cos θsenϕ, rsenθsenϕ, r cos ϕ), θ ∈ [0, π/2], ϕ ∈ [0, π/2], r ∈ [0, 1]<br />

con jacobiano igual a r2senϕ se obtiene<br />

<br />

− F 2 <br />

dv = −48 z 2 dv = −48<br />

Ω<br />

= −48<br />

π<br />

2<br />

0<br />

dθ<br />

π<br />

2<br />

0<br />

Ω ′<br />

cos 2 ϕsenϕdϕ<br />

1<br />

0<br />

π<br />

2<br />

0<br />

dθ<br />

π<br />

2<br />

0<br />

dϕ<br />

r 4 dr = −48 π 1<br />

2 3<br />

1<br />

0<br />

r 2 cos 2 ϕr 2 senϕdr =<br />

cos 3 ϕ 0<br />

π/2<br />

1<br />

5<br />

= −48π<br />

2<br />

1 1<br />

3 5<br />

= −8π<br />

5<br />

con lo que se comprueba que el resultado es el mismo que en el caso de la integral de superficie.<br />

4)<br />

4.1. Γ es la curva cerrada resultante de la intersección del plano x + y + z = 1 con la superficie<br />

cilíndrica x 2 +y 2 = 1. Puesto que la proyección de la curva sobre el plano xy es precisamente la curva<br />

x 2 + y 2 = 1, se puede obtener una parametrización de Γ parametrizando dicha proyección mediante<br />

x = cos t, y = sent, t ∈ [0, 2π] (nótese que la orientación concuerda con la fijada en el enunciado) y<br />

entrando con dichos valores en z = 1 − x − y, es decir, la parametrización γ(t) dada por<br />

Por definición de integral curvilínea<br />

<br />

2π<br />

Gdr = G(γ(t)) · γ ′ (t)dt<br />

Γ<br />

=<br />

=<br />

0<br />

2π<br />

0<br />

2π<br />

0<br />

x = cos t, y = sent, z = 1 − cos t − sent, t ∈ [0, 2π]<br />

[sent(1 − cos t − sent)(−sent) − cos t(1 − cos t − sent)(cos t)] dt =<br />

(sen 2 t + cos 2 t)(−1 + cos t + sent) dt =<br />

2π<br />

0<br />

(−1 + cos t + sent)dt = −2π<br />

pues 2π<br />

0 cos tdt = 2π<br />

sentdt = 0. Nótese que si en vez de simplificar en la tercera igualdad se<br />

0<br />

desarrolla el producto se obtiene<br />

<br />

2π<br />

Gdr = (− cos 2 t − sen 2 t + cos 3 t + sen 3 t + cos tsen 2 t + cos 2 tsent)dt =<br />

Γ<br />

=<br />

0<br />

2π<br />

0<br />

(−1 + cos 3 t + sen 3 t + cos tsen 2 t + cos 2 tsent)dt = −2π<br />

pues 2π<br />

0 cos3 tdt = 2π<br />

sen 0 3tdt = 2π<br />

0 cos tsen2t = 2π<br />

0 cos2 tsentdt = 0.<br />

4.2. Aplicando el teorema de Stokes<br />

<br />

<br />

Gdr = rotGdσ = Fdσ<br />

Γ<br />

S<br />

S<br />

donde S es cualquier superficie regular que se apoya en Γ y cuya orientación es coherente con la de<br />

Γ. Por simplicidad elegimos como S la porción del plano x + y + z = 1 contenida dentro de Γ y<br />

orientada según el vector normal unitario n = (1, 1, 1)/ √ 3. Parametrizamos S en cartesianas en la<br />

forma<br />

(x, y, z) = Φ(x, y) = (x, y, f(x, y)), (x, y) ∈ D<br />

donde f(x, y) = 1 − x − y y D = {(x, y) : x2 + y2 ≤ 1}, y utilizando que<br />

<br />

<br />

<br />

<br />

∂Φ ∂Φ <br />

(x, y) × (x, y) <br />

∂x ∂y =<br />

<br />

2 2 <br />

∂f ∂f<br />

1 + + = 1 + (−1)<br />

∂x ∂y<br />

2 + (−1) 2 = √ 3


se obtiene<br />

<br />

Gdr =<br />

Γ<br />

=<br />

<br />

S <br />

F · ndσ = 1<br />

<br />

√ (x + y − 2z)dσ =<br />

3 S<br />

1<br />

<br />

√ (x + y − 2(1 − x − y))<br />

3 D<br />

√ 3dxdy =<br />

<br />

(−2 + 3x + 3y)dxdy = −2 dxdy = −2Area(D) = −2π<br />

D<br />

<br />

donde se ha utilizado que xdxdy = ydxdy = 0 puesto que la función subintegral es impar<br />

D D<br />

en x (resp. en y) y D es simétrico respecto del eje x = 0 (resp. y = 0). Nótese que, como debe ser,<br />

se ha obtenido el mismo resultado que en el apartado 4.1.<br />

D


AMPLIACIÓN DE CÁLCULO (Curso 2003/2004) Convocatoria de junio 10.06.04<br />

Solución del<br />

PROBLEMA 1<br />

(4 puntos)<br />

Este ejercicio consta de cuatro preguntas independientes, cada una de las cuales vale 1 punto.<br />

Este ejercicio consta de cuatro preguntas independientes, cada una de las cuales vale 1 punto.<br />

1) Decidir razonadamente para qué valores del parámetro real p converge la integral impropia:<br />

1<br />

0<br />

t<br />

dt.<br />

| log t| p<br />

La integral es impropia en ambos extremos del intervalo; el logaritmo es negativo en (0, 1), así pues<br />

| log t| = − log t. Hagamos el cambio de variable x = − log t, la integral se escribe<br />

∞<br />

I =<br />

0<br />

e −x<br />

x p e−x dx =<br />

∞<br />

y la homotecia y = 2x nos permite reconocer una gamma,<br />

I = 2 p−1<br />

∞<br />

0<br />

0<br />

e −2x x −p dx<br />

e −y y −p dy = 2 p−1 Γ(1 − p)<br />

que, como sabemos, es convergente para 1 − p > 0, es decir p < 1.<br />

2) Se considera el sólido limitado por las superficies x 2 + z 2 = y 2 , y = 0, y = 1, orientado por el<br />

vector normal exterior. Calcular el flujo del campo F(x, y, z) = (y 2 + z 2 , −y 2 , 2yz) a través de su<br />

superficie lateral cónica.<br />

El campo F es polinómico y solenoidal, div F(x, y, z) = −2y + 2y = 0. Llamemos V al sólido, Σ<br />

a la superficie lateral cónica y D a la superficie plana de ecuaciones x2 + z2 ≤ 1, y = 1. En virtud<br />

del teorema de Gauss se tiene:<br />

<br />

div F = F + F = 0,<br />

<br />

luego para calcular la integral pedida<br />

por el vector j,<br />

<br />

F = −<br />

Σ<br />

D<br />

V<br />

Σ<br />

Σ<br />

D<br />

F basta con calcular el flujo a través del disco D orientado<br />

<br />

F = − F · j = − (−y<br />

D<br />

D<br />

2 <br />

) dσ = dσ =<br />

D<br />

Área (D) = π.


AMPLIACIÓN DE CÁLCULO (Curso 2003/2004) Convocatoria de junio 10.06.04<br />

Solución del<br />

PROBLEMA 2<br />

(3 puntos)<br />

Sea Γ la curva definida por las ecuaciones cartesianas<br />

y F el campo vectorial dado por<br />

Se pide:<br />

2x − y = 0, z − x 3/2 = 0 (x, y, z ≥ 0)<br />

F(x, y, z) = (xy 2 , x 2 y, −zx 2 ) .<br />

1) Hallar la longitud del arco de Γ determinado por los puntos (0, 0, 0) y (1, 2, 1).<br />

2) Sea C la circulación de F sobre el arco de la curva Γ determinado por el punto (0, 0, 0) y un<br />

punto P arbitrario de la misma. Estudiar si el valor de C alcanza un máximo. En caso afirmativo,<br />

determinar las coordenadas del punto P correspondiente.<br />

3) Sea α una función de clase C 1 en R tal que α(0) = 0 y Λ una curva definida por las ecuaciones<br />

cartesianas<br />

2x − y = 0 , z − α(x) = 0 .<br />

Determinar las funciones α tales que se anule la circulación de F desde (0,0,0) hasta cualquier punto<br />

de la curva Λ.<br />

Respuesta: Se entregará esta hoja y, a lo sumo, una adicional.<br />

1) En primer lugar se debe parametrizar la curva Γ. Puesto que Γ está definida por la intersección<br />

de dos superficies dadas por ecuaciones del tipo f(x, y) = 0, g(x, z) = 0, es decir, en la primera no<br />

interviene la z y en la segunda no interviene la y, la parametrización de la curva se puede llevar a<br />

cabo de forma trivial utilizando la x como parámetro. Por ejemplo se puede tomar la parametrización<br />

r(t) dada por<br />

x = t, y = 2t, z = t 3/2<br />

con t ≥ 0 pues según el enunciado sólo se considera la curva en el primer octante (de hecho, en el<br />

resto de R3 la curva no está ni siquiera definida).<br />

Ahora, puesto que (0, 0, 0) corresponde a t = 0 y (1, 2, 1) a t = 1 se tiene que la longitud pedida<br />

es, por definición,<br />

L =<br />

1<br />

0<br />

r ′ (t) dt =<br />

1<br />

0<br />

<br />

1 2 + 2 2 +<br />

<br />

3<br />

2 t1/2<br />

2 dt =<br />

1<br />

0<br />

<br />

5 + 9 4<br />

tdt =<br />

4 9<br />

2<br />

3<br />

<br />

5 + 9<br />

4 t<br />

3/2 | t=1<br />

t=0= 29√<br />

40√<br />

29− 5<br />

27 27<br />

2) Puesto que es fácil parametrizar la curva Γ y el campo F tiene una expresión sencilla, la<br />

circulación de F entre dos puntos cualesquiera se puede calcular directamente mediante la definición,<br />

es decir, mediante el cálculo de una integral curvilínea (si eso no fuese así, se podría comprobar si<br />

F es conservativo y en tal caso la circulación se podría calcular a través de un potencial escalar de<br />

F). Consideramos así la curva entre el punto (0, 0, 0) y un punto genérico correspondiente a un valor<br />

u ≥ 0 del parámetro (es decir, el punto (u, 2u, u3/2 )). Entonces, la circulación como función de u es<br />

u<br />

u<br />

u<br />

C(u) =<br />

0<br />

F(r(t)) · r ′ (t)dt =<br />

0<br />

t(4t 2 ), t 2 (2t), −t 3/2 t 2 · (1, 2, 3<br />

2 t1/2 )dt =<br />

0<br />

(8t 3 − 3<br />

2 t4 )dt


(nótese que una expresión del tipo t<br />

0 F(r(t)) · r′ (t)dt sería incorrecta, pues t denotaría a la vez a<br />

la variable de integración y al límite de integración).<br />

Obsérvese que puesto que lo que se quiere estudiar es la existencia de máximos de C(u), se puede<br />

trabajar estudiando las derivadas de C(u), no hace falta calcular la integral anterior. En efecto, por el<br />

teorema fundamental del cálculo, al ser la función subintegral continua, C(u) es derivable y además<br />

C ′ (u) = 8u 3 − 3<br />

2 u4<br />

Los ceros de C ′ (u) son u = 0 y u = 16.<br />

Para saber si son extremos podemos acudir a la segunda<br />

3<br />

derivada, C ′′ (u) = 24u2 − 6u3 , comprobando que C ′′ ( 16<br />

16<br />

) < 0, lo que indica que en u = hay un<br />

3 3<br />

máximo local. En cuanto a u = 0, se tiene C ′′ (0) = 0 (con lo que se debe acudir a derivadas de<br />

orden superior), C ′′′ (0) = 0 y C (4 (0) > 0 con lo que en u = 0 hay un mínimo local. Para estudiar<br />

la existencia de máximo global, basta darse cuenta que el límite de C(u) cuando u tiende a ∞ vale<br />

−∞ y que C( 16<br />

16<br />

) > C(0) para poder afirmar que el máximo global se da en u = , que corresponde<br />

3 3<br />

al punto ( 16<br />

√<br />

32 64 , , 3).<br />

3 3 9<br />

3) Razonando de la misma manera que en el apartado 1, para la curva dada se puede tomar la<br />

parametrización w(t) dada por<br />

x = t, y = 2t, z = α(t)<br />

donde t pertenece a un conjunto todavía sin determinar.<br />

La circulación de F entre el punto (0, 0, 0) y un punto genérico al que asignamos un valor de<br />

parámetro u es<br />

u<br />

H(u) = F(w(t)) · w ′ u 2 2 2<br />

(t)dt = t(4t ), t (2t), −α(t)t · (1, 2, α ′ u<br />

(t))dt = (8t 3 − α(t)α ′ (t)t 2 )dt<br />

0<br />

0<br />

Sabemos que H(u) debe ser nula para todo valor de u. La condición necesaria y suficiente es<br />

que la función subintegral sea identicamente nula, es decir,<br />

8t 3 − α(t)α ′ (t)t 2 = 0 para todo t (A)<br />

Para razonar esto rigurosamente, se puede proceder de la siguiente forma. Si se cumple (A) entonces<br />

claramente H(u) es nula para todo valor de u. Recíprocamente, si H(u) = 0 para todo u, derivando y<br />

aplicando el teorema fundamental del cálculo se obtiene (A), como se quería demostrar. Otra forma<br />

de razonar lo anterior es que la única posibilidad de que H(u) sea nula para todo valor de u es que<br />

F sea ortogonal a la curva en todos los puntos, es decir, F(w(t)) · w ′ (t) = 0 para todo t, con lo que<br />

se obtiene (A).<br />

Se ha obtenido una ecuación diferencial de variables separadas para α. Se tiene así, dividiendo<br />

por t 2<br />

α(t)α ′ (t) = 8t<br />

e integrando<br />

α2 (t)<br />

2 = 4t2 + C ⇔ α 2 (t) = 8t 2 + K ⇔ α(t) = ± √ 8t2 + K<br />

donde K es una constante. Como α(0) debe ser cero, se tiene que K = 0 y resulta así que hay dos<br />

soluciones dadas por<br />

α(t) = ±2 √ 2t<br />

BAREMO DE CALIFICACIÓN (sobre 10 puntos)<br />

Apdo. 1 −→ 2 puntos<br />

Apdo. 2. −→ 4 puntos<br />

Apdo. 3. −→ 4 puntos<br />

0


AMPLIACIÓN DE CÁLCULO (Curso 2003/2004) Convocatoria de septiembre 16.09.04<br />

PROBLEMA 1 (4 puntos) Este ejercicio consta de tres preguntas independientes. La dos primeras<br />

valen 1 punto y la tercera 2 puntos.<br />

1) Decidir razonadamente para qué valores de los parámetros reales p y q converge la integral<br />

impropia: ∞<br />

1<br />

t p (log t) q dt.<br />

Respuesta a 1): Se trata de una integral impropia debido a que el integrando puede no estar<br />

acotado en el extremo inferior del intervalo y también debido a que el intervalo de integración no<br />

está acotado superiormente.<br />

Para el estudio de su convergencia es conveniente realizar el cambio de variable: x = log t, del<br />

que se obtiene: ∞<br />

∞<br />

1<br />

t p (log t) q dt =<br />

0<br />

x q e (p+1)x dx.<br />

Ambas integrales se comportan de la misma manera en lo que a su convergencia se refiere. Consideramos<br />

pues la segunda y distinguimos los siguientes casos:<br />

A) p+1 > 0. Puesto que la integral es impropia en los dos extremos del intervalo, será convergente<br />

si y sólo si lo son simultáneamente las dos integrales:<br />

a<br />

I1 = x q e (p+1)x ∞<br />

dx , I2 = x q e (p+1)x dx,<br />

0<br />

donde a > 0 es cualquier número real (finito). Ahora bien, si p > −1, I2 es divergente para todo<br />

valor de q ∈ R y, por tanto, la integral de partida es siempre divergente en este caso (lo que se puede<br />

concluir sin necesidad de analizar el comportamiento de I1).<br />

La demostración de que I2 es divergente se puede hacer de forma sencilla utilizando el siguiente<br />

• Criterio de Comparación: Sea f integrable en [a, x] para todo x > a. Entonces si lím x→+∞ x n f(x)<br />

es finito y no nulo o infinito, para algún n ≤ 1, la integral<br />

a<br />

+∞<br />

a<br />

f(x) dx es divergente.<br />

Bata entonces tener en cuenta que, para p + 1 > 0 y para todo q ∈ R, se tiene<br />

lím<br />

x→∞ xq+1/2 e (p+1)x = ∞.<br />

B) p + 1 = 0. En este caso, la integral de partida será convergente si lo son simultáneamente:<br />

a<br />

J1 = x q ∞<br />

dx , J2 = x q dx,<br />

0<br />

donde a > 0 es cualquier número real (finito). Sin más que aplicar la definición de integral impropia<br />

se comprueba que J1 y J2 no convergen simultáneamente para ningún valor de q ∈ R\{0} y que,<br />

para q = 0, la integral de partida es, por definición, divergente.<br />

C) p + 1 < 0. En esta situación podemos utilizar la función Gamma de Euler:<br />

∞<br />

Γ(α) = x (α−1) e −x dx<br />

0<br />

a


(que converge si y solo si α > 0), dado que el cambio de variable (p + 1)x = y (con p + 1 < 0)<br />

transforma la integral de partida en<br />

1<br />

|p + 1| q+1<br />

∞<br />

y q e −y 1<br />

dy = Γ(q + 1)<br />

|p + 1| q+1<br />

0<br />

Por tanto, para p < −1, la integral propuesta es convergente si y solo si q > −1 y diverge si q ≤ −1.<br />

En resumen:<br />

∞<br />

La integral<br />

1<br />

t p (log t) q dt es convergente si y sólo si p < −1 y q > −1 y es divergente en otro<br />

caso.<br />

1 punto.<br />

2) Calcular razonadamente todos los valores posibles que puede tomar el flujo del campo vectorial<br />

F(x, y, z) := 2 z 2 x i + z 2 y j − z 3 k<br />

a través de una superficie regular cuyo borde orientado es la curva que resulta de la intersección de<br />

la esfera x 2 + y 2 + z 2 = 1 y el cilindro x 2 + z 2 = 1/4, y ≥ 0.<br />

Respuesta a 2): En primer lugar y dado que se trata de calcular un flujo es conveniente comprobar<br />

si el campo vectorial es solenoidal o no (es decir si su divergencia es o no nula). Con la notación<br />

F(x, y, z) := (F1(x, y, z), F2(x, y, z), F3(x, y, z)), se tiene:<br />

∇ · F = ∂F1<br />

∂x<br />

+ ∂F2<br />

∂y<br />

+ ∂F3<br />

∂z = 2z2 + z 2 − 3z 2 = 0<br />

y, por tanto, el campo es, en efecto, solenoidal en R 3 . Como R 3 es un dominio estrellado y F es de<br />

clase C 1 en R 3 , podemos aplicar el Teorema de Gauss que, en particular, nos permite concluir que el<br />

flujo de F a través de cualquier superficie cerrada es cero. De aquí se puede deducir que el flujo de F<br />

a través de cualquier superficie con el mismo borde orientado es el mismo, lo que se puede razonar<br />

como sigue:<br />

Dos superficies regulares cualesquiera que tengan el mismo borde orientado siempre forman una<br />

superficie cerrada. Sean entonces Σ + 1 y Σ + 2 dos superficies con un borde orientado común (que induce<br />

un sentido para el vector normal a ambas) y Σ − 1 y Σ − 2 las mismas superficies pero con la otra<br />

orientación posible. Sea ahora Σ + = Σ + −<br />

1 Σ2 la superficie cerrada correspondiente (orientada según<br />

el vector normal saliente o entrante). Entonces, el flujo de F a través de Σ + es nulo y, por tanto:<br />

<br />

<br />

<br />

<br />

<br />

<br />

F dσ = F dσ + F dσ = 0 =⇒ F dσ = − F dσ = F .<br />

Σ +<br />

Σ +<br />

1<br />

Σ −<br />

2<br />

ALTERNATIVA: Una vez comprobado que el campo es solenoidal, una alternativa más sencilla<br />

al razonamiento (basado en el teorema de Gauss) que se acaba de dar consiste en utilizar el teorema<br />

de Stokes de la siguiente forma.<br />

Puesto que F es solenoidal en R3 , está garantizada la existencia de otro campo vectorial G, que es<br />

un potencial vector de F en R3 , de forma que se cumple F = ∇ × G en R3 . Entonces, para cualquier<br />

superficie Σ con un borde orientado Γ se puede aplicar el teorema de Stokes y, por tanto, se tiene:<br />

<br />

<br />

F dσ ≡ ∇ × G dσ = G ds<br />

Σ<br />

Σ<br />

Σ +<br />

1<br />

Γ<br />

Σ −<br />

2<br />

Σ +<br />

2


de donde se concluye que el flujo de F solo depende del borde orientado considerado y no de la<br />

superficie que se considere (con ese borde).<br />

Para calcular el flujo pedido, basta entonces considerar una superficie cualquiera con el borde<br />

descrito en el enunciado que es la circunferencia x2 + z2 = 1/4 situada en el plano y = √ 3/2.<br />

Por sencillez, elegimos el círculo C := {(x, y, z) ∈ R3 : x2 + z2 ≤ 1/4 , y = √ 3/2}, que se puede<br />

parametrizar mediante γ = (x(r, θ), y(r, θ), z(r, θ)) con:<br />

√<br />

3<br />

x(r, θ) = r cos θ , y(r, θ) = , z(r, θ) = r sin θ (r ∈ (0, 1/2) , θ ∈ (−π, π)) ,<br />

2<br />

con lo que las dos posibilidades para el vector normal a C son: (0, ±r, 0), que van a dar lugar a dos<br />

posibles valores que puede tomar el flujo pedido (siempre que sea distinto de cero).<br />

Calculamos pues los dos valores posibles del flujo de F a través de C que se corresponden con las<br />

dos orientaciones posibles ((0, r, 0) (C + ) y (0, −r, 0) (C− )) de su vector normal.<br />

<br />

C +<br />

F dσ =<br />

=<br />

1/2<br />

0<br />

√ 3<br />

2<br />

π<br />

−π<br />

1/2<br />

0<br />

2 2 3<br />

2 z(r, θ) x(r, θ) , z(r, θ) y , − z(r, θ) · ⎝<br />

π<br />

−π<br />

r 3 sin 2 θ dr dθ =<br />

y, por tanto, el otro valor que puede tomar el flujo es −<br />

√ 3 π<br />

128 ,<br />

√ 3 π<br />

128 .<br />

⎛<br />

0<br />

r<br />

0<br />

⎞<br />

⎠ dr dθ<br />

ALTERNATIVA: Para calcular los valores posibles del flujo se puede utilizar el teorema de<br />

Stokes en la forma antes mencionada, lo que exige la obtención de un potencial vector de F, que<br />

siempre se puede elegir de la forma G(x, y, z) = (G1(x, y, z), G2(x, y, z), 0). Para calcularlo se identifican<br />

componentes en la igualdad F = ∇×G, de lo que resulta el sistema de ecuaciones en derivadas<br />

parciales siguiente:<br />

− ∂G2<br />

∂z = 2z2 x ,<br />

∂G1<br />

∂z = z2 y<br />

∂G2<br />

∂x<br />

− ∂G1<br />

∂y = −z3 ,<br />

de cuya resolución se obtiene: G1(x, y, z) = − 2<br />

3z3y + M(x, y), G2(x, y, z) = 1<br />

3z3x + N(x, y), donde<br />

M(x, y) y N(x, y) son funciones arbitrarias que deben satisfacer la condición ∂M ∂N<br />

− = 0, que<br />

∂x ∂y<br />

se cumple si se elige M = N = 0.<br />

Así pues, un potencial vector de F es G = − 2<br />

3z3y , 1<br />

3z3x , 0 y el flujo pedido vendrá dado por:<br />

<br />

π <br />

F dσ = G ds = −<br />

Σ<br />

Γ<br />

−π<br />

2<br />

3 z(θ)3y(θ) , 1<br />

3 z(θ)3 ⎛<br />

⎞<br />

−(1/2) sin θ<br />

x(θ) , 0 · ⎝ 0 ⎠ dθ<br />

(1/2) cos θ<br />

√ π<br />

3<br />

= sin<br />

48<br />

4 √<br />

3 π<br />

θ dθ =<br />

128 ,<br />

−π<br />

donde Γ es la circunferencia x 2 +z 2 = 1/4 situada en el plano y = √ 3/2 para la que se ha considerado<br />

la parametrizacíon x(θ) = (1/2) cos(θ), y(θ) = √ 3/2, z(θ) = (1/2) sin(θ) (θ ∈ (−π, π)).<br />

El otro valor posible, − √ 3 π/128, se obtiene recorriendo la circunferencia en sentido contrario al<br />

considerado en el cálculo anterior.<br />

1 punto.


AMPLIACIÓN DE CÁLCULO (Curso 2003/2004) Convocatoria de septiembre 16.09.04<br />

PROBLEMA 2 (3 puntos)<br />

Sean f : R → R una función de clase C 1 , y V el campo vectorial definido por<br />

V(x, y, z) = yzi + xzj + y f(x) k.<br />

1) Determinar las funciones f para las cuales el campo V es solenoidal. En tal caso, hallar todos los<br />

potenciales vectores de V que sean de la forma L(x, y, z)i + M(y, z)j.<br />

2) Sea Γ la curva intersección de x 2 + y 2 + z 2 = 1 (x, y, z ≥ 0) con los tres planos coordenados,<br />

y sea Σ el cono formado por los segmentos de recta que parten del punto (1, 1, 1) y terminan en los<br />

puntos de Γ.<br />

Si f(x) = x + a, calcular la constante a para que el flujo de V a través de Σ sea nulo.<br />

Respuesta: Se entregará esta hoja y, a lo sumo, una adicional.<br />

1) Un campo regular es solenoidal si y sólo si su divergencia es nula. Dado que<br />

div (V) = ∂(yz)<br />

∂x<br />

+ ∂(xz)<br />

∂y<br />

+ ∂(yf (x))<br />

∂z<br />

se tiene que V es solenoidal para toda función f ∈ C 1 (R).<br />

Como V es solenoidal en el dominio estrellado R 3 , admite un potencial vector F, es decir,<br />

V = rot(F); si éste es de la forma F(x, y, z) = L(x, y, z)i + M(y, z)j debe cumplirse que<br />

<br />

<br />

<br />

i j k<br />

(yz, xz, yf (x)) = V = ∂<br />

∂<br />

∂x<br />

∂y<br />

L(x, y, z) M(y, z) 0<br />

∂<br />

∂z<br />

<br />

<br />

<br />

<br />

<br />

=<br />

<br />

−<br />

= 0,<br />

∂M(y, z)<br />

,<br />

∂z<br />

∂L(x, y, z)<br />

, −<br />

∂z<br />

<br />

∂L(x, y, z)<br />

.<br />

∂y<br />

Igualando las primeras componentes e integrando respecto a z se tiene que M(y, z) = −yz 2 /2 +<br />

m(y). Ídem para las segundas componentes, de donde se obtiene que L(x, y, z) = xz2 /2 + l(x, y);<br />

introduciendo esta función en la tercera componente del rotacional, se deduce que<br />

∂l(x, y)<br />

yf (x) = −<br />

∂y<br />

⇔ l(x, y) = −y2 f (x) + h(x).<br />

2<br />

Finalmente, todos los potenciales vectores pedidos son de la forma:<br />

2 xz<br />

F(x, y, z)=<br />

2 − y2 <br />

f (x)<br />

+ h(x) i + −<br />

2<br />

yz2<br />

<br />

+ m(y) j<br />

2<br />

donde h, m son funciones cualesquiera de C 1 (R) .<br />

2) Como V = rot(F), F ∈ C1 (R3 ) y Σ es unión de superficies regulares, podemos aplicar el Teorema<br />

de Stokes: <br />

V = rot(F) = F<br />

Σ Σ<br />

Γ<br />

puesto que la curva Γ es el borde de la superficie Σ. Como queremos que dicha integral sea nula, en<br />

este caso es indiferente la orientación de la curva Γ, pues la orientación contraria cambiaría de signo<br />

la integral, que seguiría valiendo cero.


Calculemos, por tanto, dicha integral curvilínea, parametrizando Γ. Esta curva es unión de tres<br />

arcos, obtenidos por la intersección del triángulo esférico x 2 + y 2 + z 2 = 1 (x, y, z ≥ 0) con cada uno<br />

de los tres planos coordenados. Por tanto, consta de:<br />

⎧<br />

⎨<br />

⎩<br />

Γ1 (t) = (cos(t), sen (t), 0) t ∈ [0, π/2] en el plano z = 0,<br />

Γ2 (t) = (0, cos(t), sen (t)) t ∈ [0, π/2] en el plano x = 0,<br />

Γ3 (t) = (sen (t) , 0, cos(t)) t ∈ [0, π/2] en el plano y = 0.<br />

Por otro lado, escogemos la forma más sencilla del potencial vector F ya calculado tomando, por<br />

ejemplo, las funciones h y m nulas. Además, obsérvese que, para f (x) = x + a, se tiene que<br />

La integral curvilínea <br />

<br />

<br />

<br />

Γ1<br />

Γ2<br />

Γ3<br />

Γ<br />

F(x, y, z)= 1 2 2 1 2<br />

xz − y (x + a) i − yz<br />

2<br />

2<br />

j<br />

F es la suma de las tres integrales siguientes:<br />

F = 1<br />

π/2 2<br />

−sen (t) (cos (t) + a), 0, 0 · (−sen (t) , cos (t) , 0) dt =<br />

2 0<br />

= 1<br />

π/2<br />

sen<br />

2 0<br />

3 (t) (cos (t) + a)dt = 1 a<br />

+<br />

8 3<br />

F = 1<br />

π/2<br />

<br />

π/2<br />

2 2 1<br />

−a cos (t) , − cos(t)sen (t) , 0 · (0, −sen (t) , cos (t)) dt = sen<br />

2 0<br />

2 0<br />

3 (t) cos (t) dt = 1<br />

8<br />

F = 1<br />

π/2<br />

<br />

π/2<br />

2 1<br />

sen (t) cos (t) , 0, 0 · (cos (t) , 0, −sen (t)) dt = sen (t) cos<br />

2<br />

2<br />

3 (t) dt. = 1<br />

8 .<br />

0<br />

La suma de las tres integrales debe ser cero, de donde<br />

0 = 3 a<br />

+<br />

8 3<br />

⇔ a = −9<br />

8 .<br />

OTRA FORMA: Si se cierra la superficie Σ mediante otra superficie S, de forma que Ω sea el<br />

volumen contenido entre ellas, como V ∈ C1 (R3 ) podemos aplicar el Teorema de la divergencia<br />

de Gauss: <br />

VdS + VdS = div (V) dV = 0<br />

Σ<br />

S<br />

Ω<br />

donde se ha utilizado que V siempre es solenoidal. Se pide que la integral sobre Σ sea nula, así que<br />

es suficiente que <br />

VdS = 0 para alguna superficie regular S que también tenga por borde a la<br />

S<br />

curva Γ. Puede elegirse S, por ejemplo, como la unión de las porciones de los tres planos coordenados<br />

limitados por la curva Γ en el primer octante; o bien, como el triángulo esférico del primer octante.<br />

Hagamos los cálculos, por ejemplo, para el triángulo esférico parametrizado como S = {(x, y, z)/<br />

z = 1 − x2 − y2 , (x, y) ∈ D} siendo D = {(x, y)/ x2 + y2 ≤ 1, x, y ≥ 0}. La normal dada por la<br />

parametrización en cada punto es (x, y, z(x, y))/|z(x, y)|, luego:<br />

<br />

0 =<br />

<br />

VdS = (yz, xz, y(x + a)) · (x, y, z) 1<br />

<br />

dxdy =<br />

z<br />

(3xy + ay)dxdy.<br />

S<br />

D<br />

Calculando dichas integrales dobles, se llega al mismo resultado a = − 9<br />

8 .<br />

0<br />

D


AMPLIACIÓN DE CÁLCULO (Curso 2004/2005) Convocatoria de febrero 15.02.05<br />

PROBLEMA 1 (4 puntos) (Este problema consta de dos apartados independientes).<br />

1) Se considera la función compleja de variable compleja definida por<br />

f(z) =<br />

sen πz<br />

p(z)<br />

+ cos πz<br />

p ′ (z) ,<br />

donde p(z) = z 3 (z − α) (α ∈ C) y se supone que sen πα = 0 y cos πα = 0. Se pide:<br />

1.1) Determinar la parte principal de los desarrollos de f en serie de Laurent alrededor de z = 0<br />

y z = α convergentes en discos perforados con centro en z = 0 y z = α, respectivamente.<br />

<br />

1.2) Calcular f(z)dz, siendo Γ la frontera de la intersección de los dos dominios del plano<br />

Γ<br />

complejo definidos por |z| + |z − α| ≤ 2|α| y 2|z| ≤ |α|, recorrida en sentido positivo.<br />

(2 puntos)<br />

2) Sea S la superficie de revolución engendrada al girar la curva de ecuación z = √ y con 1 ≤ y ≤ 2<br />

(situada en plano Y Z) alrededor del eje OZ.<br />

Calcular ángulo sólido de visión desde el origen de la porción de S situada en el primer octante.<br />

(2 puntos)<br />

Respuesta:<br />

1.1) Para determinar la parte principal de la serie de Laurent de la función f alrededor de los dos<br />

puntos considerados se procede, en primer lugar, a identificar el tipo de singularidad que presenta<br />

f en cada uno de ellos. Esta información permite reconocer la forma de la serie de Laurent y, en<br />

particular, permite saber qué coeficientes hay que calcular en cada caso.<br />

(a) El punto z = 0 es un polo doble ya que se cumple:<br />

lím z<br />

z→0 2 1 + 3π<br />

f(z) = −<br />

3α ,<br />

donde el límite se calcula aplicando una vez la regla de L’Hopital. Por lo tanto, la serie de<br />

Laurent de f convergente en un disco perforado de centro el punto z = 0 tiene la forma:<br />

f(z) = a−2 a−1<br />

+<br />

z2 z +<br />

∞<br />

n=0<br />

an z n ,<br />

con lo que los coeficientes que hay calcular son a−2 y a−1. Para ello se tiene:<br />

z 2 f(z) = a−2 + a−1z +<br />

∞<br />

n=0<br />

an z n+2 =⇒ a−2 = lím z<br />

z→0 2 1 + 3π<br />

f(z) = −<br />

3α ,<br />

donde el límite es el mismo que se ha calculado para identificar la singularidad que presenta f<br />

en el punto z = 0.<br />

Falta únicamente determinar el coeficiente a−1 que es, por definición, el residuo de f en el polo<br />

doble z = 0; por tanto:<br />

a−1 =<br />

d<br />

Res[f, z = 0] = lím<br />

z→0 dz (z2 <br />

d sen(πz) d cos(πz)<br />

f(z)) = lím<br />

+ lím<br />

z→0 dz z(z − α) z→0 dz (4z − 3α)<br />

= − π<br />

α2 4<br />

−<br />

9α2 4 + 9π<br />

= −<br />

9α2 ,


donde el primer límite se calcula aplicando dos veces la regla de L’Hopital y el segundo es<br />

inmediato.<br />

Así pues, la parte principal de la serie de Laurent de f convergente en un disco perforado de<br />

centro el punto z = 0 es:<br />

1 + 3π 4 + 9π<br />

− −<br />

3α z2 9α2 z .<br />

(b) El punto z = α es un polo simple ya que se cumple:<br />

lím (z − α) f(z) =<br />

z→α sen(πα)<br />

α3 = 0 (por hipótesis) ,<br />

donde el límite es inmediato. Por lo tanto, en este caso, la serie de Laurent de f que converge<br />

en un disco perforado de centro el punto z = α tiene la forma:<br />

f(z) = b−1<br />

z − α +<br />

∞<br />

bn (z − α) n .<br />

Hay que determinar el coeficiente b−1 que es, por definición, el residuo de f en el polo simple<br />

z = α y, por tanto, coincide con el límite que se acaba de calcular; es decir:<br />

n=0<br />

b−1 = Res[f, z = α] = lím (z − α) f(z) =<br />

z→α sen(πα)<br />

α3 .<br />

Así pues, la parte principal de la serie de Laurent de f convergente en un disco perforado de<br />

centro el punto z = α es:<br />

sen(πα)<br />

α 3 (z − α) .<br />

1.2) Se trata de calcular la integral de f a lo largo de una curva cerrada ya que la intersección<br />

de los dos dominios dados (el limitado por la elipse de focos z = 0 y z = α y el limitado por la<br />

circunferencia de centro z = 0 y radio |α|/2) siempre tienen una intersección no vacía. Se puede, por<br />

tanto, aplicar el teorema de los residuos de Cauchy, según el cual:<br />

<br />

f(z)dz = 2π i <br />

n(Γ, zk) Res[f, z = zk] ,<br />

Γ<br />

k<br />

donde la suma se extiende a los puntos singulares zk de f contenidos en el dominio limitado por<br />

la curva Γ y n(Γ, zk) es el índice de Γ con respecto a zk, del que se sabe que es un número entero<br />

positivo, pues la curva se recorre en sentido positivo.<br />

En estas condiciones, es necesario determinar todos los puntos singulares de f que son los dos<br />

considerados anteriormente, es decir, el polo doble z = 0 y el polo simple z = α y, además, z = 3α/4<br />

que es un cero simple de p ′ (z) y puede ser una singularidad evitable de f (si cos(3απ/4) = 0) o un<br />

polo simple de f (si cos(3απ/4) = 0), ya que se cumple:<br />

lím<br />

z→3α/4<br />

(z − 3α<br />

4<br />

4 cos(3πα/4)<br />

) f(z) =<br />

9α2 .<br />

Falta por decidir qué puntos singulares pertenecen al dominio acotado limitado por Γ que serán<br />

aquellos que cumplan simultáneamente las dos desigualdades que definen los dos dominios dados. Es<br />

sencillo comprobar que el único punto que las verifica es z = 0, pues z = α y z = 3α/4 no pertenecen<br />

al disco de centro z = 0 y radio |α|/2. Por tanto:<br />

<br />

Γ<br />

f(z)dz = 2π i n(Γ, z = 0) Res[f, z = 0] , = − 2πi<br />

4 + 9π<br />

n(Γ, z = 0) .<br />

9α2


2) Dada una superficie Σ en R3 y un punto p ∈ R3 se denomina ángulo sólido de visión de Σ desde p<br />

al área que resulta de la intersección de la esfera de centro p y radio uno y el cono sólido de vértice p<br />

formado por todas las rectas que, trazadas desde p, intersecan la superficie Σ en al menos un punto.<br />

En el problema propuesto el punto p es el origen y la superficie Σ es la que se obtiene por<br />

revolución de la curva z = √ y (1 ≤ y ≤ 2) alrededor del eje OZ. La ecuación cartesiana de esta<br />

superficie es z = x2 + y2 . Puesto que solamente hay que considerar la porción situada en el<br />

primer octante, una parametrización de Σ viene dada por:<br />

R(θ, u) = (x(θ, u), y(θ, u), z(θ, u)) con:<br />

⎧<br />

⎨<br />

⎩<br />

x(θ, u) = u sen θ<br />

y(θ, u) = u cos θ<br />

z = √ u<br />

⎫<br />

⎬<br />

⎭<br />

π<br />

(0 ≤ θ ≤ , 1 ≤ u ≤ 2) .<br />

2<br />

Por otra parte, puesto que se trata de una superficie de revolución, el cono sólido a considerar<br />

en este caso se obtiene también por revolución alrededor del eje OZ del haz de rectas (situadas en<br />

el plano Y Z) que, trazadas desde el origen, intersecan la curva z = √ y (1 ≤ y ≤ 2) al menos una<br />

vez. Este haz está constituido por todas las rectas cuya pendiente está comprendida entre √ 2/2 y 1<br />

(z = ay con √ 2/2 ≤ a ≤ 1) y cada una de ellas tiene un único punto en común con la curva z = √ y<br />

(como se puede comprobar resolviendo la ecuación ay = √ y con √ 2/2 ≤ a ≤ 1). Esto último significa<br />

que desde el origen “se ve toda una cara de la superficie considerada” , lo que permite calcular el<br />

ángulo sólido de Σ (o de la cara de Σ “que se ve desde el origen”) como el flujo del campo r/r3 sobre Σ (en concreto, sobre la cara de Σ “que no se ve desde el origen”). Debe tenerse en cuenta<br />

que para que el ángulo sólido se pueda calcular mediante este procedimiento el flujo debe calcularse<br />

sobre la porción de la superficie “que se ve desde el origen” (en este caso la totalidad de la superficie).<br />

Denotando por Σ + esta cara, la normal a considerar es:<br />

∂R(θ, u)<br />

∂u<br />

× ∂R(θ, u)<br />

∂θ<br />

=<br />

√<br />

u √ <br />

sen θ, cos θ, u .<br />

2<br />

Teniendo en cuenta que sobre la superficie considerada el campo r/r 3 tiene la expresión<br />

R(θ, u)<br />

=<br />

R(θ, u)3 el ángulo sólido ΩΣ que se desea calcular es:<br />

ΩΣ =<br />

<br />

= 1<br />

2<br />

Σ +<br />

π/2<br />

0<br />

r<br />

ds =<br />

r3 2<br />

dθ<br />

1<br />

π/2<br />

0<br />

1<br />

(u 2 + u) 3/2<br />

2<br />

dθ<br />

1<br />

1 1<br />

=<br />

(1 + u) 3/2 2<br />

√ u<br />

2(u 2 + u) 3/2<br />

π/2<br />

0<br />

u sen θ, u cos θ, √ u ,<br />

⎛ ⎞<br />

√ <br />

u sen θ<br />

u sen θ, u cos θ, u ⎝ u<br />

√<br />

cos θ ⎠ du<br />

u<br />

<br />

− 2<br />

2 <br />

√ dθ =<br />

1 + u 1<br />

π<br />

2 √ √ √ <br />

3 − 2 .<br />

6<br />

2 (Procedimiento alternativo). En lugar de utilizar el flujo del campo r/r 3 , se trata de identificar<br />

directamente la intersección entre el cono sólido antes descrito y la esfera de centro el origen y<br />

radio uno. Una vez identificado este casquete esférico, E, el ángulo sólido se obtendrá calculando su<br />

área. Pero E se obtiene de la intersección de la esfera con el cono sólido que resulta de girar alrededor<br />

del eje OZ el haz de rectas z = ay con √ 2/2 ≤ a ≤ 1 antes mencionado y que está situado en el<br />

primer octante. Una parametrización del mismo es:<br />

E(θ, φ) = (sen φ cos θ , sen φ sen θ , cos φ) ,<br />

π<br />

4<br />

√<br />

π 2<br />

π<br />

≤ φ ≤ − arc tg , 0 ≤ θ ≤<br />

2 2 2 ,<br />

donde θ varía entre 0 y π/2 por estar en el primer octante y los extremos del intervalo de variación<br />

del azimut φ se obtienen restando de π/2 los ángulos cuyas tangentes son las pendientes de las rectas


z = y (que es π/4) y z = ( √ 2/2) y (que es arc tg √ 2/2), que son las que cortan a la curva z = √ y en<br />

sus dos extremos.<br />

Teniendo en cuenta que <br />

∂E(θ, φ)<br />

∂θ<br />

el ángulo sólido ΩΣ viene dado por<br />

ΩΣ =<br />

=<br />

<br />

π<br />

E<br />

2 √ 6<br />

ds =<br />

π/2<br />

dθ<br />

0<br />

√ √ <br />

3 − 2 ,<br />

× ∂E(θ, φ)<br />

π/2−arc tg √ 2/2<br />

π/4<br />

∂φ<br />

sen φ dφ = π<br />

2<br />

<br />

<br />

<br />

= sen φ ,<br />

<br />

cos π<br />

4<br />

− cos<br />

<br />

π<br />

− arc tg<br />

2<br />

donde se han utilizado las relaciones cos(π/2 − a) = sen a y sen arc tg a = a/ √ 1 + a 2 .<br />

√ <br />

2<br />

2


AMPLIACIÓN DE CÁLCULO (Curso 2004/2005) Convocatoria de febrero 15.02.05<br />

PROBLEMA 2 (3 puntos)<br />

1) Calcular el campo escalar u : R 3 −→ R de clase C 1 para que el campo vectorial F definido por:<br />

sea conservativo en R 3 .<br />

F (x, y, z) = (xy 2 z 2 − 2xy, u(x, y, z), x 2 y 2 z + 1) y F (0, y, z) = (0, y, 1) ,<br />

2) En tal caso, calcular la circulación de F sobre la curva Γ definida por las ecuaciones<br />

x − z = 0<br />

(xz + yz + y 2 − 3) − √ 1 − x 2 = 0<br />

desde el punto A := (0, 2, 0) hasta el punto B := (1, 1, 1).<br />

3) Calcular la circulación del campo vectorial G definido mediante<br />

G(x, y, z) = (y 2 x 2 z − 2yz, x 3 yz − xz + y, x 3 y 2 + 1)<br />

sobre el mismo arco de la curva Γ considerado en el apartado anterior.<br />

Respuesta:<br />

1) Si F es irrotacional, como el dominio R3 es estrellado, F será conservativo. Imponemos, pues:<br />

0 = −→<br />

<br />

<br />

i j k<br />

<br />

rotF = <br />

∂ ∂ ∂<br />

<br />

∂x ∂y ∂z<br />

xy2z 2 − 2xy u(x, y, z) x2y2 <br />

<br />

<br />

<br />

<br />

<br />

<br />

z + 1 <br />

<br />

= 2x 2 yz − ∂u ∂u<br />

, 0 ,<br />

∂z ∂x − 2xyz2 <br />

+ 2x<br />

=⇒ ∂u<br />

∂x = 2xyz2 − 2x =⇒ u = x 2 yz 2 − x 2 + α(y, z)<br />

=⇒ 2x 2 yz = ∂u<br />

∂z = 2x2yz + ∂α<br />

∂z<br />

luego α(y, z) solo depende de y. Por otra parte,<br />

y se tiene<br />

(y, z) =⇒ ∂α<br />

∂z<br />

F (0, y, z) = (0, y, 1) =⇒ y = u(0, y, z) = α(y, z),<br />

u = x 2 yz 2 − x 2 + y.<br />

2) Llamamos L, M, N a las tres componentes de F . Fijado un punto del dominio, por ejemplo, el<br />

origen, un potencial escalar U, es decir, un campo que verifica F = −−→<br />

grad U, se puede obtener por la<br />

conocida fórmula<br />

x<br />

y<br />

z<br />

U(x, y, z) = L(r, 0, 0) dr + M(x, s, 0) ds + N(x, y, t) dt<br />

0<br />

y<br />

0<br />

0<br />

= (−x 2 z<br />

+ s) ds + (x 2 y 2 t + 1) dt<br />

=<br />

0<br />

<br />

−x 2 s + s2<br />

s=y 2<br />

s=0<br />

= −x 2 y + y2<br />

2 + x2y2z 2<br />

+ z.<br />

2<br />

0<br />

2 2 2 t=z<br />

x y t<br />

+ + t2<br />

2<br />

t=0<br />

= 0,


La circulación que se pide será<br />

<br />

F · dr = U(1, 1, 1) − U(0, 2, 0) = −1.<br />

Γ<br />

3) El campo G no es conservativo pero, en el plano x = z, coincide con F . Por tanto, como Γ<br />

está contenida en ese plano, se tiene:<br />

<br />

G · dr = F · dr = −1.<br />

Γ<br />

Γ


AMPLIACIÓN DE CÁLCULO (Curso 2004/05) Convocatoria de Junio 28-06-05<br />

Problema 2.– (3 puntos) Este problema consta de dos preguntas independientes.<br />

1. (1 punto) Se considera una superficie regular orientada S cuyo borde orientado es la curva<br />

Γ. Sea v un vector no nulo de R3 , y sea r = (x, y, z) un punto genérico de Γ. Determínese<br />

razonadamente el valor de α que hace cierta la igualdad:<br />

<br />

α v ds = v × r dr.<br />

S<br />

2. (2 puntos) Sea C una curva simple y acotada del plano R 2 . Sea f un campo escalar positivo y<br />

de clase 1 a trozos en un dominio que contiene a C. Sea S la superficie regular definida por<br />

Se pide:<br />

<br />

a) Demostrar la igualdad A(S) =<br />

S = {(x, y, z) ∈ R 3 , (x, y) ∈ C, 0 ≤ z ≤ f(x, y)}.<br />

C<br />

Γ<br />

f, donde A(S) representa el área de la superficie S.<br />

b) Calcular explícitamente el área A(S) cuando la curva C viene parametrizada por ϕ(t) =<br />

(t, sen t), t ∈ [0, 2π] y la función f(x, y) = |y cos x|.<br />

Solución:<br />

Se entregará esta hoja y, a lo sumo, una adicional.<br />

1. Apliquemos el Teorema de Stokes, que relaciona la integral de línea de un campo F sobre la<br />

curva Γ con el flujo de su rotacional (rot (F)) a través de la superficie S:<br />

<br />

<br />

rot (F) ds = F dr.<br />

S<br />

En este caso el campo vectorial es<br />

<br />

<br />

i<br />

F (r) = v × r = <br />

j<br />

k<br />

v1<br />

v2<br />

v3<br />

<br />

x <br />

<br />

y <br />

<br />

z = (v2z − v3y) i+ (v3x − v1z) j+ (v1y − v2x) k,<br />

y su rotacional es<br />

rot (F) =<br />

∂F3<br />

∂y<br />

Γ<br />

<br />

∂F2 ∂F1 ∂F3 ∂F2 ∂F1<br />

− i+ − j+ − k =2v<br />

∂z ∂z ∂x ∂x ∂y<br />

con lo cual la expresión del enunciado es cierta cuando y sólo cuando α = 2.<br />

2. a) Si la curva C viene parametrizada como<br />

C = {(x (t) , y (t)), t ∈ [a, b]},


entonces la integral de línea del enunciado es:<br />

<br />

Γ<br />

b<br />

f = f (x (t) , y (t)) (x ′ (t) , y ′ (t)) dt.<br />

Por otro lado, una parametrización de la superficie es<br />

a<br />

S = {Φ (t, z) = (x (t) , y (t) , z) , t ∈ [a, b], 0 ≤ z ≤ f (x (t) , y (t))} ;<br />

calculemos su área:<br />

<br />

A (S) =<br />

S<br />

b <br />

f(x(t),y(t)) <br />

ds =<br />

<br />

∂Φ<br />

∂t<br />

a<br />

0<br />

× ∂Φ<br />

∂z<br />

<br />

<br />

<br />

dt dz.<br />

En este caso se tiene que<br />

<br />

<br />

<br />

∂Φ ∂Φ <br />

× <br />

∂t ∂z = (x′ (t) , y ′ (t) , 0) × (0, 0, 1) = (x ′ (t) , y ′ (t) , 0) = (x ′ (t) , y ′ (t)) ,<br />

donde se ha utilizado que el producto vectorial de dos vectores ortogonales tiene por<br />

norma el producto de las normas. Con todo ello, queda comprobado que el área coincide<br />

con aquella integral de línea:<br />

b f(x(t),y(t))<br />

A (S) =<br />

(x ′ (t) , y ′ b<br />

(t)) dt dz = f(x (t) , y (t)) (x ′ (t) , y ′ <br />

(t)) dt =<br />

a<br />

0<br />

b) Apliquemos el resultado anterior al caso particular en que la curva es<br />

C = {ϕ (t) = (t, sen t), t ∈ [0, 2π]}<br />

y f(x, y) = |y cos x| . Entonces f(x (t) , y (t)) = |sen t cos t| , y por otro lado (x ′ (t) , y ′ (t)) =<br />

(1, cos t) = √ 1 + cos 2 t, así que el área puede calcularse como<br />

A (S) =<br />

2π<br />

0<br />

π/2<br />

= 4<br />

0<br />

f(x (t) , y (t)) (x ′ (t) , y ′ (t)) dt =<br />

a<br />

2π<br />

sent cos t √ 1 + cos2 t dt = 4<br />

√<br />

− 1 + cos2 t<br />

3<br />

0<br />

|sen t cos t| √ 1 + cos 2 t dt =<br />

3 t=π/2<br />

t=0<br />

= 4<br />

<br />

2<br />

3<br />

√ <br />

2 − 1 .<br />

Γ<br />

f.


AMPLIACIÓN DE CÁLCULO (Curso 2004/05) Convocatoria de Junio 28-06-05<br />

PROBLEMA 3 (3 puntos)<br />

Se definen las funciones<br />

x<br />

f(x) = e −t2<br />

0<br />

2 1<br />

e<br />

dt ; g(x) =<br />

0<br />

−x2 (t2 +1)<br />

t2 + 1 dt.<br />

Obviamente, f es derivable en R. Es fácil ver que g también lo es, y que su derivada se obtiene<br />

derivando el integrando respecto de x. Estos hechos pueden darse por válidos, y no se pide su<br />

demostración en este ejercicio. Se pide:<br />

(a) Calcular f ′ (x) + g ′ (x), simplificando el resultado todo lo que se pueda.<br />

(b) Calcular f(x) + g(x), simplificando el resultado todo lo que se pueda.<br />

(c) Calcular con todo rigor lím x→+∞ g(x).<br />

(d) Utilizando (b) y (c), calcular el valor de<br />

Γ<br />

<br />

1<br />

.<br />

2<br />

Nota. Solo se puntuará el apartado (d) si se utilizan (b) y (c).<br />

Respuesta: Se entregará esta hoja y, a lo sumo, una adicional.<br />

(a) Se tiene<br />

f ′ (x) = 2e −x2<br />

x<br />

e −t2<br />

dt ;<br />

0<br />

1<br />

g ′ (t<br />

(x) = −2x<br />

0<br />

2 + 1)e−x2 (t2 +1)<br />

t2 dt = −2xe<br />

+ 1<br />

−x2<br />

0<br />

= −2xe −x2<br />

x<br />

x<br />

du −u2<br />

e = −2e−x2 e<br />

x −u2<br />

du<br />

donde se ha hecho el cambio de variable xt = u.<br />

Por tanto, f ′ (x) + g ′ (x) ≡ 0.<br />

luego<br />

0<br />

0<br />

1<br />

e −x2t2 dt<br />

(b) Integrando la identidad anterior, se tiene f(x) + g(x) ≡ K, pero f(0) = 0, y<br />

de donde<br />

g(0) =<br />

π<br />

4<br />

1<br />

0<br />

dt<br />

t 2 + 1<br />

π<br />

= [arc tg t]1 0 =<br />

4<br />

= f(0) + g(0) = K,<br />

f(x) + g(x) ≡ π<br />

4 .<br />

(c) Como x 2 ≤ x 2 (t 2 + 1), se tiene −x 2 (t 2 + 1) ≤ −x 2 , por tanto,<br />

0 ≤ g(x) =<br />

1<br />

0<br />

e −x2 (t 2 +1)<br />

t 2 + 1<br />

dt ≤ e−x2<br />

1<br />

0<br />

dt<br />

t 2 + 1<br />

= π<br />

4 e−x2<br />

,


de donde,<br />

luego<br />

(d) Para p > 0, se tiene<br />

Γ(p) =<br />

π<br />

0 ≤ lím g(x) ≤ lím<br />

x→+∞ x→+∞ 4 e−x2 = 0,<br />

lím g(x) = 0.<br />

x→+∞<br />

∞<br />

x<br />

0<br />

p−1 e −x ∞<br />

dx = 2 t<br />

0<br />

2p−1 e −t2<br />

donde se ha hecho el cambio x = t2 . Haciendo p = 1/2,<br />

∞<br />

1<br />

Γ = 2 e<br />

2<br />

−y2<br />

dy.<br />

pero<br />

luego<br />

Tomando límites en la identidad obtenida en (b) y, usando (c),<br />

<br />

π<br />

<br />

lím f(x) = lím − g(x) =<br />

x→+∞ x→+∞ 4 π<br />

4 ,<br />

π<br />

4<br />

0<br />

∞<br />

= lím f(x) =<br />

x→+∞<br />

0<br />

e −t2<br />

2 dt ,<br />

Γ<br />

<br />

1<br />

=<br />

2<br />

√ π.<br />

dt,


AMPLIACIÓN DE CÁLCULO (Curso 2004/05) Convocatoria de septiembre 13.09.05<br />

PROBLEMA 1 (4 puntos)<br />

Considérese la curva Γ definida por las ecuaciones<br />

x 2 + y 2 + z 2 = 9 , (x − 1) 2 + y 2 + z 2 = 6 ,<br />

con una orientación tal que su proyección sobre el plano Y Z se recorre con orientación positiva. Se<br />

pide:<br />

1) Hallar unas ecuaciones paramétricas de Γ.<br />

2) Sea G el campo en R 3 definido por<br />

Calcúlese la circulación de G a lo largo de Γ.<br />

G(x, y, z) = (sen 2 (x + y) , −z 3 , y 3 ) .<br />

3) Sea a ∈ R 3 un vector fijo y r = (x, y, z). Calcúlense los valores de p ∈ R para los cuales el campo<br />

vectorial F = (a · r) p · r es solenoidal en un cierto subconjunto de R 3 que se especificará.<br />

4) Se considera la superficie Σ formada por los segmentos que unen el punto (1, 0, 0) y los puntos de<br />

Γ, orientada de forma que los vectores normales a la misma tienen la primera componente negativa.<br />

Calcúlese el flujo del campo H(x, y, z) = (1/x 2 , y/x 3 , z/x 3 ) a través de Σ.<br />

Respuesta: Se entregará esta hoja y, a lo sumo, una adicional.<br />

1) La curva Γ es la intersección de dos esferas y por lo tanto será una circunferencia. Desarrollando<br />

la expresión (x − 1) 2 + y 2 + z 2 = 6 y utilizando que x 2 + y 2 + z 2 = 9 se obtiene que Γ se puede definir<br />

por las ecuaciones<br />

x = 2<br />

y 2 + z 2 = 5<br />

que muestran que Γ es una circunferencia situada sobre el plano x = 2. De aquí se deduce inmediatamente<br />

que la aplicación r(t) dada por<br />

x = 2<br />

y = √ 5 cos t<br />

z = √ 5sen t, t ∈ [0, 2π)<br />

es una posible parametrización de Γ (donde, como pide el enunciado, la orientación de la proyección<br />

de Γ sobre el plano Y Z se recorre positivamente).<br />

2) Γ es una curva C1 y G es un campo C∞ en R3 luego la circulación está bien definida. Teniendo en<br />

cuenta que r ′ (t) = 0, − √ 5sen t, √ 5 cos t , la primera componente del campo G no intervendrá en<br />

el cálculo de la circulación C pedida. Se tiene<br />

2π<br />

C = G · dr = G(r(t)) · r ′ (t)dt =<br />

=<br />

Γ<br />

2π<br />

= 25<br />

0<br />

2π<br />

0<br />

0<br />

<br />

(− √ 5sen t) 3<br />

cos 4 t + sen 4 t dt<br />

<br />

− √ <br />

5sen t + ( √ 5 cos t) 3<br />

√ <br />

5 cos t dt =


Utilizando las propiedades de las funciones seno y coseno<br />

se obtiene<br />

2π<br />

0<br />

cos 4 tdt =<br />

2π<br />

0<br />

sen 4 tdt = 4<br />

π<br />

2<br />

C = 200<br />

π<br />

2<br />

0<br />

cos 4 tdt<br />

integral que se puede resolver por varios métodos. Por ejemplo,<br />

C = 200 1<br />

Γ(5/2)Γ(1/2)<br />

β(5/2, 1/2) = 2001<br />

2 2 Γ(3)<br />

0<br />

= 200 1<br />

2<br />

cos 4 tdt<br />

3 1<br />

2<br />

2 Γ(1/2)2<br />

2<br />

= 75<br />

2 π<br />

3) Si a es nulo, para p ≥ 0 el campo es idénticamente nulo en R 3 , y por tanto solenoidal en dicho<br />

conjunto, y no estaría definido en ningún punto si p < 0. Sea entonces a ∈R 3 no nulo. Si p ≥ 0 el<br />

campo F(r) = (a · r) p r está definido y es de clase infinito en R 3 y si p < 0, F sólo está definido, y es<br />

de clase infinito, en el conjunto R 3 −{r : a · r =0}, es decir, en todo R 3 salvo en el plano perpendicular<br />

a a y que pasa por el origen. Entonces, en los puntos en los que F es C 1 ,<br />

divF(r) = grad( (a · r) p ) · r+ (a · r) p divr =<br />

= p (a · r) p−1 grad (a · r) + (a · r) p 3 =<br />

p (a · r) p−1 a · r+ (a · r) p 3 = (p + 3) (a · r) p<br />

donde se ha utilizado que grad(a · r) = a. Por tanto, divF(r) = 0 en un conjunto Ω ⊂ R 3 si y sólo si<br />

(p + 3) (a · r) p = 0 en Ω, con lo que:<br />

Para p = −3, F es solenoidal en Ω = R 3 − {r : a · r =0}<br />

Para cualquier p ≥ 0, F es solenoidal sobre el plano a · r =0<br />

4) La superficie Σ es una porción de superficie cónica con vértice (1, 0, 0) y orientada según ”la<br />

normal exterior”. Como x = 0 sobre todos los puntos de Σ, H es C1 en un entorno de Σ y el flujo<br />

está bien definido. El cálculo del flujo parametrizando Σ y calculando la integral doble asociada,<br />

aunque posible, parece a priori complicado, por lo que es razonable buscar posibles alternativas. Se<br />

observa que el campo H es un caso particular de la familia de campos definida en el apartado anterior.<br />

Concretamente corresponde a tomar p = −3 y a = (1, 0, 0). Por ello se sabe que H está definido y es<br />

de clase 1 en R3 − {r : x = 0} y además, divH(r) = 0 en dicho conjunto. Si la superficie Σ se cierra<br />

con otra superficie S de forma que el conjunto A delimitado por las mismas no contenga a ningún<br />

punto del plano x = 0, H es de clase 1 en A y por ello se puede aplicar el teorema de Gauss. Así<br />

<br />

H · dσ+ H · dσ = divH(r)dv = 0<br />

Σ<br />

S<br />

donde S está orientada según la normal exterior. Por comodidad, se puede tomar como superficie S<br />

la porción del plano x = 2 que está encerrada por Γ, estando su orientación definida por el vector<br />

n = (1, 0, 0). Entonces se tiene<br />

<br />

φ : =<br />

<br />

= −<br />

Σ<br />

S<br />

<br />

H · dσ = −<br />

1<br />

dσ = −1<br />

x2 4<br />

donde se ha utilizado que, sobre S, x = 2.<br />

Baremo de corrección (sobre 4 puntos)<br />

-Apdo. 1. 0.8 ptos. -Apdo. 2. 0.8 ptos.<br />

-Apdo. 3. 1 pto. -Apdo. 4. 1.4 ptos.<br />

<br />

S<br />

S<br />

A<br />

<br />

H · dσ = − H · ndσ =<br />

S<br />

dσ = − 1<br />

Área(S) = −5π<br />

4 4


AMPLIACIÓN DE CÁLCULO (Curso 2005/2006) Convocatoria de febrero 07.02.06<br />

PROBLEMA 1 (3 puntos)<br />

Sea Σ la superficie definida por la ecuación cartesiana a |x|+b |y|+z 2 = 1, donde a y b son constantes<br />

positivas, y sea Ω el sólido encerrado por ella.<br />

1. Se sabe que la densidad en cada punto de Ω es proporcional a la suma de las distancias del<br />

punto a los planos XZ e Y Z. Se pide determinar su masa calculando explícitamente una integral<br />

triple.<br />

2. Sobre Σ hay definida una distribución de carga de forma que la densidad superficial de carga<br />

en cada punto P de Σ es proporcional a la distancia del origen al plano tangente a Σ en P . Se<br />

pide calcular la carga total de Σ.<br />

Respuesta: Se entregará esta hoja y, a lo sumo, una adicional.<br />

De la ecuación que define a Σ se sigue que (x, y, z) ∈ Σ si y sólo si (−x, y, z) ∈ Σ. Análogamente<br />

(x, y, z) ∈ Σ ⇔ (x, −y, z) ∈ Σ y (x, y, z) ∈ Σ ⇔ (x, y, −z) ∈ Σ por lo que Σ es simétrica respecto de<br />

los tres planos coordenados. Además, cortando Σ por planos z = cte, se obtiene a |x| + b |y| = 1 − z 2 ,<br />

por lo que Σ sólo está definida cuando 1 − z 2 ≥ 0, es decir, para z ∈ [−1, 1]. Para algunos de estos<br />

valores de z, las curvas intersección son las que muestra la figura de la izquierda<br />

-1<br />

-0.5<br />

0<br />

0.5<br />

1<br />

-1 -0.5 0<br />

0.5<br />

0<br />

-0.5<br />

1 -1<br />

1<br />

0.5<br />

y por tanto Σ es la superficie cerrada<br />

(simétrica con respecto al origen) que<br />

se muestra a la derecha :<br />

<br />

Se pretende calcular la integral triple M = Ω ρ(x, y, z)dxdydz donde ρ(x, y, z) = K1 (|x| + |y|)<br />

y K1 es una constante. Puesto que ρ es una función par en x, par en y y par en z y Ω es simétrico<br />

respecto de los tres planos coordenados, se puede escribir M = 8 Ωˆ ρ(x, y, z)dxdydz donde ˆ Ω es<br />

la porción de Ω contenida en el primer octante. Además, puesto que en dicho octante x, y y z son<br />

no negativas, se puede escribir<br />

M = 8K1<br />

<br />

(x + y) dxdydz<br />

ˆΩ<br />

Puesto que la ecuación de Σ no se modifica si se intercambian a por b y x por y, parece razonable<br />

<br />

expresar M = 8K1 ˆΩ xdxdydz + 8K1 ˆΩ ydxdydz, de forma que el valor de la segunda integral<br />

será igual al de la primera intercambiando a por b. Calculemos pues I := Ωˆ xdxdydz. Por el<br />

teorema de Fubini, fijando primero z e integrando en x e y se obtiene<br />

<br />

I = xdxdydz =<br />

ˆΩ<br />

1 <br />

0<br />

Tz<br />

<br />

xdxdy dz<br />

donde Tz es el triángulo definido por ax + by ≤ 1 − z2 , x, y ≥ 0. Para evaluar la primera integral<br />

<br />

se puede tener en cuenta que, por definición de centro de gravedad, xdxdy = xG(Tz)Area(Tz),<br />

Tz<br />

donde xG(Tz) es la coordenada x del centro de gravedad de Tz , supuesto éste homogéneo. Como


el centro de gravedad de un triángulo está situado a 1/3 de su altura, se obtiene que Area(Tz) =<br />

1<br />

2<br />

(1−z2 ) (1−z<br />

a<br />

2 )<br />

b<br />

y xG(Tz) = 1 (1−z<br />

3<br />

2 )<br />

a<br />

por lo que<br />

<br />

Tz<br />

xdxdy = 1<br />

6a2 <br />

1 − z<br />

b<br />

23 Si no se trabaja con el centro de gravedad de Tz, se puede utilizar Fubini para, integrando primero<br />

en y y luego en x escribir<br />

<br />

Tz<br />

xdxdy =<br />

1−z 2<br />

a<br />

0<br />

⎛<br />

x ⎝<br />

1−z 2 −ax<br />

b<br />

= · · · = 1<br />

6a2 <br />

1 − z<br />

b<br />

23 0<br />

⎞<br />

dy⎠<br />

dx = 1<br />

b<br />

1−z 2<br />

a<br />

con lo que, como debe ser, se llega al mismo resultado. En definitiva<br />

I =<br />

=<br />

1<br />

6a2 1 <br />

1 − z<br />

b 0<br />

23 dz = 1<br />

6a2b 1<br />

6a2 16<br />

b 35 =<br />

8<br />

105a2b 1<br />

0<br />

0<br />

x(1 − z 2 − ax)dx =<br />

<br />

1 − 3z 2 + 3z 4 − z 6<br />

dz = · · · =<br />

Usando la propiedad de simetría a la que se aludió anteriormente,<br />

<br />

ydxdydz =<br />

ˆΩ<br />

8<br />

105b 2 a<br />

Si uno no se da cuenta de dicha propiedad de simetría, se puede proceder de la siguiente forma<br />

<br />

ydxdydz =<br />

ˆΩ<br />

=<br />

1 <br />

0<br />

⎛<br />

1<br />

0<br />

⎝ 1<br />

a<br />

Tz<br />

1−z 2<br />

b<br />

Por lo tanto, la masa de Ω es<br />

0<br />

<br />

ydxdy dz =<br />

M = 64K1<br />

105<br />

⎛<br />

1 1−z<br />

⎝<br />

0<br />

2<br />

b<br />

0<br />

⎞<br />

⎛<br />

y ⎝<br />

1−z 2 −by<br />

a<br />

y(1 − z 2 − ay)dy⎠<br />

dz = 1<br />

6b2a <br />

1<br />

a2 1<br />

+<br />

b ab2 <br />

0<br />

1<br />

= 64K1<br />

105a2 (a + b)<br />

b2 0<br />

⎞<br />

⎞<br />

dx⎠<br />

dy⎠<br />

dz =<br />

<br />

1 − z 2 3<br />

dz = · · · =<br />

8<br />

105b 2 a<br />

2) En primer lugar hay que hacer constar que en las aristas de Σ el plano tangente no está definido<br />

y por ello tampoco lo está la densidad de carga. Sin embargo como el conjunto de dichas aristas tiene<br />

contenido de Jordan nulo en R 2 , la circunstancia anterior no influye en el cálculo de la carga total<br />

de Σ.<br />

Para cada punto (x, y, z) de Σ la distancia al origen del plano tangente a Σ en (x, y, z) es igual a<br />

la que se obtiene al considerar los puntos (−x, y, z), (x, −y, z) y (x, y, −z) por lo que por simetría se<br />

puede escribir que la carga total Q de Σ es<br />

Q = 8 ˆ Q<br />

donde ˆ Q es la carga total de la porción ˆ Σ de Σ contenida en el primer octante.<br />

Para calcular el plano Π tangente a ˆ Σ en el punto (x, y, z) se puede calcular el gradiente de la<br />

función g(x, y, z) := ax + by + z 2 − 1, ∇g(x, y, z) = (a, b, 2z) y entonces (X, Y, Z) pertenece a Π si y<br />

sólo si verifica la ecuación<br />

a (X − x) + b (Y − y) + 2z (Z − z) = 0


es decir,<br />

aX + bY + 2zZ − (ax + by + 2z 2 ) = 0<br />

que, utilizando que (x, y, z) ∈ ˆ Σ y que por ello ax + by + z 2 = 1, se puede escribir<br />

La distancia de Π al origen es<br />

aX + bY + 2zZ − (1 + z 2 ) = 0<br />

|a0 + b0 + 2z0 − (1 + z 2 )|<br />

√ a 2 + b 2 + 4z 2<br />

=<br />

1 + z 2<br />

√ a 2 + b 2 + 4z 2<br />

y por ello la densidad superficial de carga q(x, y, z) definida en Σ es<br />

Por tanto<br />

q(x, y, z) = K2<br />

1 + z 2<br />

√ a 2 + b 2 + 4z 2<br />

Q = 8 ˆ <br />

<br />

1 + z<br />

Q = 8 q(x, y, z)dσ = 8K2<br />

ˆΣ<br />

ˆΣ<br />

2<br />

√<br />

a2 + b2 + 4z2 dσ<br />

Para parametrizar ˆ Σ, la mejor opción es expresar la superficie mediante una ecuación cartesiana<br />

explícita. Además, y puesto que g(x, y, z) es cuadrática en z pero lineal en x e y, es conveniente<br />

despejar o bien x o bien y como función de las otras variables. Por la simetría del problema en<br />

x e y ambas opciones son equivalentes en cuanto a dificultad. Así por ejemplo expresamos x =<br />

h(y, z) = 1<br />

a (1 − by − z2 ), (y, z) ∈ B donde B es la proyección de ˆ Σ sobre el plano yz, es decir,<br />

B = <br />

(y, z) : 0 ≤ z ≤ 1, 0 ≤ y ≤ 1<br />

b (1 − z2 ) <br />

. Puesto que ∂h<br />

∂y<br />

Q = 8K2<br />

= 8K2<br />

= 8K2<br />

a<br />

= 8K2<br />

ab<br />

= b<br />

a<br />

y ∂h<br />

∂z<br />

−2z = se tiene<br />

a<br />

<br />

1 + b2 4z2<br />

+ dydz =<br />

a2 a2 <br />

(1 + z<br />

a B<br />

2 )dydz =<br />

<br />

1 + z<br />

ˆΣ<br />

2<br />

<br />

1 + z<br />

√ dσ = 8K2<br />

a2 + b2 + 4z2 B<br />

2<br />

√<br />

a2 + b2 + 4z2 <br />

1 + z<br />

B<br />

2 1√<br />

8<br />

√ a2 + b2 + 4z2dydz =<br />

a2 + b2 + 4z2 a<br />

1<br />

(1 + z<br />

0<br />

2 1<br />

b<br />

)<br />

(1−z2 <br />

)<br />

dy dz =<br />

0<br />

8K2<br />

1<br />

(1 + z<br />

ab 0<br />

2 )(1 − z 2 )dz =<br />

1<br />

0<br />

(1 − z 4 )dz = 32K2<br />

5ab<br />

Si, a pesar de que no es la mejor opción, se parametriza ˆ Σ despejando z en función de x y de<br />

y, en la forma z = β(x, y) = √ 1 − ax − by, (x, y) ∈ D = {(x, y) : ax + by ≤ 1, x, y ≥ 0} se puede<br />

proceder como sigue. Puesto que<br />

∂β<br />

∂x =<br />

−a<br />

2 √ 1 − ax − by<br />

;<br />

∂β<br />

∂y =<br />

−b<br />

2 √ 1 − ax − by<br />

entonces, usando que, en D, |1 − ax − by| = 1 − ax − by :<br />

<br />

1 + z<br />

Q = 8K2 ˆΣ<br />

2<br />

√ dσ =<br />

a2 + b2 + 4z2 <br />

1 + (1 − ax − by)<br />

= 8K2 <br />

D a2 + b2 <br />

<br />

<br />

a<br />

1 +<br />

+ 4(1 − ax − by)<br />

2<br />

4(1 − ax − by) +<br />

b2 4(1 − ax − by) dxdy<br />

<br />

<br />

1 + (1 − ax − by) 1<br />

= 8K2 <br />

D<br />

2<br />

= 4K2<br />

a2 + b2 + 4(1 − ax − by)<br />

<br />

<br />

= 4K2<br />

D<br />

D<br />

a2 + b2 + 4(1 − ax − by)<br />

√ dxdy =<br />

1 − ax − by<br />

1 + (1 − ax − by)<br />

√ 1 − ax − by dxdy =<br />

dxdy<br />

√ 1 − ax − by + 4K2<br />

<br />

D<br />

<br />

1 − ax − bydxdy


Calculemos I := D (1 − ax − by)−1/2dxdy aplicando el teorema de Fubini, integrando primero<br />

respecto de y y luego respecto de x:<br />

<br />

I : =<br />

=<br />

Análogamente<br />

Por tanto<br />

(1 − ax − by) −1/2 dxdy =<br />

1/a<br />

D<br />

0<br />

1/a <br />

2<br />

0 b (1 − ax − by)1/2 | y=0<br />

y= 1<br />

b (1−ax)<br />

<br />

= 2 2<br />

b 3a (1 − ax)3/2 | x=0<br />

x= 1 =<br />

a<br />

4<br />

3ab<br />

<br />

J : =<br />

=<br />

(1 − ax − by) 1/2 dxdy =<br />

1/a<br />

D<br />

0<br />

1/a <br />

2<br />

0 3b (1 − ax − by)3/2 | y=0<br />

y= 1<br />

b (1−ax)<br />

<br />

= 2 2<br />

3b 5a (1 − ax)5/2 | x=0<br />

x= 1 =<br />

a<br />

4<br />

15ab<br />

Q = 4K2 (I + J) = 4K2<br />

1<br />

b (1−ax)<br />

0<br />

dx = 2<br />

b<br />

1/a<br />

0<br />

1<br />

b (1−ax)<br />

0<br />

dx = 2<br />

3b<br />

(1 − ax − by) −1/2 dy<br />

(1 − ax) 1/2 dx =<br />

(1 − ax − by) 1/2 dy<br />

1/a<br />

<br />

4 4<br />

+ =<br />

3ab 15ab<br />

32K2<br />

5ab<br />

0<br />

<br />

(1 − ax) 3/2 dx =<br />

<br />

dx =<br />

dx =


AMPLIACIÓN DE CÁLCULO (Curso 2005/2006) Convocatoria de febrero 07.02.06<br />

PROBLEMA 2 (3 puntos)<br />

1.1 Sea f un campo escalar de clase 2 en R 3 . Enúnciese y justifíquese una condición necesaria y<br />

suficiente sobre f para que exista un campo vectorial F en R 3 de forma que ∇f = rot F.<br />

1.2 Sea F un campo vectorial de clase 2 en R 3 . Enúnciese y justifíquese una condición necesaria y<br />

suficiente sobre F para que exista un campo escalar f en R 3 de forma que rot F = ∇f.<br />

Nota: Los resultados teóricos que se precisen deben enunciarse con todo rigor pero sin demostración.<br />

2.1 Determínese g : R → R con g(0) = 0, g(1) = −2 para que el campo escalar f(x, y, z) =<br />

x 2 + g(y) + z 2 cumpla la condición hallada en (1.1) y, en tal caso, calcúlese un potencial vector<br />

de ∇f de la forma Q(x, z)j + R(x, y)k.<br />

2.2 Sean Q, R los campos escalares calculados en (2.1). Determínense todos los campos escalares<br />

P : R 3 → R para los cuales F(x, y, z) = P (x, y, z)i + Q(x, z)j + R(x, y)k satisface la condición<br />

hallada en (1.2).<br />

Respuesta:<br />

1.1 (0,5 puntos) Para que un campo vectorial de clase 1 en un abierto estrellado (en este caso todo<br />

el espacio R 3 ) sea un rotacional es necesario y suficiente que sea solenoidal; esto es, que su<br />

divergencia sea nula. Ahora bien, div ∇f = ∆f. Así pues, la condición sobre f es que sea un<br />

campo escalar armónico. En tal caso F es un potencial vector de ∇f.<br />

1.2 (0,5 puntos) Un campo vectorial de clase 1 en un abierto simplemente conexo es un gradiente<br />

si, y solamente si, es irrotacional; es decir, su rotacional es nulo. Por lo tanto, la condición<br />

sobre F es que rot rot F = 0. Se dice que f es un potencial escalar de rot F.<br />

2.1 (1 punto) Tenemos que imponer que el campo f(x, y, z) = x 2 +g(y)+z 2 sea armónico; calculemos<br />

su laplaciano:<br />

∆f(x, y, z) = ∂2 f<br />

∂x 2 + ∂2 f<br />

∂y 2 + ∂2 f<br />

∂z 2 = 2 + g′′ (y) + 2,<br />

el laplaciano es nulo si y solo si g ′′ (y) = −4, integramos: g ′ (y) = −4y + a, g(y) = −2y 2 + ay + b<br />

e imponemos las condiciones g(0) = 0, g(1) = −2, con lo que se llega a<br />

g(0) = a = 0, g(1) = −2 + a + b = −2 ⇒ a = b = 0, g(y) = −2y 2 .<br />

Por tanto el campo buscado es f(x, y, z) = x 2 − 2y 2 + z 2 . Ahora se pide calcular un potencial<br />

vector del gradiente de f de la forma Q(x, z)j+R(x, y)k; calculemos el rotacional de este campo<br />

e igualémoslo al gradiente de f:<br />

<br />

<br />

<br />

<br />

<br />

<br />

<br />

<br />

<br />

i<br />

∂<br />

∂x<br />

0<br />

j<br />

∂<br />

∂y<br />

Q(x, z)<br />

<br />

k <br />

<br />

∂ <br />

<br />

= 2x i − 4y j + 2z k ⇐⇒<br />

∂z <br />

<br />

R(x, y) <br />

∂R ∂Q<br />

∂Q<br />

− = 2x, −∂R = −4y, = 2z.<br />

∂y ∂z ∂x ∂x


Integremos ∂R/∂x y ∂Q/∂x en la variable x:<br />

Y ahora impongamos la primera relación:<br />

∂R<br />

∂y<br />

∂R<br />

= 4y ⇒ R(x, y) = 4xy + r(y),<br />

∂x<br />

∂Q<br />

= 2z ⇒ Q(x, z) = 2xz + q(z).<br />

∂x<br />

− ∂Q<br />

∂z = 2x ⇒ 4x + r′ (y) − 2x − q ′ (z) = 2x ⇐⇒ r ′ (y) − q ′ (z) = 0 ⇐⇒ r ′ = q ′ = cte<br />

Si el valor de la constante es a se tiene r(y) = ay + b y q(z) = az + c con a, b, c constantes<br />

que podemos elegir arbitrariamente, en particular un potencial vector de ∇f es G(x, y, z) =<br />

2xzj + 4xyk.<br />

2.2 (1 punto) Si F es de clase 2, tenemos que imponer que P también sea de clase 2, esto nos<br />

permitirá integrar las ecuaciones en derivadas parciales que surjan, asimismo nos garantiza la<br />

igualdad de las derivadas parciales cruzadas de segundo orden.<br />

Tenemos que imponer que rot rot F se anule en R 3 ; ahora bien, como G(x, y, z) = Q(x, z)j +<br />

R(x, y)k es un potencial vector de un gradiente (rot G = ∇f), se cumple rot rot G = 0, además<br />

el rotacional es un operador lineal:<br />

rot F = rot(P i) + rot G = rot(P i) + ∇f,<br />

por lo que basta con calcular los campos P para los cuales rot rot(P i) = 0. Calculemos, en<br />

primer lugar, rot(P i):<br />

<br />

<br />

<br />

i j k <br />

<br />

<br />

∂ ∂ ∂ <br />

<br />

rot(P i) = <br />

=<br />

<br />

∂x ∂y ∂z <br />

<br />

P (x, y, z) 0 0 <br />

∂P ∂P<br />

j −<br />

∂z ∂y k<br />

y ahora rot rot(P i) e igualemos a cero:<br />

<br />

∂P ∂P<br />

rot j −<br />

∂z ∂y k<br />

<br />

<br />

<br />

<br />

<br />

<br />

= <br />

<br />

<br />

<br />

<br />

i<br />

∂<br />

∂x<br />

0<br />

j<br />

∂<br />

∂y<br />

∂P<br />

∂z<br />

k<br />

∂<br />

∂z<br />

−∂P<br />

∂y<br />

<br />

<br />

<br />

<br />

<br />

2<br />

∂ P<br />

= −<br />

<br />

∂y<br />

<br />

<br />

2 + ∂2P ∂z2 <br />

i + ∂2P ∂x∂y j + ∂2P k = 0.<br />

∂x∂z<br />

De ∂ 2 P/∂x∂y = 0 se deduce que ∂P/∂x no es función de y, existirá una función H1 de dos<br />

variables de forma que ∂P/∂x = H1(x, z). Derivamos respecto de z:<br />

∂ 2 P<br />

∂x∂z<br />

Integremos ∂P/∂x = H2(x) en la variable x,<br />

= ∂H1<br />

∂z = 0 ⇒ H1(x, z) = H2(x).<br />

P (x, y, z) = h(x) + C(y, z)<br />

donde h es una primitiva de H2 y C es arbitraria. Por último, al imponer la nulidad de la primera<br />

componente del doble rotacional: ∂ 2 P /∂y 2 + ∂ 2 P /∂z 2 = 0 se concluye que el laplaciano de C<br />

es nulo, así pues las funciones P buscadas son<br />

P (x, y, z) = h(x) + C(y, z), h ∈ C 2 (R), C ∈ C 2 (R 2 ), ∆C = 0.<br />

Gabriela Sansigre.


AMPLIACIÓN DE CÁLCULO (Curso 2005/2006) Convocatoria de febrero 07.02.06<br />

NOMBRE ......................... APELLIDOS ...........................................<br />

PROBLEMA 3 (4 puntos)<br />

Número de matrícula........................<br />

Se considera el lóbulo Γ de la lemniscata ρ 2 = a 2 cos 2θ contenido en el semiplano x ≥ 0, siendo<br />

a > 0 una constante. Se pide:<br />

(1) Dados dos puntos A,B ∈ Γ, de ángulos polares respectivos ϕ < ψ, calcular las coordenadas<br />

(x0,y0) del centro de masas G del arcoAB en función de la longitud l del arco (que no es necesario<br />

calcular en ningún momento en este problema), de ϕ y de ψ.<br />

(2) Si O es el origen de coordenadas, demostrar que la recta OG es la bisectriz del ánguloAOB.<br />

Para ello, se sugiere considerar el arco C de una circunferencia centrada en O, de radio arbitrario,<br />

formado por los puntos de ángulo polar θ ∈ [ϕ,ψ], calcular las coordenadas (x ∗ 0,y ∗ 0) del centro de<br />

masas G ∗ de C, y comparar las posiciones de G y G ∗ respecto de O.<br />

(3) Calcular las coordenadas (xs,ys) del centro de masas del dominio plano encerrado por el<br />

lóbulo de la lemniscata. Hallar el volumen engendrado por ese dominio al girar 360 ◦ en torno al eje<br />

OY .<br />

Respuesta: Se entregará esta hoja y, a lo sumo, una adicional.<br />

(1) Sean<br />

− π<br />

4<br />

≤ ϕ < ψ ≤ π<br />

4 .<br />

Derivando respecto de θ en la ecuación de la lemniscata ρ 2 = a 2 cos 2θ, se tiene<br />

de donde<br />

2ρρ ′ = −2a 2 sen 2θ ⇒ ρ ′ = − a2 sen 2θ<br />

ρ<br />

ρ 2 + ρ ′2 = a 2 cos 2θ + a 4 sen 2 2θ<br />

a 2 cos 2θ<br />

= a2<br />

cos 2θ .<br />

Por tanto, a título informativo y sin completar el cálculo, la longitud l del arcoAB es<br />

l =<br />

ψ<br />

Las coordenadas del centro de masas verifican:<br />

lx0 =<br />

=<br />

ly0 =<br />

AB<br />

ψ<br />

= a 2<br />

ϕ<br />

ϕ<br />

ψ<br />

AB<br />

<br />

ρ(θ) 2 + ρ ′ (θ) 2 ψ dθ<br />

dθ = a √ .<br />

ϕ cos 2θ<br />

xds =<br />

ψ<br />

ϕ<br />

a √ cos 2θ cos θ<br />

ϕ<br />

<br />

ρ(θ) cos θ ρ(θ) 2 + ρ ′ (θ) 2 dθ<br />

a<br />

√ cos 2θ dθ<br />

cos θ dθ = a 2 [sen θ] ψ<br />

ϕ = a2 (sen ψ − sen ϕ).<br />

y ds =<br />

ψ<br />

ϕ<br />

<br />

ρ(θ) sen θ ρ(θ) 2 + ρ ′ (θ) 2 dθ<br />

1<br />

,


Por tanto,<br />

y<br />

0<br />

ψ<br />

ϕ<br />

B<br />

G<br />

•<br />

Γ A<br />

a<br />

C<br />

G∗ •<br />

Figura 1: Centro de masas de un arco de lemniscata<br />

=<br />

ψ<br />

= a 2<br />

ϕ<br />

ψ<br />

a √ cos 2θ sen θ<br />

ϕ<br />

G(x0,y0) =<br />

a<br />

√ cos 2θ dθ<br />

sen θ dθ = a 2 [− cos θ] ψ<br />

ϕ = a2 (cos ϕ − cosψ).<br />

a 2<br />

l<br />

(senψ − sen ϕ), a2<br />

l<br />

R<br />

x<br />

(cos ϕ − cos ψ)<br />

(2) Sea C el arco de circunferencia de radio R (podemos suponer R > a, aunque no es indispensable),<br />

formado por los puntos de ángulo polar θ ∈ [ϕ,ψ] (Figura 1). Una parametrización de C<br />

es<br />

<br />

x(θ) = R cos θ<br />

r(θ) ≡<br />

θ ∈ [ϕ,ψ],<br />

y(θ) = R sen θ<br />

de donde |r ′ (θ)| = R para todo θ ∈ [ϕ,ψ].<br />

La longitud del arco C es l ∗ = R(ψ − ϕ). Las coordenadas del centro de masas G ∗ de C son:<br />

Se tiene entonces<br />

x ∗ 0 =<br />

<br />

1<br />

xds =<br />

R(ψ − ϕ) C<br />

<br />

<br />

1 ψ<br />

R cos θ R dθ<br />

R(ψ − ϕ) ϕ<br />

= R<br />

R<br />

[sen θ]ψ ϕ = (sen ψ − sen ϕ).<br />

ψ − ϕ ψ − ϕ<br />

y ∗ <br />

<br />

1<br />

1 ψ<br />

0 = y ds =<br />

R(ψ − ϕ) C R(ψ − ϕ) ϕ<br />

= R<br />

R<br />

[− cos θ]ψ ϕ = (cosϕ − cos ψ).<br />

ψ − ϕ ψ − ϕ<br />

x ∗ 0<br />

x0<br />

y∗ 0<br />

y0<br />

Rl<br />

=<br />

(ψ − ϕ)a2 Rl<br />

= ,<br />

(ψ − ϕ)a2 R sen θ R dθ<br />

y ambos cocientes son iguales. Por tanto, O, G y G ∗ son colineales, y los ángulos polares de G y G ∗<br />

coinciden.<br />

Ahora bien, es obvio, por la simetría del arco C, que el ángulo polar de G ∗ es<br />

ϕ + ψ<br />

2<br />

2<br />

,<br />

.


por tanto, este es también el ángulo polar de G, luego OG es la bisectriz del ánguloAOB.<br />

(3) Por simetría, ys = 0. Sea D el dominio encerrado por el lóbulo de la lemniscata. Su área es<br />

<br />

Área =<br />

Para calcular xs necesitamos la integral:<br />

Por tanto,<br />

I : =<br />

<br />

= 2<br />

<br />

xdxdy =<br />

D<br />

π/4<br />

0<br />

π/4<br />

cos θ dθ<br />

= 2<br />

= a 2<br />

= a2<br />

2<br />

D ′<br />

√<br />

a cos 2θ<br />

0<br />

<br />

dxdy =<br />

D<br />

π/4<br />

0<br />

π/4<br />

0<br />

dθ<br />

D ′<br />

√<br />

a cos 2θ<br />

0<br />

cos 2θ dθ<br />

[sen 2θ]π/4<br />

0<br />

ρ 2 cos θ dρdθ<br />

ρ 2 dρ<br />

= 2a3<br />

3 0<br />

cos θ(cos 2θ) 3/2 dθ = 2a3<br />

3<br />

(cambio u := sen θ)<br />

1/ √ 2<br />

= 2a3<br />

(1 − 2u<br />

3 0<br />

2 ) 3/2 du<br />

<br />

cambio u = 1<br />

<br />

√ sen t<br />

2<br />

π/2<br />

ρdρdθ<br />

ρdρ<br />

= a2<br />

2 .<br />

π/4<br />

= 2a3<br />

(1 − sen<br />

3 0<br />

2 t) 3/2 1 √ costdt<br />

2<br />

√<br />

2<br />

=<br />

3 a3<br />

π/2<br />

cos<br />

0<br />

4 √<br />

2<br />

t dt =<br />

6 a3 <br />

1 5<br />

B ,<br />

2 2<br />

√<br />

2<br />

=<br />

6 a3<br />

<br />

1 5<br />

Γ Γ √<br />

2 2 2<br />

=<br />

Γ(3) 6 a3<br />

3 1<br />

2 2 Γ<br />

<br />

1<br />

2<br />

2<br />

= πa3√ 2<br />

16<br />

.<br />

xs = I<br />

Área = πa√2 .<br />

8<br />

Por el Teorema de Guldin, el volumen engendrado es:<br />

0<br />

cos θ(cos 2 θ − sen 2 θ) 3/2 dθ<br />

2<br />

Vol = 2πxs Área = π2 a 3√ 2<br />

8<br />

3<br />

.


AMPLIACIÓN DE CÁLCULO (Curso 2005/2006) Convocatoria de junio 20.06.06<br />

PROBLEMA 1 (4 puntos)<br />

Sean a = (a1,a2,a3) un vector constante y no nulo y k una constante real estrictamente positiva.<br />

Considérese el campo vectorial F(r) definido por:<br />

donde r = (x,y,z).<br />

F(r) =<br />

(a1x,a2y,a3z)<br />

(24x2 + 4y2 + 2z2 ,<br />

) k<br />

1. 1.a) Determínense razonadamente todos los valores posibles del vector a y de la constante k<br />

para los que el campo F(r) es irrotacional, indicando el mayor dominio posible en el que<br />

posee esta propiedad. (1 punto).<br />

1.b) Sea Σ una superficie regular con borde orientado Γ que no contiene el origen. Para los<br />

valores de a y k obtenidos en 1.a), calcúlese razonadamente la circulación del campo F(r)<br />

sobre el borde Γ. (0,5 puntos).<br />

2. 2.a) Sean Σ1 y Σ2 las superficies esféricas de ecuaciones cartesianas (x − 1) 2 + y 2 + z 2 = 9 y<br />

(x − 1) 2 + y 2 + z 2 = 1/4, respectivamente. Para a = (2, 2, 2) y k = 3/2, calcúlese el flujo<br />

del campo F(r) a través de la cara exterior de Σ1 y de Σ2. (1,5 puntos)<br />

2.b) Determínense razonadamente todos los valores posibles del vector a y de la constante<br />

k para los que el campo F(r) es solenoidal, indicando el dominio en el que posee esta<br />

propiedad. (1 punto).<br />

Respuesta: Se entregará esta hoja y, a lo sumo, una adicional.<br />

1.a) Para determinar los valores de a y k que se piden, calculamos el rotacional del campo:<br />

rot F(r) = ∇ × F(r) =<br />

4<br />

(24x2 + 4y2 + 2z2 ) k+1<br />

<br />

<br />

(a2 − 2a3)kyz , (a1 − 12a3)kxz , (2a1 − 12a2)kxy<br />

expresión que es válida en todo R 3 salvo el origen puesto que k > 0.<br />

Para que el campo tenga rotacional nulo (sea irrotacional) sus tres componentes deben anularse.<br />

Teniendo en cuenta que k no puede ser cero, esto da lugar a un sistema lineal homogéneo de tres<br />

ecuaciones con tres incógnitas, cuya expresión es:<br />

⎛<br />

0<br />

⎜<br />

⎝1<br />

1<br />

0<br />

⎞ ⎛ ⎞ ⎛ ⎞<br />

−2 a1 0<br />

⎟ ⎜ ⎟ ⎜ ⎟<br />

−12⎠<br />

⎝a2⎠<br />

= ⎝0⎠<br />

.<br />

2 −12 0 a3 0<br />

Dado que el rango de la matriz de coeficientes de este sistema es 2, existen infinitas soluciones que se<br />

obtienen expresando dos de las incógnitas en términos de la tercera. Por ejemplo, eligiendo las dos<br />

primeras ecuaciones se tiene:<br />

a1 = 12a3 , a2 = 2a3 .<br />

Así pues, el campo F(r) dado es irrotacional para todo valor de k > 0 y para<br />

y posee esta propiedad en R 3 \{(0, 0, 0)}.<br />

a = a3(12, 2, 1) con a3 ∈ R,


1.b) De partida se tiene: ∇ × F(r) = 0 en R3 \{(0, 0, 0)}. Dado que, por hipótesis, se trata de<br />

una superficie regular con borde orientado, podemos aplicar el teorema de Stokes, según el cual:<br />

<br />

<br />

F(r) · dr = [∇ × F(r)] · dr = 0,<br />

Γ<br />

Σ<br />

puesto que la superficie no contiene el origen.<br />

También se puede razonar del siguiente modo: Puesto que la curva Γ es el borde de una superficie<br />

regular que no contiene el origen, es necesariamente una curva cerrada en R 3 que tampoco lo contiene.<br />

Esto permite afirmar que siempre se puede encontrar un dominio estrellado (o estelar), Ω ⊂ R 3 , que<br />

contenga la curva y no el origen. En Ω (que es estelar y contiene a Γ) el campo es irrotacional y, por<br />

tanto, conservativo lo que implica que su circulación a lo largo de cualquier curva cerrada contenida<br />

en él es cero.<br />

2.a) A la hora de calcular el flujo de un campo vectorial a través de una superficie es siempre<br />

conveniente comprobar, antes de aplicar la definición de flujo, si el campo es o no solenoidal. Ello<br />

se justifica por la existencia del Teorema de Gauss cuya aplicación, si es posible, puede simplificar<br />

considerablemente los cálculos.<br />

En este caso, para los valores dados del vector a = (2, 2, 2) y la constante k = 3/2, se comprueba<br />

fácilmente que, en efecto, se trata de un campo solenoidal, puesto que ∇ · F(r) = 0. Además, el<br />

campo dado presenta esta propiedad en R 3 \{(0, 0, 0)}.<br />

• Consideremos, en primer lugar la superficie Σ1 de ecuación cartesiana (x − 1) 2 + y 2 + z 2 = 9.<br />

Como el dominio de R 3 limitado por esta superficie esférica contiene el origen y la divergencia del<br />

campo no está definida en ese punto, no es posible aplicar el Teorema de Gauss directamente en este<br />

caso. La integral de volumen de la divergencia del campo extendida al dominio limitado por Σ1 no<br />

está definida.<br />

Sin embargo si es posible utilizarlo para sustituir la superficie dada por otra que simplifique los<br />

cálculos. El razonamiento es el siguiente: Dada la superficie esférica Σ1, siempre se puede encontrar<br />

otra regular y cerrada Σ ′ que no contenga el origen con la siguiente propiedad: el dominio limitado<br />

por Σ ′ en R 3 contiene el origen y, además, está contenido en el dominio limitado por Σ. En estas<br />

condiciones, la aplicación del Teorema de Gauss al dominio D ⊂ R 3 comprendido entre las dos<br />

superficies permite afirmar que el flujo a través de las caras exteriores de ambas superficies coincide,<br />

lo que es debido a que el campo vectorial es solenoidal en ese dominio D (que no contiene el origen).<br />

Podemos entonces elegir Σ ′ como la superficie de ecuación cartesiana 24x 2 + 4y 2 + 2z 2 = ɛ,<br />

donde siempre se puede encontrar un ɛ > 0 de forma que se cumplan las condiciones sobre Σ ′ antes<br />

mencionadas. Así pues:<br />

<br />

Σ1<br />

F(r) · dσ =<br />

<br />

Σ ′<br />

F(r) · dσ = 2<br />

ɛ 3/2<br />

<br />

Σ ′<br />

r · dσ ,<br />

donde r = (x,y,z) y, en la última igualdad, se ha tenido en cuenta que el valor del campo sobre Σ ′<br />

es, precisamente: F(r)| Σ ′ = 2<br />

ɛ3/2 r, motivo por el cual se ha elegido Σ′ en la forma dada.<br />

Falta por calcular la última integral extendida a Σ ′ y para ello podemos volver a aplicar el teorema<br />

de Gauss para escribir:<br />

2<br />

ɛ 3/2<br />

<br />

Σ ′<br />

r · dσ = 2<br />

ɛ 3/2<br />

<br />

V ′<br />

∇ · r dV = 6<br />

ɛ 3/2<br />

donde se ha tenido en cuenta que ∇ · r = 3.<br />

Finalmente, la última integral es el volumen del elipsoide 24x 2 + 4y 2 + 2z 2 ≤ ɛ, cuyos semiejes<br />

son ɛ<br />

24 , ɛ<br />

4 , ɛ<br />

πɛ3/2<br />

. Por tanto, el volumen es 2<br />

6 √ y el flujo pedido:<br />

3<br />

<br />

F(r) · dσ = π √ .<br />

3<br />

Σ1<br />

<br />

V ′<br />

dV


• En el caso de la superficie esférica Σ2, de ecuación cartesiana (x − 1) 2 + y 2 + z 2 = 1/4, resulta<br />

que el domino de R 3 que limita no contiene el origen y, por tanto, el campo F(r) es solenoidal en el.<br />

La aplicación directa de Teorema de Gauss nos permite concluir que el flujo de F(r) a través de Σ2<br />

es cero.<br />

2.b) Para determinar los valores del vector a y de la constante k > 0 para los que el campo<br />

vectorial dado es solenoidal, calculamos su divergencia:<br />

divF(r) = ∇ · F(r) =<br />

2<br />

(24x2 + 4y2 + 2z2 ) k−1<br />

<br />

12(a1 + a2 + a3 − 2a1k)x 2 +<br />

2(a1 + a2 + a3 − 2a2k)y 2 + (a1 + a2 + a3 − 2a3k)z 2<br />

.<br />

Al igualar a cero resulta el siguiente sistema lineal homogéneo de tres ecuaciones:<br />

⎛<br />

1 − 2k<br />

⎜<br />

⎝ 1<br />

1<br />

1 − 2k<br />

1<br />

1<br />

⎞ ⎛ ⎞<br />

a1<br />

⎟ ⎜ ⎟<br />

⎠ ⎝a2⎠<br />

=<br />

1 1 1 − 2k a3<br />

⎛ ⎞<br />

0<br />

⎜ ⎟<br />

⎝0⎠<br />

.<br />

0<br />

Para que admita soluciones distintas de la idénticamente nula, el determinate de la matriz de coeficientes<br />

debe ser cero. De ello se obtiene: 4k 2 (3 − 2k) = 0 =⇒ k = 3/2 ó k = 0, donde la última<br />

solución se descarta pues, por hipótesis, k es estrictamente positiva.<br />

Para k = 3/2 el rango de la matriz de coeficientes anterior es 2 y, por tanto, estamos en una<br />

situación análoga a la que se planteó en el apartado 1.a); es decir, existen infinitas soluciones que se<br />

obtienen expresando dos de las incógnitas en términos de la tercera. Por ejemplo:<br />

−2a1 + a2 = −a3<br />

a1 − 2a2 = −a3<br />

⎫<br />

⎪⎬<br />

⎪⎭ =⇒ a1 = a2 = a3 .<br />

Así pues, el campo F(r) dado es solenoidal para k = 3/2 y para<br />

a = a3(1, 1, 1) con a3 ∈ R,<br />

y, dado el valor de k, posee esta propiedad en R 3 \{(0, 0, 0)}.


AMPLIACIÓN DE CÁLCULO (Curso 2005/2006) Convocatoria de junio 20.06.06<br />

PROBLEMA 2 (3 puntos)<br />

1) Sea k < 0. La integral impropia ∞<br />

x k−1 cosx dx<br />

¿es convergente? ¿Es absolutamente convergente?<br />

2) Sea k < 0. La integral impropia ∞<br />

x k sen x dx<br />

¿es convergente?<br />

3) Aplicar lo anterior para estudiar la convergencia de la integral impropia<br />

∞ √<br />

2<br />

t sen(t )dt.<br />

Nota: cada apartado vale un punto.<br />

1<br />

1<br />

1<br />

Respuesta: Se entregará esta hoja y, a lo sumo, una adicional.<br />

1) Acotamos el valor absoluto del integrando:<br />

<br />

<br />

x k−1 cos x ≤ x k−1 = 1<br />

x 1−k<br />

y dado que 1 − k > 1 se tiene que<br />

∞ dx<br />

< ∞<br />

1 x1−k así que por el criterio de mayoración, se tiene que la integral ∞<br />

1 xk−1 cos x dx converge absolutamente,<br />

y, en particular, converge.<br />

2) Integremos por partes, con el fin de relacionar esta integral con la integral del apartado anterior.<br />

Lo hacemos sobre un intervalo finito [1,M], y luego hacemos tender M a infinito:<br />

M<br />

x<br />

1<br />

k sen x dx = cos 1 − M k M<br />

cos M + k x<br />

1<br />

k−1 cos x dx<br />

∞<br />

x k ∞<br />

sen x dx = cos 1 + k x k−1 cos x dx < ∞<br />

1<br />

donde, por un lado, se ha utilizado que, para k < 0,<br />

<br />

<br />

M k cos M k<br />

≤ M −−−→<br />

M→∞ 0<br />

y por otro lado, el hecho de que ∞<br />

1 xk−1 cos x dx es finita (por el apartado anterior).<br />

Por todo ello, ∞<br />

1 xk sen x dx es convergente para k < 0.<br />

3) Realizamos el cambio de variable t2 = x, que no varía los límites de integración:<br />

∞ √ ∞<br />

2<br />

t sen(t ) dt = 4√ 1<br />

x sen x<br />

1<br />

1 2 √ <br />

1 ∞<br />

dx = x<br />

x 2 1<br />

−1<br />

4 sen x dx<br />

que se trata de una integral impropia como la del apartado anterior, donde k = −1 < 0, por lo<br />

4<br />

que concluimos que es una integral convergente.<br />

1


AMPLIACIÓN DE CÁLCULO (Curso 2005/2006) Convocatoria de junio 20.06.06<br />

PROBLEMA 3 (3 puntos)<br />

Se considera el arco C de la lemniscata ρ 2 = a 2 cos 2θ contenido en el primer cuadrante (x ≥ 0,<br />

y ≥ 0), siendo a > 0 una constante. Se pide:<br />

1. Calcular las coordenadas (x0,y0) del centroide del arco C en función de la longitud l del arco<br />

(que no es necesario calcular en ningún momento en este problema).<br />

2. Se considera ahora el dominio plano acotado D limitado por C en el primer cuadrante. Se pide<br />

hallar las coordenadas (xD,yD) del centroide de D. Si aparece una función hiperbólica inversa,<br />

se deberá expresar el resultado en términos de la función logaritmo.<br />

Nota: La primera pregunta vale un punto y la segunda dos.<br />

Respuesta: Se entregará esta hoja y, a lo sumo, una adicional.<br />

1. Es bien sabido que, en el primer cuadrante, la lemniscata existe solamente para ángulos polares<br />

comprendidos entre 0 y π/4.<br />

Derivando respecto de θ en la ecuación de la lemniscata ρ 2 = a 2 cos 2θ, se tiene<br />

de donde<br />

2ρρ ′ = −2a 2 sen 2θ ⇒ ρ ′ = − a2 sen 2θ<br />

ρ<br />

ρ 2 + ρ ′2 = a 2 cos 2θ + a 4 sen 2 2θ<br />

a 2 cos 2θ<br />

= a2<br />

cos 2θ .<br />

Por tanto, a título informativo y sin completar el cálculo, la longitud l del arco C es<br />

l =<br />

π/4<br />

0<br />

<br />

ρ(θ) 2 + ρ ′ (θ) 2 π/4 dθ<br />

dθ = a √ .<br />

0 cos 2θ<br />

y<br />

0<br />

D<br />

C<br />

a x<br />

Figura 1: Arco de lemniscata en el primer cuadrante<br />

,


Las coordenadas del centroide del arco C verifican:<br />

π/4 <br />

lx0 = xds = ρ(θ) cos θ<br />

Por tanto,<br />

=<br />

C<br />

π/4<br />

= a 2<br />

<br />

ly0 =<br />

=<br />

0<br />

π/4<br />

0<br />

C<br />

π/4<br />

= a 2<br />

2. El área del dominio D es:<br />

0<br />

a √ cos 2θ cos θ<br />

y ds =<br />

0<br />

π/4<br />

0<br />

a<br />

√ cos 2θ dθ<br />

cos θ dθ = a 2 [sen θ] π/4<br />

0<br />

π/4<br />

0<br />

a √ cos 2θ sen θ<br />

ρ(θ) 2 + ρ ′ (θ) 2 dθ<br />

= a2<br />

√ 2 .<br />

<br />

ρ(θ) sen θ ρ(θ) 2 + ρ ′ (θ) 2 dθ<br />

a<br />

√ cos 2θ dθ<br />

sen θ dθ = a 2 [− cos θ] π/4<br />

0<br />

(x0,y0) =<br />

<br />

Área =<br />

=<br />

a 2<br />

l √ 2<br />

D<br />

π/4<br />

0<br />

= a2<br />

2<br />

, a2<br />

l<br />

<br />

dxdy =<br />

dθ<br />

π/4<br />

= a2<br />

4<br />

Para calcular xD necesitamos la integral:<br />

<br />

Ix : = xdxdy = ρ 2 cos θ dρdθ<br />

=<br />

D<br />

π/4<br />

0<br />

cos θ dθ<br />

π/4<br />

D ′<br />

√<br />

a cos 2θ<br />

0<br />

0<br />

<br />

= a2 1 − 1<br />

<br />

√ .<br />

2<br />

<br />

1 − 1<br />

<br />

√ .<br />

2<br />

D ′<br />

√<br />

a cos 2θ<br />

0<br />

cos 2θ dθ<br />

[sen 2θ]π/4<br />

0<br />

ρ 2 dρ<br />

= a3<br />

3 0<br />

cos θ(cos 2θ) 3/2 dθ = a3<br />

3<br />

(cambio u := sen θ)<br />

= a3<br />

3<br />

1/ √ 2<br />

0<br />

(1 − 2u 2 ) 3/2 du<br />

<br />

cambio u = 1 √ 2 sen t<br />

π/2<br />

ρdρdθ<br />

ρdρ<br />

= a2<br />

4 .<br />

π/4<br />

= a3<br />

(1 − sen<br />

3 0<br />

2 t) 3/2 1 √ cos t dt<br />

2<br />

√<br />

2<br />

=<br />

6 a3<br />

π/2<br />

cos<br />

0<br />

4 √<br />

2<br />

t dt =<br />

12 a3 <br />

1 5<br />

B ,<br />

2 2<br />

√<br />

2<br />

=<br />

12 a3<br />

<br />

1 5<br />

Γ Γ √<br />

2 2 2<br />

=<br />

Γ(3) 12 a3<br />

3 1<br />

2 2 Γ<br />

<br />

1<br />

2<br />

2<br />

= πa3√ 2<br />

32<br />

.<br />

0<br />

cosθ(cos 2 θ − sen 2 θ) 3/2 dθ<br />

2


Por tanto,<br />

xD = Ix<br />

Área = πa√2 8<br />

Análogamente, para calcular yD, necesitamos la integral:<br />

Ahora bien,<br />

luego<br />

Iy : =<br />

=<br />

<br />

D<br />

π/4<br />

0<br />

<br />

y dxdy =<br />

sen θ dθ<br />

π/4<br />

D ′<br />

√<br />

a cos 2θ<br />

0<br />

ρ 2 sen θ dρdθ<br />

ρ 2 dρ<br />

= a3<br />

3 0<br />

sen θ(cos 2θ) 3/2 dθ = a3<br />

3<br />

(cambio u := cos θ)<br />

1/ √ 2<br />

= a3<br />

−(2u<br />

3 1<br />

2 − 1) 3/2 du<br />

<br />

cambio t := arg ch <br />

u √ 2 <br />

= a3<br />

3<br />

0<br />

= a3<br />

3 √ 2<br />

arg ch √ 2<br />

√<br />

arg ch 2<br />

0<br />

− 1<br />

√ 2 sh 4 t dt<br />

sh 4 t dt .<br />

sh 4 t = 1<br />

16 (et − e −t ) 4<br />

π/4<br />

= 1<br />

16 (e4t − 4e 2t + 6 − 4e −2t + e −4t )<br />

= 1<br />

(2 ch 4t − 8 ch 2t + 6) ,<br />

16<br />

Iy = a3<br />

48 √ √<br />

arg ch 2<br />

(2 ch 4t − 8 ch 2t + 6)dt<br />

2 0<br />

= a3<br />

48 √ 2<br />

= a3<br />

48 √ 2<br />

Por otra parte, si w = arg chv, se tiene:<br />

Como<br />

<br />

sh 4t<br />

− 4 sh 2t + 6t<br />

2<br />

0<br />

.<br />

sen θ(cos 2 θ − sen 2 θ) 3/2 dθ<br />

arg ch √ 2<br />

0<br />

<br />

6 √ 2 − 8 √ 2 + 6 arg ch √ 2 <br />

v = chw = 1<br />

2 (ew + e −w )<br />

⇒ 2ve w = e 2w + 1<br />

⇒ e 2w − 2ve w + 1 = 0<br />

⇒ e w = v ± √ v 2 − 1<br />

⇒ w = log <br />

v ± √ v 2 − 1 <br />

log <br />

v + √ v 2 − 1 <br />

y log <br />

v − √ v 2 − 1 <br />

.<br />

.


tienen signos opuestos, y la función arg ch toma solo valores no negativos, se tiene<br />

arg ch √ 2 = log √ <br />

2 + 1 .<br />

Por tanto,<br />

de donde<br />

Iy = a3<br />

= a 3<br />

yD = Iy<br />

Área<br />

48 √ 2<br />

√ 2<br />

16<br />

= a<br />

12<br />

<br />

2 √ 2 + 6 log <br />

1 + √ 2 <br />

<br />

log 1 + √ 2 <br />

− 1<br />

<br />

24<br />

<br />

3 √ 2 log <br />

1 + √ 2 <br />

− 2 <br />

,<br />

.


AMPLIACIÓN DE CÁLCULO (Curso 2005/2006) Convocatoria de septiembre 12.09.06<br />

PROBLEMA 1 (4 puntos)<br />

Se consideran el campo escalar u(x, y, z) = (9x 2 + 4y 2 + z 2 ) 1/2 y los campos vectoriales:<br />

donde P (x, y, z) =<br />

F(x, y, z) =<br />

1<br />

u(x, y, z)<br />

r<br />

u(x, y, z) 3 y G(x, y, z) = (P (x, y, z) , Q(x, y, z) , 0) ,<br />

y z<br />

(9x2 + 4y2 , Q(x, y, z) = −<br />

)<br />

1<br />

u(x, y, z)<br />

x z<br />

(9x 2 + 4y 2 )<br />

1) Calcúlese el rotacional del campo G(x, y, z) sabiendo que se cumple:<br />

∂Q(x, y, z)<br />

∂x<br />

−<br />

∂P (x, y, z)<br />

∂y<br />

=<br />

z<br />

.<br />

u(x, y, z) 3<br />

y r = (x, y, z).<br />

¿Es G(x, y, z) un potencial vector de F(x, y, z)? En caso afirmativo, indíquese el dominio de<br />

R 3 en el que se da esta propiedad. Además, obténgase razonadamente un potencial vector del<br />

campo F(x, y, z) cuya tercera componente no sea nula. (1 punto.)<br />

2) Considérese la curva Γ que resulta de la intersección de la superficie elíptica de ecuación cartesiana<br />

9x 2 +4y 2 +z 2 = 2 con el plano z = 1, orientada de forma que su proyección sobre el plano<br />

z = 0 se recorre en sentido positivo. Sea Σ la parte de la superficie elíptica anterior que tiene<br />

por borde orientado la curva Γ y está situada en el semiespacio z ≥ 1. Utilícese la definición<br />

de flujo y de circulación para calcular el flujo de F(x, y, z) a través de Σ y la circulación de<br />

G(x, y, z) a lo largo de la curva Γ.<br />

(2 puntos.)<br />

3) Enúnciese con toda precisión el teorema de Stokes. Decídase razonadamente si los resultados<br />

obtenidos en el apartado anterior contradicen o no lo establecido en este teorema. (1 punto.)<br />

Respuesta:<br />

1) El rotacional del campo G(x, y, z) es, por definición:<br />

rot G :=<br />

Teniendo en cuenta que:<br />

se tiene:<br />

−<br />

∂Q(x, y, z)<br />

∂z<br />

∂P (x, y, z)<br />

∂z<br />

<br />

=<br />

−<br />

=<br />

∂Q(x, y, z)<br />

∂z<br />

, ∂P (x, y, z)<br />

∂z<br />

, ∂Q(x, y, z)<br />

∂x<br />

∂u<br />

∂z = z (9x2 + 4y 2 + z 2 ) −1/2 = z<br />

u ,<br />

x<br />

(9x 2 + 4y 2 )<br />

y<br />

(9x 2 + 4y 2 )<br />

∂<br />

∂z<br />

∂<br />

∂z<br />

<br />

z<br />

u<br />

<br />

z<br />

u<br />

=<br />

=<br />

x<br />

(9x 2 + 4y 2 )<br />

y<br />

(9x 2 + 4y 2 )<br />

1<br />

u<br />

−<br />

1<br />

u<br />

<br />

∂P (x, y, z)<br />

.<br />

∂y<br />

− z<br />

u 2<br />

− z<br />

u 2<br />

<br />

∂u<br />

∂z<br />

<br />

∂u<br />

∂z<br />

= x<br />

u 3<br />

= y<br />

.<br />

u3 Como se indica en el enunciado, la tercera componente de rot G es z/u 3 y, por tanto, se da la<br />

igualdad rot G =<br />

r<br />

u(x, y, z) 3 = F; es decir, G es un potencial vector de F en todo conjunto de R3 en<br />

el que estén definidas las derivadas parciales ∂P/∂y, ∂P/∂z, ∂Q/∂x y ∂Q/∂z. Esto ocurre en todo


R 3 menos el eje z (nótese que para x = y = 0 el denominador de las dos primeras componentes de<br />

G se anula). Así pues, G es un potencial vector de F en el conjunto<br />

C = R 3 \ <br />

(x, y, z) ∈ R 3 : x = y = 0 <br />

.<br />

Una forma de razonar para obtener a partir de G otro potencial vector de F, consiste en tener<br />

en cuenta que el rotacional de un gradiente (cuando está definido) es siempre nulo y, por tanto,<br />

basta considerar un campo escalar V (x, y, z) de clase C 2 en R 3 para que G + gradV sea también un<br />

potencial vector de F. La tercera componente de este nuevo potencial vector será ∂V/∂z, por tanto<br />

habrá que elegir el campo escalar V (x, y, z) de forma que ∂V/∂z = 0. Un ejemplo sencillo puede ser<br />

V (x, y, z) = z, en cuyo caso se obtiene el potencial vector de F dado por (P (x, y, z), Q(x, y, z), 1),<br />

cuya tercera componente no es nula.<br />

2) Circulación de G sobre Γ: La curva Γ es una elipse situada en el plano z = 1 que tiene por<br />

ecuación cartesiana<br />

9x 2 + 4y 2 = 1 , z = 1 ,<br />

con lo que una de sus parametrizaciones es r(θ) = (x(θ), y(θ), z(θ)), con:<br />

x(θ) = 1<br />

1<br />

cos θ , y(θ) = sen θ , z(θ) = 1 ,<br />

3 2<br />

y θ ∈ [0, 2π] para que su proyección sobre el plano z = 0 se recorra en sentido positivo.<br />

El valor de campo G sobre Γ es: G |Γ = 1<br />

pedida es:<br />

√ (y, −x, 0) con lo que, por definición, la circulación<br />

2<br />

<br />

Γ<br />

Gdr =<br />

<br />

Γ<br />

G |Γ dr = 1<br />

√ 2<br />

2π<br />

0<br />

⎛<br />

−<br />

1<br />

⎜<br />

sen θ , −1 cos θ , 0 · ⎝<br />

2 3 1<br />

⎞<br />

sen θ 3<br />

1 ⎟<br />

cos θ 2 ⎠ dθ = −<br />

0<br />

π<br />

3 √ 2 .<br />

Flujo de F a través de Σ: Considerando como parámetro el ángulo de las coordenadas elípticas y<br />

la coordenada z, una parametrización de la superficie considerada es<br />

r(θ, v) = (x(θ, v), y(θ, v), z(θ, v)) ,<br />

con:<br />

√<br />

2 − v2 x(θ, v) = cos θ ,<br />

3<br />

√<br />

2 − v2 y(θ, v) = sen θ ,<br />

2<br />

z(θ, v) = v , θ ∈ [0, 2π] , v ∈ [1, √ 2] .<br />

La cara, Σ + , de la superficie orientada que hay que considerar es la que tiene por vector normal<br />

∂r(θ, v) ∂r(θ, v)<br />

×<br />

∂θ ∂v =<br />

√ √<br />

2 − v2 2 − v2 cos θ , sen θ ,<br />

2<br />

3<br />

v<br />

<br />

.<br />

6<br />

Por tanto, el flujo pedido es, por definición:<br />

<br />

Σ +<br />

F · ds =<br />

=<br />

1<br />

2 √ 2<br />

1<br />

2 √ 2<br />

2π<br />

0<br />

2π<br />

0<br />

√ 2<br />

1<br />

√ 2<br />

1<br />

<br />

∂r(θ, v) ∂r(θ, v)<br />

r(θ, v) · × dv dθ<br />

∂θ ∂v<br />

1 π<br />

dv dθ =<br />

3 3 √ √ <br />

2 − 1 =<br />

2<br />

π<br />

√ <br />

2<br />

1 − .<br />

3 2<br />

Nota: Aunque no es lo que se pide en el enunciado, el flujo anterior puede calcularse aplicando<br />

adecuadamente el teorema de Gauss, ya que el campo F es solenoidal en R 3 \ {(0, 0, 0)}.


3) Un enunciado del teorema de Stokes puede ser el siguiente:<br />

Sea Σ una superficie regular y orientable cuyo borde es la curva Γ. Sea H un campo vectorial de<br />

clase C 1 en un abierto A ⊆ R 3 que contiene a Σ y a su borde Γ. Entonces:<br />

<br />

Σ<br />

<br />

rot H · dσ =<br />

Γ<br />

H · dr ,<br />

donde la cara de Σ y la orientación de Γ que se consideran son las que resultan de aplicar la “regla<br />

del sacacorchos”.<br />

Para decidir si los resultados obtenidos en el apartado anterior contradicen o no este teorema hay<br />

que comprobar, en primer lugar, que los campos F y G verifican sus hipótesis. Pero toda superficie<br />

cuyo borde sea la curva Γ contiene al menos un punto que pertenece al eje z en el que el campo G<br />

no es de clase C 1 . Así pues, el teorema de Stokes no se puede aplicar en este caso y el hecho de que,<br />

según muestran los cálculos anteriores,<br />

no lo contradice.<br />

<br />

Σ +<br />

F =<br />

<br />

Σ +<br />

rotG =<br />

<br />

Γ<br />

Gdr ,


AMPLIACIÓN DE CÁLCULO (Curso 2005/2006) Convocatoria de septiembre 12.09.06<br />

PROBLEMA 2 (3 puntos): Este ejercicio consta de dos apartados independientes.<br />

1) Calcúlese el volumen del sólido interior a la esfera unidad centrada en el origen, e interior al<br />

cilindro x 2 + y 2 = 1/4. (1 punto)<br />

2) Sea el recinto D = {(x,y), 0 < x < 1, 0 < y < 1, x 2 + y 2 > 1} y C su curva frontera, orientada<br />

positivamente. Considerando el campo vectorial<br />

relaciónense las integrales<br />

<br />

D<br />

<br />

x<br />

F(x,y) =<br />

1 + x2 <br />

+ y2, 0 ,<br />

xy<br />

(1 + x 2 + y 2 ) 2dxdy,<br />

<br />

C<br />

F · dr<br />

y calcúlese sólo una de ellas. (2 puntos)<br />

Respuesta: Se entregará esta hoja y, a lo sumo, una adicional.<br />

1) El volumen pedido puede calcularse de muchas formas distintas, por ejemplo: como volumen de<br />

sólido de revolución, o bien como suma de volumen del cilindro más volumen de dos casquetes<br />

esféricos, o bien pasando a coordenadas esféricas, o cilíndricas, etc. Aquí incluimos la siguiente<br />

solución alternativa, como volumen proyectable sobre la región plana D : x 2 + y 2 ≤ 1/4 , ya<br />

que la sólido viene definido por<br />

Ω =<br />

Su volumen V es<br />

<br />

V =<br />

<br />

(x,y,z) ∈ R 3 <br />

: (x,y) ∈ D, − 1 − x2 − y2 <br />

≤ z ≤ 1 − x2 − y2 <br />

.<br />

Ω<br />

<br />

dx dy dz =<br />

y, por ejemplo, pasando a polares en D,<br />

⎛<br />

<br />

⎝<br />

D<br />

√ 1−x2−y2 √<br />

− 1−x2−y2 2π 1/2 <br />

V = 2 ρ 1 − ρ<br />

0 0<br />

2dρ dθ = 4π<br />

<br />

−<br />

3<br />

<br />

1 − ρ 23<br />

⎞<br />

<br />

dz⎠<br />

dx dy = 2<br />

ρ=1/2<br />

2<br />

ρ=0<br />

= 4π<br />

3<br />

<br />

D<br />

<br />

1 − x 2 − y 2 dxdy<br />

1 − 3√ 3<br />

8<br />

<br />

=<br />

<br />

8 − 3 √ 3 <br />

π<br />

.<br />

6


2) El recinto D del enunciado aparece en la siguiente figura, y se observa que su borde C es una<br />

curva de Jordan que consta claramente de tres arcos C1, C2, C3:<br />

1<br />

0<br />

y<br />

C2<br />

C3<br />

D<br />

Podemos aplicar el Teorema de Green al campo F que es de clase C 1 en el dominio D, con lo<br />

cual se relaciona su circulación a lo largo de C con una integral doble sobre D:<br />

<br />

C<br />

1<br />

<br />

<br />

∂Q ∂P<br />

F · dr = (P,Q) ·dr = − dx dy.<br />

C<br />

D ∂x ∂y<br />

Además, en este caso Q es nula y P(x,y) = x<br />

1+x 2 +y 2 así que<br />

<br />

C<br />

C1<br />

<br />

F · dr = −<br />

D<br />

∂P<br />

<br />

dx dy =<br />

∂y D<br />

x<br />

2xy<br />

(1 + x2 + y2 ) 2dx dy<br />

con lo que se deduce que la integral curvilínea es dos veces la integral doble del enunciado.<br />

Ya que ambas están relacionadas como acabamos de decir, calculamos una cualquiera de ellas,<br />

por ejemplo, la integral curvilínea. La curva C orientada positivamente consta de tres partes:<br />

un segmento vertical C1, otro horizontal C2 y un arco de circunferencia C3; por ello, hay que<br />

calcular sobre cada una de esas curvas la integral de línea<br />

En el segmento vertical dx = 0, luego <br />

<br />

C<br />

<br />

<br />

F · dr = Pdx + 0dy =<br />

C<br />

C<br />

C1<br />

Pdx = 0.<br />

x<br />

1 + x 2 + y 2dx<br />

Por otro lado, el segmento horizontal está formado por los puntos (x, 1) con x ∈ [0, 1], pero hay<br />

que cambiarlo de orientación, así que queda:<br />

<br />

C2<br />

1<br />

Pdx = −<br />

0<br />

x <br />

2 + x2dx = −1 log<br />

2<br />

<br />

2 + x 2x=1 1<br />

= (log (2) − log (3)).<br />

x=0 2<br />

Por último el arco de circunferencia tiene la orientación opuesta a la de (x(t),y (t)) = (cost, sen t)<br />

con t ∈ [0,π/2], luego (x ′ (t),y ′ (t)) = (−sen t, cos t) y entonces<br />

<br />

C3<br />

π/2 x (t)<br />

Pdx = −<br />

0 2 x′ π/2<br />

(t)dt =<br />

0<br />

Finalmente, la integral curvilínea pedida es<br />

<br />

C<br />

<br />

cos t<br />

sen tdt = −<br />

2 1<br />

4 cos2 t=π/2<br />

t =<br />

t=0<br />

1<br />

4 .<br />

<br />

F · dr = Pdx + Pdx + Pdx =<br />

C1 C2 C3<br />

1 1<br />

+ (log (2) − log (3))<br />

4 2<br />

mientras que la integral doble es la mitad:<br />

<br />

D<br />

xy<br />

(1 + x2 + y2 1 1<br />

) 2dx dy = + (log (2) − log (3)).<br />

8 4


AMPLIACIÓN DE CÁLCULO (Curso 2005/2006) Convocatoria de septiembre 12.09.06<br />

PROBLEMA 3 (3 puntos)<br />

Este ejercicio consta de tres apartados independientes; cada uno de ellos vale 1 punto.<br />

1) Estúdiese la convergencia de la integral impropia<br />

∞<br />

0<br />

e −x<br />

√ x 3 − x 2 − x + 1 dx.<br />

2) Determinar los valores reales de m y n para los que la integral<br />

∞ (x<br />

1<br />

3 − 1) m<br />

x3n+1 dx<br />

converge y calcular su valor para los casos en que sean enteros.<br />

3) Calcular el valor de t para el cual la función M(t) =<br />

valor mínimo en el intervalo [1, 10].<br />

Solución:<br />

1)<br />

∞ e<br />

I =<br />

−x<br />

√ dx<br />

x3 − x2 − x + 1<br />

0<br />

∞ e<br />

t<br />

−xt t e<br />

dx + 3<br />

x 0<br />

−x2<br />

dx alcanza su<br />

x + 1<br />

En primer lugar, y debido a la presencia de la raíz cuadrada, comprobaremos que la función subradical<br />

es no negativa en [0, ∞) y que por ello la función subintegral está bien definida. Para ello<br />

factorizaremos el polinomio x 3 − x 2 − x + 1. Utilizando Ruffini y tanteando la raíz x = 1 se obtiene<br />

de inmediato que x 3 − x 2 − x + 1 = (x − 1) 2 (x + 1). Se observa que esta función es no negativa en<br />

[0, ∞) por lo que la raíz cuadrada está bien definida. Por otro lado, y puesto que x = −1 está fuera<br />

del intervalo de integración, la función subintegral<br />

f(x) =<br />

e −x<br />

√ x 3 − x 2 − x + 1 =<br />

e −x<br />

|x − 1| (x + 1) 1/2<br />

está definida en todo [0, ∞) salvo en el punto x = 1, en cuyo entorno la función no está acotada (pues<br />

límx→1 + = ∞ y límx→1− = ∞). Por consiguiente, la integral es impropia por dos razones: la presencia<br />

de un intervalo de integración no acotado y el hecho de que f no está acotada en el entorno de x = 1.<br />

Obsérvese que f es siempre no negativa, con lo que la convergencia de la integral es equivalente a la<br />

convergencia absoluta y podemos aplicar los criterios de convergencia para integrales impropias en<br />

las que la función subintegral no cambia de signo. Por definición, la integral será convergente si y<br />

sólo si las tres integrales<br />

1<br />

a<br />

∞<br />

α = f(x)dx , β = f(x)dx , γ = f(x)dx,<br />

0<br />

1<br />

cada una de las cuales es impropia sólo por un motivo, son convergentes, donde a es cualquier<br />

número del intervalo (1, ∞).<br />

1<br />

a


y ∞<br />

1<br />

La integral γ converge absolutamente pues (recuérdese que f ≥ 0)<br />

lím<br />

x→∞<br />

e −x<br />

|x−1|(x+1) 1/2<br />

1<br />

x 2<br />

= 0<br />

dx<br />

x2 es convergente ( ∞ dx<br />

1 xp converge si y sólo si p > 1). En cuanto a α<br />

lím<br />

x→1 −<br />

e −x<br />

|x−1|(x+1) 1/2<br />

1<br />

1−x<br />

= lím<br />

x→1 −<br />

que es finito y no nulo, por lo que 1<br />

0 f(x)dx y 1<br />

0<br />

integral es divergente ( 1<br />

0<br />

dx<br />

1−x<br />

e −x<br />

(1−x)(x+1) 1/2<br />

1<br />

1−x<br />

= e−1<br />

2 1/2<br />

tienen el mismo carácter. Como esta segunda<br />

dx<br />

(1−x) p converge si y sólo si p < 1) se sigue que α es divergente y por<br />

consiguiente, e independientemente del carácter de β, I es divergente. Aunque ya no es necesario<br />

analizar el carácter de β, ésta resulta también divergente, pues<br />

que es finito y no nulo, y a<br />

1<br />

2) Sea I(m, n) := ∞<br />

1<br />

lím<br />

x→1 +<br />

e −x<br />

|x−1|(x+1) 1/2<br />

1<br />

x−1<br />

= lím<br />

x→1 +<br />

dx<br />

x−1 es divergente ( a<br />

1<br />

dx<br />

x−1<br />

e −x<br />

(x−1)(x+1) 1/2<br />

1<br />

x−1<br />

= e−1<br />

2 1/2<br />

converge si y sólo si p < 1)<br />

(x 3 −1) m<br />

x 3n+1 dx. Teniendo en cuenta la forma de la función subintegral y el hecho<br />

de que debemos calcular de forma explícita el valor de I(m, n), parece razonable intentar relacionar<br />

I con la función beta, β(p, q) = 1<br />

0 yp−1 (1 − y) q−1 dy. Para transformar el intervalo (1, ∞) (para la x)<br />

en el intervalo (0, 1) (para la y) parece razonable realizar el cambio x = φ(y) = 1/y (que obviamente<br />

es un difeomorfismo al ser φ biyectivo y tanto φ como φ −1 de clase uno). Se tiene entonces<br />

∞ (x<br />

I(m, n) :=<br />

3 − 1) m<br />

<br />

1<br />

0<br />

y<br />

dx =<br />

3 − 1 m 3n+1 1<br />

x 3n+1<br />

1<br />

1<br />

y<br />

<br />

− 1<br />

y2 <br />

dy =<br />

1<br />

0<br />

<br />

1 − y 3 m<br />

y 3n−3m−1 dy<br />

que no es una β debido al término y 3 dentro del paréntesis. Para subsanar esto se puede hacer el<br />

cambio y = η(u) = u 1/3 , es decir, u = η −1 (y) = y 3 que es un difeomorfismo de (0, 1) a (0, 1). Entonces<br />

I(m, n) =<br />

1<br />

= 1<br />

3<br />

0<br />

1<br />

<br />

1 − y 3m y 3n−3m−1 1<br />

dy =<br />

0<br />

0<br />

(1 − u) m u 3n−3m−1<br />

3<br />

(1 − u) m u n−m−1 du = 1<br />

β(m + 1, n − m)<br />

3<br />

<br />

1 2<br />

u− 3 du =<br />

3<br />

Por supuesto, el procedimiento anterior es totalmente equivalente a haber realizado un único<br />

cambio correspondiente a la composición de los dos anteriores, es decir, x = α(u) = u −1/3 , u = x −3 .<br />

Utilizando las propiedades de la función β se obtiene que I(n, m) converge si y sólo si m > −1 y<br />

n > m. Además, en el caso de que m y n cumplan las condiciones anteriores y sean enteros (es decir,<br />

m = 0, 1, 2, ... y n = m + 1, m + 2, ... se puede escribir<br />

I(m, n) = 1<br />

1<br />

β(m + 1, n − m) =<br />

3 3<br />

= 1 m!(n − m − 1)!<br />

3 n!<br />

Γ(m + 1)Γ(n − m)<br />

Γ(n + 1)<br />

3) Para calcular el mínimo de M derivaremos en la expresión de M. Para ello, en primer lugar<br />

comprobaremos que la función M está definida en [1, 10] y que es derivable en dicho intervalo. Para<br />

simplificar los razonamientos podemos expresar ∞ e<br />

t<br />

−xt<br />

x dx = 10 e<br />

t<br />

−xt<br />

x dx + ∞<br />

10 e−xt dx con el fin de<br />

x<br />

separar la presencia de la variable t en los límites de integración por un lado y la la presencia<br />

2<br />

=


de un intervalo de integración no acotado por otro. Obsérvese que el partir la integral utilizando<br />

x = 10 es arbitrario, se podrían haber utilizado otros valores. Ahora denotemos α(t) := 10<br />

t<br />

β(t) := ∞<br />

10 e−xt<br />

0 e−x2 dx con lo que M(t) = α(t) + β(t) + 3γ(t).<br />

x dx y γ(t) := t<br />

x+1<br />

e −xt<br />

x dx,<br />

Para cada t ∈ [1, 10], γ(t) es una integral en sentido propio, pues la función subintegral está aco-<br />

tada en [0, 10]. Puesto que e−x2 es continua en [0, 10] y toda función continua es integrable, γ(t)<br />

x+1<br />

está definida en [1, 10]. Además, usando el teorema fundamental del cálculo integral, γ(t) es deriva-<br />

ble con continuidad en [1, 10] y su derivada vale e−t2<br />

t+1 .<br />

Para cada t ∈ [1, 10], α(t) es una integral en sentido propio, pues f(x, t) := e−xt<br />

está acotada<br />

x<br />

en Q := [1, 10] × [1, 10]. Como f es continua en Q la integral existe y por ello α está bien definida.<br />

existe y es de clase uno en Q se sigue que α es derivable con continuidad en [1, 10]<br />

Además como ∂f<br />

∂t<br />

y α ′ (t) = − e−t2<br />

t + 10 e−xt<br />

e−t2<br />

t (−x) dx = − x t − 10<br />

t e−xtdx. Por último, para cada t ∈ [1, 10], β(t) es una integral impropia al ser el intervalo de integración<br />

no acotado. Al ser f(x, t) acotada en [10, ∞) × [1, 10] no hay más razones por las que la integral<br />

sea impropia. Demostremos que para todo t ∈ [1, 10] la integral impropia es convergente. En efecto,<br />

<br />

para todo t ∈ [1, 10] y todo x ∈ [10, ∞) se cumple e−xt<br />

<br />

<br />

≤ x<br />

e−x<br />

x . Como ∞<br />

10 e−x dx es convergente, por<br />

x<br />

el criterio de comparación β(t) converge absolutamente para todo t ∈ [1, 10] (nótese que de hecho<br />

con el razonamiento anterior se ha demostrado, utilizando el criterio de Weierstrass, la convergencia<br />

uniforme de la integral para t ∈ [1, 10]) con lo que β está bien definida. Para demostrar la derivabilidad<br />

de β utilizamos en primer lugar que (a) f es continua en [10, ∞) × [1, 10] y ∂f<br />

existe y es continua<br />

∂t<br />

en dicho conjunto. En segundo lugar, demostraremos (b) que la integral ∞ ∂f<br />

10 ∂t dx = − ∞<br />

10 e−xtdx converge uniformemente para t ∈ [1, 10]. Para ello emplearemos el criterio de Weierstrass: para todo<br />

t ∈ [1, 10] y todo x ∈ [10, ∞) se cumple |e−xt | ≤ e−x y además ∞<br />

10 e−xdx es convergente, de donde<br />

se sigue la convergencia uniforme de la integral. De (a) y (b) se deduce que β es derivable con<br />

continuidad en [1, 10] y que β ′ (t) = ∞ ∂f<br />

10 ∂t dx = − ∞<br />

10 e−xtdx. Por lo tanto M es derivable con continuidad en [1, 10] y<br />

M ′ (t) = α ′ (t) + β ′ (t) + 3γ ′ (t) = − e−t2<br />

t −<br />

10<br />

e<br />

t<br />

−xt ∞<br />

dx − e<br />

10<br />

−xt dx + 3 e−t2<br />

t + 1 =<br />

= − e−t2<br />

t −<br />

∞<br />

e<br />

t<br />

−xt dx + 3 e−t2<br />

e−xt<br />

= −e−t2 − |<br />

t + 1 t t<br />

x=t<br />

x=∞ +3 e−t2<br />

t + 1 =<br />

= e −t2<br />

<br />

− 2<br />

<br />

3 t − 2<br />

−t2<br />

+ = e<br />

t t + 1 t(t + 1)<br />

donde se ha utilizado que al ser t positivo, límx→∞ e −xt = 0.<br />

Se observa que M ′ (t) sólo tiene un cero, t = 2, en el intervalo [1, 10]. Como M ′ (t) pasa de negativa<br />

a positiva al atravesar t = 2 podemos afirmar que t = 2 es un punto de mínimo relativo de M. Por<br />

tanto, los únicos candidatos a punto de mínimo global en [1, 10] son t = 2 y los extremos del intervalo.<br />

Como M ′ (t) < 0 en [1, 2) y M ′ (t) > 0 en (2, 10] podemos afirmar que t = 2 es el punto de mínimo<br />

global que se pide.<br />

3


AMPLIACIÓN DE CÁLCULO (Curso 2006/2007) Convocatoria de febrero 06.02.07<br />

NOMBRE . . . . . . . . . . . . . . . . . . . . . . . . . APELLIDOS . . . . . . . . . . . . . . . . . . . . . . . . . . . . . . . . . . . . . . . . . . .<br />

PROBLEMA 1 (4 puntos): Se consideran las integrales impropias<br />

Se pide:<br />

∞<br />

G(a) = e −x ∞<br />

cos(ax) dx. F (a) =<br />

0<br />

Número de matrícula. . . . . . . . . . . . . . . . . . . . . . . .<br />

0<br />

−x sen(ax)<br />

e<br />

x<br />

1. Encontrar el conjunto de valores reales de a donde G converge uniformemente. Para dichos<br />

valores, calcular explícitamente G(a). (1’5 puntos)<br />

2. Enunciar y aplicar el teorema que permite concluir que F es una función derivable en su dominio<br />

de definición. (0’75 puntos)<br />

3. Calcular la integral F (a) para los valores de a donde converge. (0’75 puntos)<br />

4. Para b > 0, relacionar la integral<br />

∞<br />

H(b) =<br />

0<br />

−xb sen x<br />

e<br />

x<br />

con la función F . Sabiendo que H converge uniformemente en b ≥ 0, deducir que<br />

∞<br />

0<br />

sen x π<br />

dx =<br />

x 2 .<br />

dx<br />

dx,<br />

(1 punto)<br />

Solución: (Se entregará esta hoja, y a lo sumo, otra más.)<br />

1. El integrando de G(a) puede acotarse en valor absoluto por una función mayorante: |e −x cos ax| ≤<br />

e −x = M(x) que es independiente de a y cuya integral impropia es convergente:<br />

∞<br />

∞<br />

M(x) dx = e<br />

0<br />

0<br />

−x dx = 1 < ∞<br />

luego, por el criterio de Weierstrass podemos afirmar que G es uniformemente convergente en<br />

todo R.<br />

Por tanto, G(a) existe para todo a real; podemos calcular su valor o bien integrando por partes<br />

dos veces, o bien usando que G(a) es la parte real de la siguiente integral:<br />

∞<br />

e<br />

0<br />

−x e iax ∞<br />

dx = e<br />

0<br />

(ia−1)x dx = 1<br />

<br />

lím<br />

ia − 1 x→∞ e(ia−1)x <br />

− 1 = 1 1 + ia<br />

=<br />

1 − ia 1 + a2 <br />

<br />

donde se ha utilizado que límx→∞ e (ia−1)x = límx→∞ |eiax | e−x = límx→∞ e−x = 0. Por tanto,<br />

su parte real es<br />

∞<br />

G(a) =<br />

0<br />

e −x ∞<br />

cos (ax) dx = Re e<br />

0<br />

−x e iax dx = Re<br />

<br />

1 + ia<br />

1 + a 2<br />

= 1<br />

1 + a 2<br />

∀a ∈ R.


2. Enunciemos el teorema pedido:<br />

Teorema: Se considera un intervalo cerrado I de la recta real, y la integral impropia<br />

Si se verifica:<br />

F (a0) converge para algún valor a0 ∈ I,<br />

∂f<br />

∂a<br />

es continua en I × [0, ∞)<br />

la integral impropia<br />

converge uniformemente en I,<br />

∞<br />

F (a) = f(a, x)dx.<br />

0<br />

∞<br />

entonces F es derivable en I, y su derivada es<br />

F ′ ∞<br />

(a) =<br />

0<br />

0<br />

∂f<br />

(a, x)dx<br />

∂a<br />

∂f<br />

(a, x)dx ∀a ∈ I.<br />

∂a<br />

Veamos que se verifican las tres hipótesis del teorema, tomando I = R:<br />

F converge en a0 = 0, pues obviamente F (0) = 0;<br />

<br />

−x sen ax e = e−x cos ax que es una función continua para (a, x) en R × [0, ∞)<br />

∂<br />

∂a<br />

x<br />

La integral impropia de la derivada es<br />

∞<br />

0<br />

<br />

∂f<br />

∞<br />

(a, x)dx = e<br />

∂a 0<br />

−x cos(ax)dx = G(a)<br />

que ya se ha visto que converge uniformemente en R.<br />

Por todo ello, el teorema garantiza que F es derivable en R, y además concluye que<br />

F ′ (a) = G(a) ∀a ∈ R.<br />

3. De la última igualdad, recuperamos F (a) = arc tg(a) + C y observando por la definición que<br />

F (0) = 0 = C finalmente se llega a la expresión<br />

F (a) = arc tg(a) ∀a ∈ R.<br />

4. Para b > 0, mediante un simple cambio de variable y = xb, se tiene que<br />

∞<br />

<br />

−xb sen x ∞<br />

−y sen(y/b)<br />

H(b) = e dx = e dy = F<br />

0 x 0 y<br />

<br />

1<br />

= arc tg<br />

b<br />

<br />

1<br />

.<br />

b<br />

Por otro lado, la convergencia uniforme de H en [0, ∞) garantiza que H es continua en dicho<br />

intervalo, y en particular en b = 0. Por ello, podemos concluir:<br />

∞<br />

0<br />

sen x<br />

x<br />

dx = H(0) = lím<br />

b→0<br />

b→0<br />

H(b) = lím arc tg<br />

+ +<br />

<br />

1<br />

=<br />

b<br />

π<br />

2 .


AMPLIACIÓN DE CÁLCULO (Curso 2006/2007) Convocatoria de febrero 6.02.07<br />

NOMBRE ......................... APELLIDOS ...........................................<br />

PROBLEMA 2 (3 puntos)<br />

Número de matrícula. .......................<br />

Dado a > 0, sea Γ la curva llamada cardioide, de ecuación polar ρ = a(1+cosθ). Sea D el dominio<br />

limitado por Γ. Se pide:<br />

(1) Hallar la longitud de Γ y el área de D. (1 punto)<br />

(2) Hallar el centroide del arco Γ. (1 punto)<br />

(3) Hallar el centroide del dominio D. (1 punto)<br />

Respuesta: Se entregará esta hoja y, a lo sumo, una adicional.<br />

Puede verse la curva y el dominio en la Figura 1.<br />

a<br />

y<br />

0<br />

D<br />

Γ<br />

Figura 1: Cardioide<br />

(1) Derivando en ρ(θ) = a(1 + cosθ), se obtiene<br />

Por tanto,<br />

y la longitud del arco es<br />

ρ ′ (θ) = −a sen θ .<br />

2a x<br />

ρ(θ) 2 + ρ ′ (θ) 2 = a 2 (1 + cosθ) 2 + a 2 sen 2 θ<br />

l(Γ) =<br />

=<br />

= 2a 2 (1 + cosθ)<br />

= 4a 2 2 θ<br />

cos<br />

2 ,<br />

2π <br />

ρ(θ)<br />

0<br />

2 + ρ ′ (θ) 2 dθ<br />

<br />

2π <br />

θ<br />

<br />

2a cos<br />

<br />

0 2<br />

dθ<br />

π<br />

= 4a cos<br />

0<br />

θ<br />

2 dθ<br />

<br />

= 8a sen θ<br />

π 2<br />

1<br />

0


= 8a .<br />

Calculamos a continuación el área de D:<br />

<br />

Área(D) = dxdy<br />

D<br />

= ρ dρdθ<br />

=<br />

D ′<br />

2π a(1+cos θ)<br />

dθ<br />

0 0<br />

2π<br />

= a2<br />

2<br />

= a2<br />

2<br />

0<br />

2π<br />

0<br />

= 3πa2<br />

2<br />

.<br />

ρ dρ<br />

(1 + cos θ) 2 dθ<br />

(1 + 2 cosθ + cos 2 θ) dθ<br />

(2) Calculamos las coordenadas (xΓ, yΓ) del centroide del arco Γ:<br />

Por simetría, yΓ = 0, luego el centroide es<br />

xΓ = 1<br />

<br />

xds<br />

l(Γ) Γ<br />

= 1<br />

2π <br />

ρ(θ) cosθ ρ(θ)<br />

8a 0<br />

2 + ρ ′ (θ) 2 dθ<br />

= 1<br />

<br />

2π<br />

<br />

θ<br />

<br />

a(1 + cosθ) cosθ 2a cos<br />

<br />

8a 0<br />

2<br />

dθ<br />

π<br />

3 θ<br />

= a cos cosθ dθ<br />

0 2<br />

<br />

<br />

π<br />

3 θ 2 θ θ<br />

= a cos cos − sen2 dθ<br />

0 2 2 2<br />

<br />

<br />

π<br />

5 θ θ θ<br />

= a cos − cos3 sen2 dθ t =<br />

0 2 2 2<br />

θ<br />

<br />

2<br />

π/2 <br />

= 2a cos<br />

0<br />

5 t − cos 3 t sen 2 t <br />

dt<br />

<br />

1 3<br />

= a B , 3 − B , 2<br />

2 2<br />

⎡ ⎤<br />

1 3<br />

⎢Γ<br />

Γ(3) Γ Γ(2)<br />

= a ⎢ 2<br />

⎣ −<br />

2 ⎥<br />

⎥<br />

7 7 ⎦<br />

Γ Γ<br />

2 2<br />

⎛<br />

⎜ 2<br />

= a ⎜<br />

⎝<br />

√ ⎞<br />

1 √<br />

π π<br />

5 3 1 √<br />

− 2 ⎟<br />

5 3 1 √ ⎠<br />

π π<br />

2 2 2 2 2 2<br />

= 4a<br />

5 .<br />

(xΓ, yΓ) =<br />

2<br />

4a<br />

5<br />

<br />

, 0 .


e<br />

(3) Para calcular las coordenadas (xD, yD) del centroide de D, necesitamos la integral:<br />

<br />

Ix =<br />

<br />

=<br />

=<br />

= a3<br />

3<br />

D<br />

xdxdy<br />

ρ 2 cosθ dρdθ<br />

D ′<br />

2π a(1+cos θ)<br />

cosθ dθ<br />

0<br />

0<br />

2π<br />

= a3<br />

3<br />

= a3<br />

3<br />

= a3<br />

3<br />

= 4a3<br />

3<br />

= 2a3<br />

3<br />

= 2a3<br />

3<br />

= 5πa3<br />

4<br />

ρ 2 dρ<br />

cos θ(1 + cosθ) 3 dθ<br />

0<br />

2π<br />

cos θ(1 + 3 cosθ + 3 cos<br />

0<br />

2 θ + cos 3 θ) dθ<br />

2π<br />

(cosθ + 3 cos<br />

0<br />

2 θ + 3 cos 3 θ + cos 4 θ) dθ<br />

2π<br />

(3 cos<br />

0<br />

2 θ + cos 4 θ) dθ (los demás términos tienen integral nula)<br />

π/2<br />

(3 cos<br />

0<br />

2 θ + cos 4 θ) dθ<br />

<br />

1 3 1 5<br />

3 B , + B ,<br />

2 2 2 2<br />

⎡<br />

⎢<br />

⎣3 <br />

1 1<br />

Γ<br />

2 2 Γ<br />

⎤<br />

1 1 5<br />

Γ Γ<br />

2<br />

+<br />

2 2 ⎥<br />

Γ(2) Γ(3)<br />

⎦<br />

.<br />

Las coordenadas (xD, yD) del centroide son:<br />

xD =<br />

Ix 2<br />

=<br />

Área(D) 3πa2 5πa 3<br />

yD = 0 , por simetría.<br />

3<br />

4<br />

= 5a<br />

6


AMPLIACIÓN DE CÁLCULO (Curso 2006/2007) Convocatoria de febrero 6.02.07<br />

NOMBRE ......................... APELLIDOS ...........................................<br />

PROBLEMA 3 (3 puntos)<br />

Se consideran el casquete del elipsoide:<br />

x 2<br />

4<br />

+ y2<br />

4<br />

Número de matrícula........................<br />

+ z2<br />

16<br />

situado en el semiespacio z ≥ 0 y la superficie cónica formada por los segmentos que unen el punto<br />

(0, 0, −8) con el borde del casquete anterior. Sea F(x,y,z) el campo vectorial definido por<br />

<br />

F(x,y,z) := y U(x,y,z), −xU(x,y,z), z(x + y + z<br />

2 )<br />

<br />

<br />

2 (z + 8)<br />

donde U(x,y,z) :=<br />

16 − x2 − y 2<br />

3/2 . Se pide:<br />

1) Calcular el flujo del campo F(x,y,z) a través del casquete elipsoidal orientado de forma que la<br />

tercera componente de su vector normal sea positiva. (1 punto)<br />

2) Calcular el flujo del campo F(x,y,z) a través de la superficie cónica orientada de forma que la<br />

tercera componente de su vector normal sea negativa. (1 punto)<br />

3) Sean Ω ∈ R 3 el domino limitado por las dos superficies definidas anteriormente, G := (xg,yg,zg)<br />

el centroide de Ω y Σ + la cara exterior de su frontera. Relacionar el flujo del campo F(x,y,z)<br />

a través de Σ + con las coordenadas de G y calcularlas. (1 punto)<br />

Respuesta: Se entregará esta hoja y, a lo sumo, una adicional.<br />

En primer lugar, dado que a lo largo del problema hay que calcular flujos del campo vectorial<br />

F(x,y,z), conviene calcular su divergencia para saber qué posibilidades hay a la hora de aplicar el<br />

teorema de Gauss. Su expresión es:<br />

∇ · F(x,y,z) =<br />

∂ [y U(x,y,z)]<br />

∂x<br />

−<br />

∂ [xU(x,y,z)]<br />

∂y<br />

= 1,<br />

+ ∂ <br />

z <br />

x + y + z<br />

<br />

2<br />

= x + y + z .<br />

∂z<br />

Se trata pues de un campo vectorial que (salvo en el origen) no es solenoidal. Nótese que el campo<br />

está definido en R3 menos el conjunto <br />

(x,y,z) ∈ R3 : (z+8)2<br />

16 − x2 − y2 < 0 <br />

.<br />

1) En vista de ello, optamos por calcular el flujo pedido parametrizando la<br />

superficie, que se muestra en la figura, y aplicando la definición (pese a que,<br />

como se demuestra en la solución alternativa que se expone más abajo, esta<br />

no es la opción más sencilla).<br />

Sea E + la cara del casquete considerada. Debido a que su proyección sobre<br />

el plano XY es el disco de ecuación cartesiana: x2 + y2 ≤ 4, utilizamos como<br />

parámetros las coordenadas polares (r,θ) del plano. Entonces, una parametrización,<br />

Σ(r,θ) = (x(r,θ),y(r,θ),z(r,θ)), de E + viene dada por las siguientes<br />

ecuaciones paramétricas:<br />

x(r,θ) = r cos θ , y(r,θ) = r cos θ , z(r,θ) = 4<br />

<br />

1 − r2<br />

4<br />

, 0 ≤ r ≤ 2, 0 ≤ θ ≤ 2π ,


con lo que el vector normal, que debe tener su tercera componente positiva, es:<br />

n(r,θ) := Σ(r,θ)<br />

∂r<br />

× Σ(r,θ)<br />

∂θ =<br />

Por definición, la expresión del flujo pedido es:<br />

<br />

<br />

F(x,y,z) · dσ =<br />

donde<br />

y, por tanto:<br />

<br />

E +<br />

E +<br />

[0,2]×[0,2π]<br />

<br />

2 2r cosθ<br />

√<br />

4 − r2 , 2r2 sin θ<br />

√<br />

4 − r2 ,r<br />

<br />

.<br />

F(x(r,θ),y(r,θ),z(r,θ)) · n(r,θ)dr dθ ,<br />

F(x(r,θ),y(r,θ),z(r,θ)) · n(r,θ) = 2r <br />

4 − r 2 + r √ 4 − r 2 (cosθ + sinθ) <br />

2<br />

F(x,y,z) · dσ = 2π 2r (4 − r<br />

0<br />

2 2<br />

) + 2r<br />

0<br />

2√ 4 − r2 2π<br />

<br />

dr (cos θ + sin θ)dθ .<br />

0<br />

Finalmente, la integral en la variable θ es cero, con lo que resulta:<br />

<br />

E +<br />

2<br />

F(x,y,z) · dσ = 4π r (4 − r<br />

0<br />

2 ) = 16π .<br />

1) Alternativa. El casquete elipsoidal no es una superficie cerrada. Sin embargo, es posible<br />

cerrarla considerando otra superficie regular cualquiera que tenga el mismo borde que el casquete.<br />

Como vamos a ver a continuación, la elección del disco de ecuación cartesiana: x 2 + y 2 ≤ 4 situado<br />

en el plano XY , resulta especialmente apropiada en este caso. La aplicación del teorema de Gauss a<br />

la superficie cerrada formada por el disco y el casquete elipsoidal da lugar a la expresión:<br />

<br />

<br />

<br />

F(x,y,z) · dσ + F(x,y,z) · dσ = (x + y + z)dV ,<br />

E +<br />

C +<br />

donde E + es la cara del casquete orientada de forma que la tercera componente de su vector normal<br />

es positiva, C + representa la cara del disco tal que la tercera componente de su vector normal es<br />

negativa y D es el recinto acotado de R3 limitado por la superficie cerrada, que viene dado por:<br />

⎧<br />

⎨<br />

D =<br />

⎩ (x,y,z) ∈ R3 : x 2 + y 2 <br />

≤ 4, 0 ≤ z ≤ 4 1 − x2<br />

⎫<br />

y2 ⎬<br />

−<br />

4 4 ⎭ .<br />

Como ya se ha mencionado, la elección del disco para obtener una superficie cerrada resulta muy<br />

conveniente. Nótese que la tercera componente del campo sobre el disco es nula (en el disco es z = 0),<br />

mientras que el vector normal al disco tiene sus dos primeras componentes nulas; es decir, el campo<br />

sobre el disco y la normal al disco son perpendiculares, con lo que el flujo es nulo. Así pues:<br />

<br />

<br />

<br />

F(x,y,z) · dσ = 0 =⇒ F(x,y,z) · dσ = (x + y + z)dV .<br />

C +<br />

E +<br />

Por otra parte, la integral de volumen que aparece en el segundo miembro es, por definición, el<br />

resultado de multiplicar el volumen de D por la suma de las componentes de su centroide. Pero este<br />

dominio es simétrico con respecto a los planos XZ e Y Z y, por tanto, las dos primeras componentes<br />

de su centroide son nulas; es decir:<br />

<br />

xdV = y dV = 0,<br />

con lo cual se obtiene la igualdad:<br />

<br />

E +<br />

D<br />

D<br />

<br />

F(x,y,z) · dσ =<br />

D<br />

D<br />

z dV ,<br />

D


según la cual el flujo pedido es igual al producto del volumen de D por la tercera componente de<br />

su centroide. La integral de volumen se puede calcular de forma sencilla utilizando coordenadas<br />

cilíndricas: <br />

z dV = z r drdθ dz ,<br />

donde r es el jacobiano y D ′ =<br />

tanto, el flujo pedido es:<br />

<br />

D<br />

z dV =<br />

<br />

D ′<br />

⎧<br />

⎨<br />

z r drdθ dz =<br />

D<br />

D ′<br />

⎩ (r,θ,z) ∈ R3 : 0 ≤ r ≤ 2, 0 ≤ θ ≤ 2π , 0 ≤ z ≤ 4<br />

2π<br />

0<br />

dθ<br />

2<br />

0<br />

dr<br />

<br />

4 1− r2<br />

4<br />

0<br />

z r dz = 16π<br />

<br />

2<br />

0<br />

1 − r2<br />

4<br />

<br />

<br />

1 − r2<br />

4<br />

⎫<br />

⎬<br />

. Por<br />

⎭<br />

r dr = 16π .<br />

Además, aunque no se pide, el centroide de D tiene por coordenadas (0, 0, 3),<br />

ya que su volumen es<br />

la mitad del volumen del elipsoide de semiejes 2, 2 y 4, que es 32π<br />

3 .<br />

2) Unas ecuaciones paramétricas para la superficie cónica que se muestra<br />

en la figura se obtienen construyendo las ecuaciones de todas los segmentos<br />

que unen el punto (0, 0, −8) con el borde del casquete elipsoidal. El borde es la<br />

circunferencia x 2 +y 2 = 4 situada en el plano z = 0, con lo que unas ecuaciones<br />

paramétricas de esta curva son:<br />

x(θ) = 2 cos θ , y(θ) = 2 sen θ , z(θ) = 0, 0 ≤ θ ≤ 2π .<br />

La ecuación del haz de segmentos mencionado es, entonces:<br />

x<br />

x(θ)<br />

= y<br />

y(θ)<br />

= z + 8<br />

z(θ) + 8<br />

= λ , 0 ≤ λ ≤ 1,<br />

de donde se obtiene la parametrización Σ(θ,λ) = (x(θ,λ),y(θ,λ),z(θ,λ)) para la superficie cónica,<br />

cuyas ecuaciones son:<br />

x(θ,λ) = 2λ cos θ y(θ,λ) = 2λ sen θ z(θ,λ) = −8 + 8λ , 0 ≤ θ ≤ 2π , 0 ≤ λ ≤ 1,<br />

y de ellas se deduce fácilmente la ecuación cartesiana de esta superficie, que es: x 2 + y 2 (z + 8)2<br />

= .<br />

16<br />

Sea S + la cara de la superficie que se debe considerar, es decir aquella cuyo vector normal tiene<br />

su tercera componente negativa y que, por tanto, es:<br />

n(θ,λ) := Σ(θ,λ)<br />

∂θ<br />

× Σ(θ,λ)<br />

∂λ<br />

= (16λ cos θ, 16λ sen θ, −4λ).<br />

El flujo se calcula ahora aplicando su definición para lo que, en general, resulta conveniente ver el<br />

valor que el campo toma sobre la superficie. Esto es especialmente adecuado en este caso, puesto que<br />

la función U(x,y,z) definida en el enunciado se anula sobre la superficie cónica. Teniendo esto en<br />

cuenta, el valor del campo sobre ella es:<br />

con lo cual<br />

y el flujo pedido es:<br />

<br />

S +<br />

F(x(θ,λ),y(θ,λ),z(θ,λ)) := (0, 0, 16(λ − 1)[λ(cos θ + sen θ) + 2(λ − 1)]) ,<br />

F(x(θ,λ),y(θ,λ),z(θ,λ)) · n(θ,λ) = −64λ 2 (λ − 1)[cos θ + sen θ] − 128λ(λ − 1) 2<br />

1<br />

F(x,y,z)·dσ = − 64λ<br />

0<br />

2 2π<br />

1<br />

(λ − 1)dλ [cos θ + sen θ]dθ −256π λ(λ−1)<br />

0<br />

0<br />

2 dλ = − 64π<br />

3 .<br />

2


2) Alternativas. El teorema de Gauss se puede utilizar aquí de forma análoga a como se<br />

describe en la solución alternativa del primer apartado. A la hora de construir la superficie cerrada,<br />

hay dos posibilidades razonables que se pueden utilizar en este caso.<br />

Una de ellas consiste en cerrar la superficie cónica por el disco x 2 +y 2 ≤ 4, z = 0, en cuyo caso el<br />

flujo pedido es igual al resultado de multiplicar el volumen del cono limitado por ambas superficies<br />

por la tercera componente de su centroide. Dado que el volumen del cono es un tercio del área de su<br />

base por su altura, es decir: 32π,<br />

resulta que el centroide del cono es (0, 0, −2) (pero este resultado<br />

3<br />

no se pide).<br />

La otra es cerrar la superficie cónica utilizando el casquete elipsoidal. Esto último constituye el<br />

objetivo del tercer apartado de este problema.<br />

3) Dado que el centroide de un sólido de R 3 se expresa en términos de una<br />

integral de volumen, en este apartado se nos pide relacionar un flujo con una<br />

integral triple. Un resultado que, en caso de que se cumplan las hipótesis,<br />

permite relacionar estos dos tipos de integrales es el teorema de Gauss. Estas<br />

hipótesis se cumplen claramente en las condiciones establecidas en el enunciado,<br />

puesto que el campo vectorial es de clase C 1 en Ω y la superficie que<br />

encierra el sólido (ver la figura) es regular a trozos. Su aplicación da lugar a la<br />

relación:<br />

<br />

Σ +<br />

<br />

F(x,y,z) · dσ =<br />

Ω<br />

<br />

∇ · F(x,y,z) =<br />

Ω<br />

(x + y + z)dV .<br />

Puesto que Ω es simétrico con respecto a los planos XZ e Y Z, el mismo<br />

argumento utilizado en la solución alternativa al primer apartado nos permite<br />

concluir que las dos primeras componentes de su centroide son nulas; es decir:<br />

<br />

xdV = ydV = 0.<br />

Por tanto, <br />

Σ +<br />

Ω<br />

Ω<br />

<br />

F(x,y,z) · dσ =<br />

Ω<br />

z dV = zg VΩ ,<br />

donde VΩ es el volumen de Ω que se obtiene como la suma del volumen de la parte correspondiente<br />

al elipsoide más el volumen del cono, es decir: VΩ = 32π 32π 64π + = . Ahora bien, obviamente:<br />

3 3 3<br />

<br />

<br />

<br />

F(x,y,z) · dσ = F(x,y,z) · dσ + F(x,y,z) · dσ ,<br />

Σ +<br />

E +<br />

donde E + es la cara del casquete elipsoidal considerado en el primer apartado, S + es la cara de la<br />

superficie cónica considerada en el segundo y los flujos de F(x,y,z) a través de ellas se han calculado<br />

en esos apartados. Así pues:<br />

zg = 1<br />

<br />

<br />

F(x,y,z) · dσ +<br />

VΩ<br />

E +<br />

y el centroide de Ω es G = (0, 0, − 1<br />

4 ).<br />

S +<br />

<br />

F(x,y,z) · dσ<br />

S +<br />

= 3<br />

<br />

16π −<br />

64π<br />

64π<br />

3<br />

<br />

= − 1<br />

4<br />

NOTA: Caso de no recordar la fórmula del volumen de un cono, este se puede calcular conside-<br />

z + 8<br />

rando que las secciones por planos z = cte del mismo, son discos de radio . El área de estos<br />

4 2 z + 8<br />

discos es π y el volumen del cono se puede calcular mediante la expresión:<br />

4<br />

0 2 z + 8<br />

π dz =<br />

−8 4<br />

32π<br />

3 .


AMPLIACIÓN DE CÁLCULO (Curso 2006/2007) Convocatoria de junio 12.06.07<br />

PROBLEMA 1 (4 puntos) Este problema consta de dos ejercicios independientes.<br />

1. (2 puntos) Determinar los valores reales de a para los cuales converge<br />

Solución:<br />

∞ e<br />

0<br />

−x − eax dx.<br />

x<br />

En primer lugar calculemos el límite del integrando en el origen, usando la regla de L’Hôpital:<br />

e<br />

lím<br />

x→0<br />

−x − eax x<br />

= −(1 + a)<br />

Como el límite existe para todo valor de a, el origen es una singularidad evitable y la integral<br />

solo es impropia en infinito.<br />

Calculemos también el límite del integrando en el infinito:<br />

e<br />

lím<br />

x→∞<br />

−x − eax x<br />

=<br />

<br />

0 a ≤ 0<br />

−∞ a > 0<br />

Es condición necesaria de convergencia que el límite del integrando en el infinito sea nulo u<br />

oscile, por tanto a ≤ 0. Dado que el origen es una singularidad evitable, vamos a descomponer<br />

la integral:<br />

∞ e<br />

0<br />

−x − eax 1 e<br />

dx =<br />

x<br />

0<br />

−x − eax ∞ e<br />

dx +<br />

x<br />

1<br />

−x − eax dx = I0 + I1(a)<br />

x<br />

I0 es propia, nos ocupamos de la convergencia de I1(a). Utilicemos el siguiente criterio de<br />

comparación:<br />

Sean f y g dos funciones continuas en [ a, ∞), con g no negativa; si ∀x ≥ a |f(x)| ≤ g(x)<br />

∞<br />

∞<br />

entonces: ( g converge ⇒ f converge absolutamente).<br />

a<br />

a<br />

∞<br />

Sabemos que e bx dx es convergente para b < 0; entonces<br />

0<br />

<br />

<br />

e<br />

<br />

<br />

−x − eax <br />

<br />

<br />

<br />

x ≤ e−x + e ax , x ≥ 1<br />

Luego para a < 0 la integral I1(a) es convergente. Por último, para a = 0 descomponemos la<br />

integral como sigue:<br />

∞ e<br />

I1(0) =<br />

1<br />

−x ∞ 1<br />

dx −<br />

x 1 x dx,<br />

el primer sumando es convergente y el segundo divergente, luego la integral es divergente.<br />

En conclusión, la integral del enunciado converge solo si a < 0.


2. (2 puntos) Determinar los valores reales de a y b para los cuales converge<br />

y calcular explícitamente I(2, 3).<br />

∞<br />

I(a, b) =<br />

Indicación: Relacionar la integral con una beta de Euler.<br />

Solución:<br />

0<br />

ta (4 + t2 dt<br />

) b<br />

Una de las expresiones de la función beta de Euler es<br />

∞<br />

β(p, q) =<br />

0<br />

xp−1 dx,<br />

(1 + x) p+q<br />

convergente para p, q > 0. Hagamos los cambios adecuados para expresar I(a, b) como una beta;<br />

dado que 4 + t 2 = 4(1 + (t/2) 2 ) el cambio natural es t = 2 √ x que transforma biyectivamente<br />

la semirrecta (0, ∞) en sí misma y no cambia los límites. Se tiene entonces:<br />

I(a, b) = 2a<br />

4b ∞ x<br />

0<br />

a−1<br />

2 2a<br />

dx =<br />

(1 + x) b 4<br />

Para que sea convergente tiene que ser<br />

Por último,<br />

<br />

p =<br />

a + 1<br />

2<br />

Γ<br />

> 0 ∧ q = b − a + 1<br />

2<br />

I(2, 3) = 22<br />

<br />

3<br />

β<br />

43 2<br />

<br />

3<br />

= Γ 1 +<br />

2<br />

1<br />

<br />

2<br />

<br />

a + 1 a + 1<br />

β , b − .<br />

b 2 2<br />

<br />

> 0<br />

<br />

3<br />

, =<br />

2<br />

1<br />

16<br />

= 1<br />

2 Γ<br />

I(2, 3) = π<br />

128 .<br />

<br />

1<br />

=<br />

2<br />

⇐⇒ b ><br />

Γ 2 3<br />

2<br />

Γ(3) ,<br />

√ π<br />

2 ⇒<br />

a + 1<br />

2<br />

> 0.<br />

Gabriela Sansigre.


AMPLIACIÓN DE CÁLCULO (Curso 2006/2007) Convocatoria de junio 12.06.07<br />

NOMBRE . . . . . . . . . . . . . . . . . . . . . . . . . APELLIDOS . . . . . . . . . . . . . . . . . . . . . . . . . . . . . . . . . . . . . . . . . . .<br />

PROBLEMA 2 (3 puntos)<br />

En R 2 se consideran el campo vectorial definido por:<br />

F(x, y) := (x 2 + y 2 ) 2 (2x − 3, 2y + 1)<br />

Número de matrícula. . . . . . . . . . . . . . . . . . . . . . . .<br />

y las circunferencias de ecuaciones cartesianas x 2 + y 2 = 2 y (x − 1) 2 + y 2 = 1, ambas recorridas en<br />

sentido positivo.<br />

Sean, Γ1 el arco de la circunferencia orientada centrada en el origen contenido en el semiplano<br />

x ≥ 1 y Γ2 el arco de la otra circunferencia orientada contenido en el semiplano x ≤ 1. Se pide:<br />

1) Calcular la circulación del campo F(x, y) a lo largo del arco orientado Γ1. (0,75 puntos)<br />

2) Sea E ⊂ R 2 el dominio acotado limitado por el arco Γ2 y por la recta x = 1. Aplicando<br />

adecuadamente el teorema de Green en E, obtener la circulación del campo F(x, y) sobre el<br />

arco orientado Γ2. (1 punto)<br />

3) Sea D ⊂ R2 el domino acotado limitado por las curvas Γ1 y Γ2, sobre el que se considera<br />

definida una densidad que en cada punto es igual al cuadrado de la distancia del punto al<br />

origen. Relacionar la circulación del campo F(x, y) a lo largo de la frontera de D recorrida en<br />

sentido positivo con las coordenadas del centro de gravedad de D (con la densidad dada) y<br />

calcularlas.<br />

(Dato: La masa total de D es MD = 1<br />

(5π − 8)). (1,25 puntos)<br />

4<br />

Respuesta: Se entregará esta hoja y, a lo sumo, una adicional.<br />

1) Puesto que la curva Γ1 es el arco de la circunferencia de ecuación cartesiana x 2 +y 2 = 2 (orientada<br />

positivamente) situado en el semiplano x ≥ 1, unas ecuaciones paramétricas de la misma son:<br />

x(t) = √ 2 cos t , y(t) = √ 2 sin t , t ∈ [− π π<br />

, ] ,<br />

4 4<br />

donde el rango de variación del parámetro t (que es el ángulo polar) se determina sin más que<br />

obtener los ángulos polares de los dos puntos en los que la circunferencia corta a la recta x = 1. Estos<br />

puntos son (1, −1), (1, 1) y sun ángulos polares − π π , , respectivamente. El sentido de recorrido se<br />

4 4<br />

corresponde con la orientación positiva de la circunferencia, por tanto Γ1 se recorre desde (1, −1)<br />

hasta (1, 1), por lo que t ∈ [− π π , 4 4 ].<br />

Para calcular la circulación utilizamos su definición:<br />

<br />

<br />

π/4<br />

F · ds = F(x(t), y(t)) · dt ,<br />

donde:<br />

F(x(t), y(t))·<br />

<br />

Por tanto: <br />

x ′ (t)<br />

y ′ (t)<br />

Γ1<br />

<br />

Γ1<br />

−π/4<br />

= 4 <br />

2 √ 2 cos t − 3 , 2 √ 2 sin t + 1 <br />

<br />

·<br />

F · ds =<br />

π/4<br />

−π/4<br />

x ′ (t)<br />

y ′ (t)<br />

4 √ 2 (cos t + 3 sin t) dt = 4 √ 2<br />

− √ 2 sin t<br />

√ 2 cos t<br />

π/4<br />

−π/4<br />

<br />

= 4 √ 2 (cos(t) + 3 sin(t)) .<br />

cos t dt = 8 .


2) Si representamos el campo F en la forma F(x, y) = (L(x, y), M(x, y)), sus<br />

dos componentes L y M (que son polinomios en dos variables) son funciones<br />

de clase C ∞ . Por otra parte, el dominio E, que se representa en la figura, es<br />

simplemente conexo. Por tanto, se puede aplicar el teorema de Green al campo<br />

y dominio dados, lo que establece la igualdad:<br />

<br />

∂E<br />

<br />

F · ds =<br />

E<br />

∂M<br />

∂x<br />

<br />

∂L<br />

− dxdy ,<br />

∂y<br />

-0.5 0 0.5 1 1.5<br />

donde ∂E representa la frontera de E recorrida en sentido positivo. La curva<br />

∂E está formada por el arco orientado Γ2 y por el segmento orientado de la recta x = 1 comprendido<br />

entres los puntos (−1, 1) y (1, 1), recorrido desde el primero hasta el segundo. Sea S este segmento<br />

orientado. De la igualdad anterior se deduce entonces:<br />

<br />

Γ2<br />

<br />

F · ds =<br />

E<br />

∂M<br />

∂x<br />

<br />

∂L<br />

− dxdy − F · ds<br />

∂y<br />

S<br />

lo que permite obtner la circulación de F sobre Γ2 calculando las dos integrales del segundo miembro.<br />

• Circulación sobre el segmento S. Unas ecuaciones paramétricas de S son:<br />

Por tanto:<br />

x(t) = 1 , y(t) = t , t ∈ [−1, 1] .<br />

<br />

<br />

1<br />

x<br />

F · ds = F(x(t), y(t)) ·<br />

S<br />

−1<br />

′ (t)<br />

y ′ 1<br />

dt =<br />

(t)<br />

−1<br />

1 <br />

= 2 1 + t<br />

0<br />

2 <br />

2<br />

dt = 2 t + t5<br />

<br />

2t3 1<br />

+<br />

5 3 0<br />

• Integral doble. Puesto que:<br />

∂M<br />

∂x<br />

la integral que hay que calcular es:<br />

<br />

E<br />

∂M<br />

∂x<br />

− ∂L<br />

∂y<br />

= 4 <br />

x 2 + y 2<br />

(x + 3 y) ,<br />

1<br />

0.5<br />

0<br />

-0.5<br />

-1<br />

(1 + 2 t) <br />

1 + t 2 2<br />

dt<br />

= 56<br />

15 .<br />

<br />

∂L<br />

− dxdy = 4x<br />

∂y<br />

E<br />

<br />

x 2 + y 2<br />

<br />

dxdy + 12 y<br />

E<br />

<br />

x 2 + y 2<br />

dxdy .<br />

Ahora bien, dado que E es simétrico con respecto al eje de abscisas, la segunda integral del<br />

segundo miembro es cero, puesto que el integrando es impar en y. Es decir:<br />

<br />

E<br />

∂M<br />

∂x<br />

<br />

∂L<br />

− dxdy = 4x<br />

∂y<br />

E<br />

<br />

x 2 + y 2<br />

dxdy .<br />

Para calcular esta integral, tenemos en cuenta que el dominio E es la mitad de la circunferencia<br />

de ecuación cartesiana (x − 1) 2 + y 2 = 1 situada a la izquierda de la recta x = 1, con lo que,<br />

en coordenadas polares trasladadas: x = 1 + r cos θ, y = r sin θ, el correspondiente dominio<br />

transformado se puede describir por las ecuaciones:<br />

E ′ =<br />

<br />

(r, θ) ∈ R 2 , π<br />

2<br />

≤ θ ≤ 3π<br />

2<br />

<br />

, 0 ≤ r ≤ 1 .<br />

E


Entonces, utilizando las mencionadas coordenadas y teniendo en cuenta que el jacobiano de la<br />

transformación es r:<br />

<br />

∂M ∂L<br />

− dxdy<br />

E ∂x ∂y<br />

=<br />

<br />

E ′<br />

4 (1 + r cos θ) <br />

1 + r 2 + 2 r cos θ <br />

r drdθ<br />

=<br />

1 3π/2<br />

dr<br />

<br />

4r(1 + r 2 ) + 4r 2 (3 + r 2 ) cos θ + 8r 3 cos 2 θ <br />

dθ<br />

0<br />

= 4π<br />

1<br />

0<br />

π/2<br />

= 4π − 48<br />

5 ,<br />

r(1 + r 2 )dr − 8<br />

donde: 3π/2<br />

cos 2 θdθ =<br />

π/2<br />

1<br />

0<br />

3π/2<br />

Finalmente, la circulación sobre el arco orientado Γ2 es:<br />

<br />

Γ2<br />

<br />

F · ds =<br />

E<br />

∂M<br />

∂x<br />

π/2<br />

r 2 (3 + r 2 1<br />

)dr + 8 r<br />

0<br />

3 <br />

dr<br />

3π/2<br />

cos<br />

π/2<br />

2 <br />

θ dθ<br />

1<br />

π<br />

(1 + cos 2θ)dθ =<br />

2 2 .<br />

<br />

<br />

∂L<br />

− dxdy − F · ds = 4π −<br />

∂y<br />

S<br />

48<br />

<br />

5<br />

− 56<br />

15<br />

= 4π − 40<br />

3 ,<br />

3) Dado que el centro de gravedad de un recinto de R 2 se expresa en términos de integrales dobles,<br />

en este apartado se nos pide relacionar una circulación con dos integrales dobles.<br />

Un resultado que, en caso de que se cumplan las hipótesis, permite re-<br />

lacionar estos dos tipos de integrales es el teorema de Green en el plano, ya<br />

utilizado en el apartado anterior. Las hipótesis se cumplen claramente en las<br />

condiciones establecidas en el enunciado, puesto que el campo vectorial es de<br />

clase C ∞ en D y el dominio D, que se muestra en la figura, es simplemente<br />

conexo. Su aplicación da lugar a la relación:<br />

<br />

∂D<br />

<br />

F · ds =<br />

D<br />

∂M<br />

∂x<br />

<br />

∂L<br />

− dxdy .<br />

∂y<br />

1<br />

0.5<br />

0<br />

-0.5<br />

-1<br />

D<br />

-0.5 0 0.5 1 1.5<br />

En esta igualdad ∂D es la frontera de D recorrida en sentido positivo que está formada por los arcos<br />

orientados Γ1 y Γ2, de forma que se tiene:<br />

<br />

∂D<br />

<br />

F · ds =<br />

Γ1<br />

<br />

F · ds +<br />

Γ2<br />

F · ds = 8 + 4π − 40<br />

3<br />

= 4π − 16<br />

3 ,<br />

donde las circulaciones del campo F sobre los arcos orientados Γ1 y Γ2 se han calculado en los dos<br />

apartados anteriores.<br />

En lo que se refiere al segundo miembro, su expresión es:<br />

<br />

D<br />

∂M<br />

∂x<br />

<br />

<br />

∂L<br />

− dxdy , = 4 x(x<br />

∂y<br />

D<br />

2 + y 2 <br />

) dxdy + 12 y(x<br />

D<br />

2 + y 2 ) dxdy ,<br />

con lo que, teniendo en cuenta la definición de las coordenadas del centro de gravedad, (Xg(D), Yg(D)),<br />

de D con la densidad dada:<br />

Xg(D) := 1<br />

<br />

x(x 2 + y 2 ) dxdy , Yg(D) := 1<br />

<br />

y(x 2 + y 2 ) dxdy ,<br />

resulta:<br />

MD<br />

<br />

D<br />

D<br />

∂M<br />

∂x<br />

MD<br />

<br />

∂L<br />

− dxdy = 4MDXg(D) + 12MDYg(D) ,<br />

∂y<br />

D


y, por tanto:<br />

<br />

F · ds = 4MDXg(D) + 12MDYg(D) = 4π −<br />

∂D<br />

16<br />

3 ,<br />

que es la relación pedida.<br />

Para calcular las dos coordenadas basta tener en cuenta la simetría de D con respecto al eje de<br />

abscisas, con lo que Yg(D) = 0. Como la masa total, MD, de D es un dato que se proporciona en el<br />

enunciado, de la anterior expresión se deduce el valor de Xg(D), que es:<br />

Xg(D) = 1<br />

<br />

4π −<br />

4MD<br />

16<br />

<br />

=<br />

3<br />

4 (3π − 4)<br />

3 (5π − 8) .


AMPLIACIÓN DE CÁLCULO (Curso 2006/2007) Convocatoria de junio 12.06.07<br />

NOMBRE . . . . . . . . . . . . . . . . . . . . . . . APELLIDOS. . . . . . . . . . . . . . . . . . . . . . . . . . . . . . . . . . . . . . . .<br />

PROBLEMA 3 (3 puntos)<br />

Número de matrícula . . . . . . . . . . . . . . . . . . . . .<br />

1. Sea la región B del plano Y Z definida por 0 ≤ z ≤ y 2 , y ∈ [1, √ 2]. Se considera el sólido Ω<br />

engendrado cuando B gira alrededor del eje y. En cada punto de Ω la densidad es proporcional<br />

al producto de las distancias de dicho punto a los planos coordenados. Se pide el momento de<br />

inercia de Ω respecto del eje y. (1 pto)<br />

2. Sea Γ la curva sobre el plano Y Z definida por z = y 2 , y ∈ [1, √ 2]. Sea Σ la superficie de<br />

revolución engendrada cuando Γ gira alrededor del eje y. Sobre Σ hay definida una distribución<br />

superficial de masa de forma que la densidad en cada punto P de la superficie es proporcional<br />

a la distancia del origen al plano tangente a Σ en el punto P . Se pide calcular la masa de Σ.<br />

(2 ptos.)<br />

Respuesta: Se entregará esta hoja y, a lo sumo, una adicional.<br />

Solución: 1. La densidad es<br />

por lo que<br />

<br />

I =<br />

<br />

= 4K<br />

ρ(x, y, z) = K |x| |z| |y|<br />

ρ(x, y, z)(x<br />

Ω<br />

2 + z 2 )dxdydz =<br />

Ω ′<br />

xzy(x 2 + z 2 )dxdydz<br />

<br />

K |x| |z| |y| (x<br />

Ω<br />

2 + z 2 )dxdydz =<br />

donde Ω ′ es la porción de Ω contenida en el primer octante. Se ha usado que Ω es simétrico<br />

respecto de los planos x = 0 y z = 0 y que la función subintegral es par en x y par en z.<br />

Para simplificar el cálculo de la integral, efectuamos el siguiente cambio a coordenadas cilíndricas<br />

(x, y, z) = φ(ρ, θ, y)<br />

x = ρsenθ , y = y , z = ρ cos θ<br />

y entonces la imagen inversa de Ω ′ es<br />

B := φ −1 (Ω ′ <br />

) = (ρ, θ, y) : 0 ≤ ρ ≤ y 2 , θ ∈ [0, π<br />

2 ], y ∈ [1, √ <br />

2]<br />

Por tanto<br />

<br />

I = 4K (ρsenθ) (ρ cos θ) yρ<br />

B<br />

2 ρdθdρdy =<br />

y2 <br />

= 4K<br />

π<br />

2<br />

0<br />

= 4K 1 1<br />

2 6<br />

cos θsenθdθ<br />

√ 2<br />

1<br />

√ 2<br />

1<br />

y<br />

y 13 dy = 1 1<br />

4K<br />

2 6<br />

0<br />

1<br />

14<br />

ρ 5 dρ<br />

dy =<br />

2 7 − 1 = 127K<br />

42


2. Parametrizamos la superficie Σ en la forma (x, y, z) = α(θ, y)<br />

y operando se obtiene <br />

x = α1(θ, y) = y 2 senθ<br />

y = α2(θ, y) = y y ∈ [1, √ 2], θ ∈ [0, 2π]<br />

z = α3(θ, y) = y 2 cos θ<br />

∂α<br />

∂θ<br />

(θ, y) × ∂α<br />

∂y<br />

<br />

<br />

(θ, y) <br />

= y21 + 4y2 Para hallar la ecuación del plano tangente a Σ en (x, y, z) obtendremos primero un vector<br />

perpendicular a la superficie en dicho punto. Para ello es conveniente trabajar con la ecuación<br />

cartesiana de Σ, que es x 2 + z 2 = y 4 o bien<br />

Se sabe que el gradiente de F en (x, y, z)<br />

F (x, y, z) = x 2 − y 4 + z 2 = 0<br />

∇F (x, y, z) = 2x, −4y 3 , 2z <br />

o bien, el vector paralelo (x, −2y 3 , z) , es perpendicular a Σ en (x, y, z). Por ello, el plano<br />

tangente a Σ en el punto (x, y, z) tiene ecuación<br />

o bien<br />

x (X − x) − 2y 3 (Y − y) + z(Z − z) = 0<br />

xX − 2y 3 Y + zZ − x 2 − 2y 4 + z 2 = 0<br />

que, puesto que x 2 + z 2 = y 4 sobre la superficie, se puede simplificar a<br />

xX − 2y 3 Y + zZ + y 4 = 0<br />

Por lo tanto, la densidad sobre la superficie tiene la forma<br />

y por lo tanto<br />

<br />

MΣ =<br />

= K<br />

|y<br />

ρΣ(x, y, z) = K<br />

4 |<br />

y<br />

= K<br />

x2 + 4y6 + z2 4<br />

y2 y<br />

= K<br />

1 + 4y2 2<br />

<br />

1 + 4y2 <br />

ρΣ(x, y, z)dσ = K<br />

Σ<br />

2π √ 2<br />

0<br />

1<br />

Σ<br />

y2 dσ =<br />

1 + 4y2 y 2<br />

1 + 4y 2 y2 1 + 4y 2 dydθ = K2π<br />

√ 2<br />

1<br />

y 4 dy = 2Kπ<br />

5<br />

<br />

4 √ <br />

2 − 1


AMPLIACIÓN DE CÁLCULO (Curso 2006/2007) Convocatoria de septiembre 18.09.07<br />

PROBLEMA 1 (3 puntos)<br />

Dada la integral impropia<br />

se pide:<br />

∞<br />

F (a) =<br />

0<br />

sen ax<br />

dx<br />

x<br />

1. Determinar el conjunto de valores reales de a para los cuales la integral F (a) converge.<br />

2. Enunciar el teorema de derivabilidad de integrales impropias dependientes de un parámetro. Estudiar<br />

si dicho teorema puede aplicarse a la función F .<br />

3. Sabiendo que F (1) = π/2, calcular F (a) allá donde exista.<br />

4. Basándose en los apartados anteriores,<br />

a) ¿Es F uniformemente convergente en todo su dominio de definición?<br />

b) ¿Es F derivable en algún intervalo?<br />

Respuesta: (Se entregará esta hoja y, a lo sumo, una adicional.)<br />

1. En primer lugar, observamos que la integral no es impropia en el origen: en ese punto el integrando<br />

presenta una singularidad evitable ya que lím x→0 sen(ax)<br />

x = a y este límite existe para cada valor de a.<br />

Así pues, la integral sólo es impropia en infinito, y allí estudiamos su convergencia.<br />

Para a = 0 la integral converge, pues obviamente F (0) = ∞<br />

0 0 dx = 0.<br />

Para a = 0, aplicaremos el Criterio de Abel-Dirichlet:<br />

“Si f, g : [b, ∞) → R son funciones de clase C 1 tales que x<br />

b f es acotada en [b, ∞), g es decreciente,<br />

f (x) g (x) dx es convergente”.<br />

y límx→∞ g (x) = 0, entonces la integral impropia ∞<br />

b<br />

En este caso, tomando, por ejemplo, b = 1, g (x) = 1/x cumple las hipótesis del teorema, y una<br />

primitiva de f (x) = sen (ax) está acotada en [1, ∞) para cada valor a = 0 :<br />

<br />

<br />

x <br />

<br />

f (t) dt<br />

=<br />

<br />

<br />

x <br />

<br />

sen (at) dt<br />

=<br />

<br />

<br />

<br />

<br />

cos (a) − cos (ax) <br />

<br />

2<br />

a ≤<br />

|a|<br />

luego F (a) converge para todo a ∈ R.<br />

1<br />

1<br />

Alternativamente: También puede estudiarse la convergencia en infinito integrando por partes<br />

<br />

B sen ax cos axx=B<br />

dx = − <br />

x<br />

ax − 1<br />

B<br />

<br />

<br />

cos ax 1 cos aB<br />

dx = cos a − −<br />

a x2 a<br />

B<br />

1<br />

B cos ax<br />

dx<br />

a x2 1<br />

x=1<br />

y haciendo tender B a infinito, el límite existe:<br />

∞<br />

<br />

sen ax cos a 1 ∞<br />

dx = −<br />

x a a<br />

pues la última integral impropia converge (absolutamente): ∞<br />

1<br />

2. Enunciemos el teorema pedido:<br />

1<br />

1<br />

1<br />

cos ax<br />

dx<br />

x2 Teorema: Se considera un intervalo abierto I ⊆ R, y la integral impropia<br />

∞<br />

F (a) = f(a, x)dx.<br />

Si se verifica:<br />

b<br />

|cos ax|<br />

x2 dx ≤ ∞ 1<br />

1 x2 dx < ∞.<br />

1


F (a0) converge para algún valor a0 ∈ I,<br />

f(a, x) y ∂f<br />

∂a (a, x) son continuas en I × [b, ∞)<br />

y la integral impropia ∞ ∂f<br />

(a, x)dx<br />

b ∂a<br />

converge uniformemente en I,<br />

entonces F es convergente y derivable en I, y su derivada es<br />

F ′ ∞ ∂f<br />

(a) = (a, x)dx ∀a ∈ I.<br />

∂a<br />

b<br />

Veamos si se verifican las tres hipótesis del teorema, tomando b = 0, para algún intervalo I ⊆ R:<br />

F converge en a0 = 0, pues sabemos que F (0) = 0 (y el apartado 3 afirma que F (1) converge);<br />

f(a, x) = sen(ax)<br />

x<br />

f(a, 0) = límx→0 sen(ax)<br />

x<br />

Por otro lado, ∂<br />

<br />

sen ax<br />

∂a x<br />

es continua en I ×(0, ∞), y puede extenderse por continuidad en x = 0 definiendo<br />

0<br />

= a.<br />

<br />

= cos ax que es una función continua para (a, x) en I × [0, ∞)<br />

La integral impropia de la derivada es<br />

∞<br />

<br />

∂f<br />

∞<br />

(a, x)dx = cos(ax)dx que DIVERGE PARA TODO a.<br />

∂a<br />

0<br />

En efecto, para a = 0 se tiene ∞<br />

0 dx = ∞; para a = 0, aplicamos la definición de convergencia<br />

de integral impropia:<br />

B<br />

lím cos(ax)dx =<br />

B→∞ 0<br />

1<br />

a lím sen (aB) NO EXISTE<br />

B→∞<br />

luego dicha integral impropia no converge para ningún valor de a. Entonces, no tiene sentido<br />

estudiar su convergencia uniforme.<br />

Por ello, se deduce que no puede aplicarse dicho teorema de derivación bajo el signo de la<br />

integral impropia F.<br />

3. Es el ejercicio 1.7, apartado 1, de la colección de ejercicios de la asignatura: Realizamos,<br />

para a > 0, el cambio de variable ax = y que no varía los límites de integración:<br />

∞<br />

<br />

sen ax ∞<br />

<br />

sen y 1 ∞ sen y<br />

π<br />

F (a) =<br />

dx =<br />

dy = dy = F (1) =<br />

x<br />

y/a a y 2 .<br />

0<br />

0<br />

Además, F es una función impar: F (−a) = −F (a) , así que para a < 0, F (a) = − π<br />

2 .<br />

Por tanto, se trata de una función constante a trozos, definida en R de la siguiente forma:<br />

<br />

F (a) =<br />

0 si a = 0<br />

.<br />

signo (a) si a = 0<br />

π<br />

2<br />

4. a) Utilizaremos el teorema de continuidad de funciones definidas como integrales impropias dependientes<br />

de un parámetro. El integrando de F es una función continua en R × [0, ∞) (o que se puede<br />

extender por continuidad); si además F convergiera uniformemente en su dominio de definición<br />

R, entonces F sería también continua en R. Pero no lo es, pues obviamente F es discontinua en<br />

0; así pues, F no converge uniformemente en R.<br />

b) De la expresión explícita de F , se deduce que es derivable en cualquier intervalo que no<br />

contenga al origen (es decir, allá donde es constante). Por ejemplo, en (0, ∞) y en (−∞, 0) .<br />

OBSERVACI ÓN: Nótese que, aunque F sea derivable en un intervalo, no siempre puede<br />

derivarse bajo el signo integral. Aún más, el hecho de no poder aplicar ese teorema de<br />

derivación no indica nada acerca de la derivabilidad o no derivabilidad de F .<br />

0


AMPLIACIÓN DE CÁLCULO (Curso 2006/2007) Convocatoria de septiembre 18.09.07<br />

NOMBRE . . . . . . . . . . . . . . . . . . . . . . . . . APELLIDOS . . . . . . . . . . . . . . . . . . . . . . . . . . . . . . . . . . . . . . . . . . .<br />

PROBLEMA 2 (3 puntos)<br />

Número de matrícula. . . . . . . . . . . . . . . . . . . . . . . .<br />

En R 3 se considera la superficie cónica Σ formada por los segmentos de recta que pasan por el<br />

punto P ≡ (0, 0, 1) y se apoyan en la curva de ecuaciones<br />

Se pide:<br />

1) (1 pto) Calcular la integral<br />

<br />

<br />

I =<br />

(x − 1) 2 + y 2 = 1<br />

z = 0<br />

Σ<br />

<br />

<br />

x 2 + (z − 1) 2 dσ<br />

2) (2 ptos) Sea Ω el sólido homogéneo limitado por el plano z = 0 y la superficie Σ. Calcular el<br />

centro de gravedad de Ω.<br />

Respuesta: Se entregará esta hoja y, a lo sumo, una adicional.<br />

Solución:<br />

1) En primer lugar calcularemos unas ecuaciones paramétricas (x, y, z) = φ(t, λ) de Σ, parametrizando<br />

los segmentos que unen P con la curva. Esta última tiene ecuaciones<br />

x = 1 + cos t<br />

y = sent ; t ∈ [0, 2π]<br />

z = 0<br />

por lo que Σ se puede parametrizar en la forma<br />

es decir,<br />

(x, y, z) = φ(t, λ) = (0, 0, 1) + λ [(1 + cos t, sent, 0) − (0, 0, 1)] ; t ∈ [0, 2π], λ ∈ [0, 1]<br />

Ahora, operando<br />

<br />

<br />

∂φ<br />

<br />

∂t<br />

Por ello<br />

I =<br />

=<br />

=<br />

x = λ(1 + cos t)<br />

y = λsent ; t ∈ [0, 2π], λ ∈ [0, 1]<br />

z = 1 − λ<br />

<br />

∂φ <br />

<br />

× <br />

∂λ = −λ (cos t, sent, cos t + 1) = λ√2 + 2 cos t + cos2 t<br />

<br />

x<br />

Σ<br />

2 + (z − 1) 2 dσ =<br />

<br />

(λ(1 + cos t))<br />

[0,2π]×[0,1]<br />

2 + λ2λ √ 2 + 2 cos t + cos2 tdtdλ =<br />

<br />

<br />

2<br />

λ 2 + 2 cos t + cos<br />

[0,2π]×[0,1]<br />

2 t 1<br />

dtdλ = λ<br />

0<br />

2 2π <br />

dλ 2 + 2 cos t + cos<br />

0<br />

2 t <br />

dt =


= 1<br />

<br />

2π<br />

2 ∗ 2π + 0 + cos<br />

3<br />

0<br />

2 <br />

tdt = 5π<br />

3<br />

2) Por definición<br />

xG =<br />

<br />

Ω xdxdydz<br />

V ol(Ω)<br />

; yG =<br />

<br />

Ω ydxdydz<br />

V ol(Ω)<br />

; zG =<br />

<br />

Ω zdxdydz<br />

V ol(Ω)<br />

Por simetría yG = 0, lo cual también se puede deducir sin más que darse cuenta que Ω es simétrico<br />

respecto del plano y = 0 y la función f(x, y, z) = y es impar en y, con lo que <br />

Ω ydxdydz = 0. Como<br />

Ω es un cono su volumen es un tercio del área de la base por la altura. Por ello<br />

V ol(Ω) = π<br />

3<br />

Calculemos el volumen también integrando. Ω es tal que su intersección con cada plano z = h es un<br />

círculo que denotamos Ch. Por ello una buena elección para integrar es integrar primero en x e y y<br />

luego en z. Por tanto<br />

<br />

1 1<br />

V ol(Ω) = dxdydz = dxdy dz = Área(Cz)dz<br />

0<br />

Ω<br />

0<br />

Para calcular Área(Cz) basta con determinar el radio rz de Cz. Para ello se puede proceder geométricamente<br />

por semejanza de triángulos, que implica<br />

2rz<br />

2<br />

Cz<br />

= 1 − z<br />

1<br />

de donde rz = 1 − z. Calculemos rz de otra forma. Para ello calcularemos la ecuación cartesiana de<br />

Σ, lo cual se puede hacer eliminando t y λ de las ecuaciones paramétricas. Se obtiene<br />

(x − 1 + z) 2 + y 2 = (1 − z) 2<br />

con lo que, de nuevo, se obtiene que rz = 1 − z. Por tanto<br />

V ol(Ω) =<br />

1<br />

0<br />

Área(Cz)dz =<br />

1<br />

0<br />

π(1 − z) 2 dz = π<br />

3<br />

Análogamente,<br />

<br />

zdxdydz<br />

Ω<br />

=<br />

1 1<br />

z dxdy dz = z<br />

0 Ch<br />

0<br />

Área(Cz)dz<br />

1<br />

= π z(1 − z)<br />

0<br />

2 =<br />

dz =<br />

1<br />

π (z − 2z<br />

0<br />

2 + z 3 )dz = π<br />

12<br />

con lo que<br />

zG =<br />

π<br />

12<br />

π<br />

3<br />

Este resultado también se podía haber obtenido directamente recordando que en un cualquier cono<br />

homogéneo el centro de gravedad está situado a una distancia de la base igual a un cuarto de la<br />

altura del cono.<br />

Una vez determinados yG y zG la posición de xG se puede determinar directamente. En efecto,<br />

para z = 1/4 se tiene rz = 1 − 1/4 = 3/4, con lo que xG = 3/4.<br />

Calculemos xG integrando. Si Ω ∗ denota la porción del cono correspondiente a y ≥ 0, y C ∗ h la<br />

porción de Ch correspondiente a y ≥ 0 tenemos, integrando primero en x luego en y y finalmente en<br />

z<br />

<br />

H : =<br />

Ω<br />

<br />

xdxdydz = 2<br />

Ω∗ xdxdydz = 2<br />

= 1<br />

4<br />

<br />

1 <br />

0<br />

C ∗ z<br />

xdxdy<br />

<br />

dz =


= 2<br />

= 2 1<br />

=<br />

⎛ ⎛ √ ⎞ ⎞<br />

1 1−z 2<br />

1−z+ (1−z) −y2 ⎝ ⎝ √ xdx⎠<br />

dy⎠<br />

dz =<br />

2<br />

0 0 1−z− (1−z) −y2 <br />

1 <br />

1−z <br />

1 − z + (1 − z)<br />

2 0 0<br />

2 − y2 2 <br />

− 1 − z − (1 − z) 2 − y2 <br />

2<br />

dy =<br />

1 1−z <br />

4 (1 − z) (1 − z)<br />

0 0<br />

2 − y2 1 1−z <br />

dy dz = 4 (1 − z) (1 − z)<br />

0<br />

0<br />

2 − y2 <br />

dy dz<br />

y efectuando el cambio y = (1 − z)u<br />

1 <br />

H =<br />

1<br />

4 (1 − z) (1 − z)<br />

0<br />

0<br />

2 − (1 − z) 2 u2 <br />

(1 − z) du dz =<br />

=<br />

1<br />

1 √ <br />

4 (1 − z) |1 − z| 1 − u2du dz<br />

=<br />

0<br />

0<br />

1<br />

4 (1 − z)<br />

0<br />

3 1 √ <br />

dz 1 − u2du = 4<br />

0<br />

1<br />

1 √ <br />

1 − u2du 4 0<br />

donde se ha usado que 1 − z ≥ 0 para z ∈ [0, 1]. Tras efectuar el cambio u =senw, se obtiene<br />

con lo que<br />

xG =<br />

H = π<br />

4<br />

π<br />

4<br />

π<br />

3<br />

= 3<br />

4


AMPLIACIÓN DE CÁLCULO (Curso 2006/2007) Convocatoria de septiembre 18.09.07<br />

PROBLEMA 3 (4 puntos)<br />

Este problema tiene por objeto mostrar un campo solenoidal en un dominio simplemente conexo de R 3<br />

que no es un rotacional.<br />

1. En una esfera centrada en el origen y de radio R, calcúlese el área de un casquete esférico Qh, de<br />

altura h (0 < h < 2R) y borde circular Ch.<br />

h<br />

C h<br />

Figura 1: Casquete Qh<br />

2. Calcúlese el valor del parámetro α para que el campo F(r) = r α r sea solenoidal en<br />

Ω = {r ∈ R 3 , 1 ≤ r ≤ 2}.<br />

3. Supóngase Qh ⊂ Ω (el casquete del primer apartado con 1 < R < 2) y orientado según el vector<br />

normal exterior a la esfera. Calcúlese F con el valor de α que hace F solenoidal. Calcúlese<br />

<br />

Qh<br />

asimismo lím<br />

h→2R<br />

F.<br />

Qh<br />

<br />

4. Sea G un campo cualquiera de clase 1 en Ω. Calcúlese lím<br />

h→2R<br />

potencial vector en Ω.<br />

G. Conclúyase que F no admite<br />

Ch<br />

Indicación: Úsese el teorema de acotación de la integral de línea. Supóngase que G es un potencial vector de F,<br />

aplíquese el teorema de Stokes y lléguese a contradicción.<br />

Respuesta:<br />

1. Calculemos el área del casquete por dos procedimientos:<br />

a) Mediante la parametrización natural en función de los ángulos polar y acimutal:<br />

Φ(ϕ, θ) = (R sen ϕ cos θ, R sen ϕ sen θ, R cos ϕ), (ϕ, θ) ∈ [0, ϕ0] × [0, 2π]<br />

con ϕ0 tal que cos ϕ0 = (R − h)/R. El vector normal asociado es<br />

∂Φ ∂Φ<br />

×<br />

∂ϕ ∂θ<br />

= R sen ϕΦ(ϕ, θ)<br />

y su norma ∂Φ/∂ϕ × ∂Φ/∂θ = R 2 sen ϕ, por lo que el área del casquete será:<br />

<br />

A(Qh) =<br />

[0,ϕ0]×[0,2π]<br />

R 2 sen ϕ dϕ dθ = 2πR 2<br />

ϕ0<br />

0<br />

sen ϕ dϕ = 2πR 2 (− cos ϕ| ϕ0<br />

0 ) = 2πRh.


) Considerando el casquete como superficie de revolución de la función no negativa<br />

f : [R − h, R] → R<br />

f(x) = √ R 2 − x 2<br />

alrededor del eje OX. Sabemos que en este caso el área viene dada por<br />

R <br />

A(Qh) = 2π f(x) 1 + f ′ (x) 2 dx<br />

R−h<br />

<br />

Cuenta habida que f(x) 1 + f ′ (x) 2 = R se concluye que el área es A(Qh) = 2πRh.<br />

Nota: El cálculo del área de un casquete esférico está propuesto en el ejercicio 7.5.(a).<br />

2. El campo F será solenoidal en Ω si su divergencia es nula. Recordemos la fórmula para calcular la<br />

divergencia de un campo que sea producto de un campo escalar g por un campo vectorial G:<br />

div(gG) = ∇g · G + g div G.<br />

En nuestro caso, g(r) = r α y G(r) = r. Calculemos la derivada parcial de g respecto de x:<br />

∂(x2 + y2 + z2 ) α/2<br />

=<br />

∂x<br />

α<br />

2 (x2 + y 2 + z 2 ) (α/2)−1 2x = αr α−2 x<br />

Análogamente, ∂g/∂y = αrα−2y, ∂g/∂z = αrα−2z, así pues ∇g(r) = αrα−2 r y div G(r) = 3,<br />

luego:<br />

div F(r) = αr α−2 r · r + 3r α = 0 ⇐⇒ (α + 3)r α = 0 ⇐⇒ α = −3.<br />

Observemos que el campo F(r) = r<br />

está bien definido en Ω, ya que la única singularidad de F es<br />

r3 el origen que no pertenece a Ω. (Para más detalles sobre campos centrales consúltese el ejercicio 6.7.)<br />

3. El vector normal unitario exterior a la esfera es r/r por lo tanto:<br />

<br />

<br />

r r<br />

1 1<br />

F = · dσ = dσ =<br />

Qh Qh r3 r Qh r2 R2 A(Qh) = 2πh<br />

R .<br />

<br />

Por último, lím<br />

h→2R<br />

2πh<br />

F = lím = 4π.<br />

h→2R R<br />

Qh<br />

4. Sea G un campo cualquiera de clase 1 en Ω; sea M una cota de G en Ω: su existencia está asegurada<br />

por la continuidad de G en el compacto Ω, y Lh la longitud de la circunferencia Ch; según el teorema<br />

de acotación de la integral de línea se tiene:<br />

<br />

<br />

<br />

<br />

G<br />

≤ MLh,<br />

Ch<br />

cuando h tiende a 2R el casquete tiende a la esfera y su borde a un punto, así que la longitud Lh<br />

tiende a cero, además la constante M es independiente de R, por lo que lím G = 0.<br />

h→2R<br />

Supongamos ahora que F admite un potencial vector G de clase 1 en Ω (esto es, F = rot G); estamos<br />

en condiciones de aplicar el teorema de Stokes al campo G sobre la superficie Qh, suponiendo el<br />

casquete y su borde Ch orientados coherentemente:<br />

<br />

<br />

F = rot G = G<br />

Qh<br />

Qh<br />

Si tomamos límite cuando h tiende a 2R en los extremos de la igualdad anterior tenemos<br />

<br />

<br />

4π = lím<br />

h→2R<br />

F = lím<br />

h→2R<br />

G = 0<br />

contradicción que viene de suponer que F admite un potencial vector.<br />

Qh<br />

Ch<br />

Ch<br />

Ch<br />

Gabriela Sansigre.


AMPLIACIÓN DE CÁLCULO (Curso 2007/2008) Convocatoria de febrero 12.02.08<br />

PROBLEMA 1 (4 puntos)<br />

Nota: Los apartados de este problema son independientes.<br />

Sea Γ el arco de la rama de la hipérbola y 2 − z 2 = 1, x = 0, limitado por los planos y = 1 e y = 2.<br />

1) Calcúlese la masa total del arco Γ cuando sobre él se considera definida una densidad lineal de<br />

masa que, en cada punto, es inversamente proporcional a la distancia del punto al origen.<br />

(1 punto)<br />

2) Sea Σ la superficie generada cuando el arco Γ gira alrededor del eje y. Sobre Σ se considera<br />

definida una densidad de masa que en cada punto de la superficie es igual a la distancia al<br />

origen del plano tangente a la superficie en ese punto. Calcúlense las coordenadas del centro<br />

de gravedad de Σ.<br />

(1 punto)<br />

3) Sea Ω ∈ R 3 el sólido acotado limitado por los planos z = 1, z = −1 y por la superficie de<br />

revolución que se obtiene cuando el arco Γ gira alrededor del eje z. Sobre Ω se considera<br />

definida una densidad de masa que en cada punto es igual a la distancia del punto al plano<br />

XY . Calcúlese el momento de inercia de Ω con respecto a la recta que pasa por el punto (1, 1, 0)<br />

y es paralela al eje z. (2 puntos)<br />

Respuesta:<br />

1) La densidad lineal de masa definida sobre el arco Γ es:<br />

d(x, y, z) =<br />

k<br />

√ x 2 + y 2 + z 2 ,<br />

donde k > 0 es la constante de proporcionalidad. Por tanto, la masa total del arco vendrá dado por<br />

la integral curvilínea:<br />

<br />

MΓ =<br />

k<br />

√ ds .<br />

x2 + y2 + z2 Γ<br />

Ahora bien, dado que la densidad es simétrica con respecto al origen y el arco es simétrico con<br />

respecto al eje OY la integral anterior es igual al doble de la integral extendida al semi-espacio<br />

z ≥ 0; es decir:<br />

<br />

MΓ = 2 d(x, y, z)ds<br />

Γz≥0<br />

Puesto que el arco está situado en el plano Y Z, una parametrización del segmento situado en el<br />

semiespacio z ≥ 0 es: r(u) = (x(u), y(u), z(u)), dónde<br />

x(u) = 0 , y(u) = u , z(u) = + √ u 2 − 1 , u ∈ [1, 2] ,<br />

con lo que el vector tangente viene dado por<br />

r ′ (u) =<br />

<br />

0, 1,<br />

<br />

u<br />

√ =⇒ r<br />

u2 − 1<br />

′ <br />

2u<br />

(u) =<br />

2 − 1<br />

u2 − 1 .


Por lo tanto,<br />

2<br />

MΓ = 2<br />

1<br />

2<br />

= 2k<br />

1<br />

k<br />

<br />

x(u) 2 + y(u) 2 + z(u) 2 r′ (u) du = 2k<br />

2<br />

1<br />

√ u 2 − 1 du = 2k ArgCh(2) = 2k log(2 + √ 3) ,<br />

1<br />

1<br />

√ 2u 2 − 1<br />

dónde la última integral se resuelve mediante el cambio de variable t = Ch(u):<br />

2<br />

1<br />

<br />

1<br />

ArgCh(2)<br />

√ du =<br />

u2 − 1 0<br />

<br />

2u 2 − 1<br />

u 2 − 1 du<br />

<br />

Sh(t) ArgCh(2)<br />

dt = dt = ArgCh(2) .<br />

|Sh(t)| 0<br />

Parametrización alternativa:<br />

El arco Γ admite la parametrización s(v) = (x(v), y(v), z(v)), dónde<br />

con vector tangente dado por<br />

Por lo tanto,<br />

MΓ =<br />

= k<br />

√ 3<br />

− √ 3<br />

x(v) = 0 , y(v) = √ v 2 + 1 , z(u) = v , v ∈ [− √ 3, √ 3] ,<br />

√ 3<br />

− √ 3<br />

s ′ (v) =<br />

<br />

0,<br />

<br />

v<br />

√ , 1 =⇒ s<br />

v2 + 1 ′ <br />

2v<br />

(v) =<br />

2 + 1<br />

v2 + 1 .<br />

k<br />

<br />

x(v) 2 + y(v) 2 + z(v) 2 s′ (v) dv = k<br />

√ 3<br />

− √ 3<br />

1<br />

√ 2v 2 + 1<br />

1<br />

√ v 2 + 1 dv = 2k ArgSh( √ 3) = 2k log(2 + √ 3) ,<br />

<br />

2v 2 + 1<br />

v 2 + 1 dv<br />

dónde, en este caso, la última integral se resuelve de forma análoga al caso anterior pero mediante el<br />

cambio de variable t = Sh(v).<br />

2) Para obtener la expresión de la densidad definida sobre la superficie Σ es necesario determinar<br />

la ecuación del plano tangente a Σ en cada uno de sus puntos. Si n(x, y, z) denota un vector normal<br />

cualquiera a la superficie en el punto (x, y, z) de la misma, la ecuación cartesiana del plano tangente<br />

es:<br />

n(x, y, z) · (X − x, Y − y, Z − z) = 0 ,<br />

donde (X, Y, Z) representan las coordenadas cartesianas de los puntos del plano. La distancia de este<br />

plano al origen y, por tanto, la densidad ρ(x, y, z) definida sobre Σ, se puede representar entonces<br />

por<br />

ρ(x, y, z) =<br />

|n(x, y, z) · (x, y, z)|<br />

n(x, y, z)<br />

Denotando por MΣ la masa total de Σ, que viene dada por la integral de superficie:<br />

MΣ =<br />

las coordenadas de su centro de gravedad,<br />

<br />

Σ<br />

ρ(x, y, z) dσ ,<br />

Cgσ = (Xg, Y g, Zg) ,<br />

.


son, por definición, las siguientes integrales de superficie:<br />

Xg = 1<br />

MΣ<br />

Y g = 1<br />

MΣ<br />

Zg = 1<br />

MΣ<br />

<br />

Σ<br />

<br />

Σ<br />

<br />

Σ<br />

x ρ(x, y, z) dσ = 0 ,<br />

y ρ(x, y, z) dσ ,<br />

z ρ(x, y, z) dσ = 0 ,<br />

donde la coordenadas Xg y Zg son nulas debido a que la superficie es simétrica con respecto al eje<br />

OY y, por tanto, la densidad es una función par con respecto a las coordenadas x, z.<br />

Para calcular las dos integrales de superficie que faltan, tenemos en cuenta que Σ es una superficie<br />

de revolución obtenida al girar el arco Γ alrededor del eje OY . Por tanto, tiene por ecuación cartesiana:<br />

Un vector normal al punto (x, y, z) es entonces:<br />

f(x, y, z) := x 2 − y 2 + z 2 − 1 = 0 , 1 ≤ y ≤ 2 .<br />

n(x, y, z) = grad(f(x, y, z)) = 2(x, −y, z) ,<br />

lo que nos permite obtener la expresión de la densidad que resulta ser:<br />

ρ(x, y, z) =<br />

|n(x, y, z) · (x, y, z)|<br />

n(x, y, z)<br />

= 2(x, −y, z) · (x, y, z)<br />

2 √ x 2 + y 2 + z 2<br />

=<br />

1<br />

√ x 2 + y 2 + z 2 ,<br />

donde se ha tenido en cuenta que, sobre la superficie, x 2 − y 2 + z 2 = 1.<br />

Por otra parte, una de las posibles parametrizaciones Σ(u, θ) = (x(u, θ), y(u, θ), z(u, θ)) tiene por<br />

ecuaciones paramétricas:<br />

x(u, θ) = √ u 2 − 1 sen θ , y(u, θ) = u , z(u, θ) = √ u 2 − 1 cos θ , u ∈ [1, 2] , θ ∈ [0, 2π] ,<br />

con lo que un vector normal es:<br />

n(u, θ) = <br />

− √ u 2 − 1 sen θ , u , √ u 2 − 1 cos θ <br />

Así pues, el valor de la densidad sobre la superficie es:<br />

con lo que se tiene:<br />

<br />

MΣ =<br />

<br />

=<br />

Finalmente,<br />

ρ(x(u, θ), y(u, θ), z(u, θ)) =<br />

Σ<br />

ρ(x, y, z) dσ =<br />

(u,θ)∈[1,2]×[0,2π]<br />

Y g = 1<br />

2π<br />

y el centro de gravedad es<br />

= 1<br />

2π<br />

<br />

<br />

(u,θ)∈[1,2]×[0,2π]<br />

dudθ = 2π .<br />

Σ<br />

2π<br />

0<br />

|n(u, θ) · Σ(u, θ)|<br />

n(u, θ)<br />

yρ(x, y, z) dσ = 1<br />

<br />

2π<br />

2<br />

dθ udu =<br />

1<br />

u2<br />

<br />

2<br />

<br />

=<br />

2 <br />

3<br />

2 ,<br />

Cgσ = (0 , 3<br />

, 0) .<br />

2<br />

=⇒ n(u, θ) = √ 2u 2 − 1 .<br />

=<br />

1<br />

n(u, θ) ,<br />

ρ(x(u, θ), y(u, θ), z(u, θ)) n(u, θ) dudθ<br />

1<br />

(u,θ)∈[1,2]×[0,2π]<br />

u dudθ


3) El sólido Ω es simétrico con respecto al eje OZ y la densidad, que viene dada por Λ(x, y, z) = |z|,<br />

es par en la variable z. Esto nos permite concluir que el centro de gravedad de Ω se encuentra situado<br />

en el eje OZ.<br />

Se puede entonces aplicar el teorema de Steiner según el cual el momento de inercia con respecto<br />

al eje pedido, I(0,1,1), se puede calcular a partir de la expresión<br />

I(0,1,1) = IOZ + MΩ D 2<br />

dónde MΩ es la masa de Ω, IOZ es el momento de inercia con respecto al eje OZ (que es paralelo al<br />

eje dado y pasa por el centro de gravedad de Ω) y D = √ 2 es la distancia entre los dos ejes.<br />

La masa de Ω y su momento de inercia con respecto al eje OZ vienen dados por las integrales<br />

triples:<br />

<br />

<br />

MΩ = |z| dxdydz , IOZ = dz(x, y, z) 2 |z| dxdydz<br />

Ω<br />

dónde dz(x, y, z) = √ x 2 + y 2 es la distancia al eje OZ.<br />

Ahora bien, la simetría de Ω con respecto al origen y el hecho de que las funciones a integrar sean<br />

pares en las tres variables x, y y z, permite calcular las dos integrales como el doble de las mismas<br />

integrales pero extendidas a la parte de Ω situada en el semi-espacio z ≥ 0. Esta parte de Ω queda<br />

descrita adecuadamente en coordenadas cilíndricas por las ecuaciones<br />

x = r sen θ y = r cos θ , z = z, θ ∈ [0, 2π] , r ∈ [0, √ z 2 + 1] , z ∈ [0, 1] .<br />

Al utilizarlas hay que tener en cuenta que el jacobinao del cambio a cilíndricas es |J| = r.<br />

Se tiene entonces:<br />

Y:<br />

MΩ =<br />

=<br />

<br />

2<br />

Ω(z≥0)<br />

2π 1<br />

2 dθ<br />

0 0<br />

IOZ =<br />

=<br />

<br />

2<br />

Ω(z≥0)<br />

2π 1<br />

2 dθ<br />

0 0<br />

<br />

z dxdydz = 2<br />

z z2 + 1<br />

2<br />

dz = 2π<br />

Ω ′<br />

1<br />

z(x 2 + y 2 <br />

) dxdydz = 2<br />

z (z2 + 1) 2<br />

4<br />

dz = π<br />

1<br />

0<br />

Ω<br />

zr dθdzdr = 2<br />

0<br />

Ω ′<br />

2π<br />

0<br />

dθ<br />

z (z 2 + 1) dz = 3π<br />

2 .<br />

zr 3 dθdzdr = 2<br />

z (z 2 + 1) 2 dz = 7π<br />

6 .<br />

2π<br />

0<br />

1<br />

0<br />

dθ<br />

z dz<br />

1<br />

0<br />

√ z 2 +1<br />

0<br />

z dz<br />

r dr<br />

√ z 2 +1<br />

Finalmente, utilizando al teorema de Steiner ya mencionado, el momento de inercia de Ω con respecto<br />

al eje paralelo al eje OZ y que pasa por el punto (0, 1, 1) es:<br />

I(0,1,1) = IOZ + 2 MΩ = 7π<br />

6<br />

+ 3π = 25π<br />

6 .<br />

0<br />

r 3 dr


AMPLIACIÓN DE CÁLCULO (Curso 2007/2008) Convocatoria de febrero 12.02.08<br />

PROBLEMA 2 (3 puntos)<br />

1. (0, 75 puntos) Sea f un campo escalar de clase 2 en R 2 . Se dice que una función g es conjugada<br />

de f si se verifican las relaciones:<br />

∂g<br />

∂x<br />

= −∂f<br />

∂y ,<br />

∂g<br />

∂y<br />

= ∂f<br />

∂x .<br />

Dese una condición necesaria y suficiente sobre f para que admita una conjugada de clase 2.<br />

2. (1 punto) Calcúlese una función h con h(0) = h ′ (0) = 1 y para la cual f(x, y) = cos x h(y)<br />

cumple la condición hallada en el primer apartado. En tal caso, determínese una conjugada g<br />

de f.<br />

3. (1, 25 puntos) Se considera la curva Γ parametrizada por ϕ(t) = (sec2 t, 2 + tg t), t ∈ [0, π/4]<br />

y el campo vectorial V = (g, f) siendo f y g las funciones del segundo apartado. Calcúlese la<br />

integral de línea V.<br />

Respuesta:<br />

Γ<br />

1. Si se busca una función g de clase dos, sus derivadas parciales de segundo orden cruzadas<br />

coinciden:<br />

lo cual se traduce en<br />

∂2g ∂x∂y = ∂2g ∂y∂x<br />

∂2f ∂x2 = −∂2 f<br />

⇐⇒ ∆f = 0;<br />

∂y2 es decir, la función f tiene que ser armónica. La condición es también suficiente; basta con<br />

integrar una cualquiera de las dos ecuaciones (operación que puede realizarse gracias a la<br />

continuidad de ∂f/∂y),<br />

e imponer que se satisfaga la otra:<br />

∂g<br />

∂y<br />

<br />

∂g<br />

x<br />

(x, y) − (0, y) =<br />

∂y 0<br />

x<br />

g(x, y) − g(0, y) =<br />

− ∂2 <br />

f<br />

x<br />

(t, y)dt =<br />

∂y2 0<br />

0<br />

− ∂f<br />

(t, y)dt<br />

∂y<br />

∂ 2 f<br />

∂x<br />

∂f ∂f<br />

(t, y)dt = (x, y) − (0, y)<br />

2 ∂x ∂x<br />

(tengamos presente que la derivación paramétrica es posible ya que tanto ∂f/∂y como ∂ 2 f/∂y 2<br />

son continuas) con lo que la función g buscada queda:<br />

x<br />

g(x, y) =<br />

siendo c una constante arbitraria.<br />

0<br />

− ∂f<br />

y<br />

(t, y)dt +<br />

∂y 0<br />

∂f<br />

(0, t)dt + c,<br />

∂x


2. Calculemos el laplaciano de f(x, y) = cos x h(y),<br />

∂2f ∂x2 + ∂2f ∂y2 = − cos x h(y) + cos x h′′ (y) = cos x(−h(y) + h ′′ (y)) = 0 ⇐⇒<br />

h ′′ (y) − h(y) = 0 ⇐⇒ h(y) = ae y + be −y , a, b constantes<br />

Al imponer las condiciones iniciales h(0) = h ′ (0) = 1 se tiene a+b = 1, a−b = 1 ⇒ a = 1, b = 0.<br />

Entonces h(y) = e y y f(x, y) = cos x e y .<br />

Para calcular g aplicamos la fórmula deducida en el apartado anterior; como nos piden una<br />

cualquiera, tomamos la constante c nula:<br />

x<br />

g(x, y) =<br />

0<br />

− ∂f<br />

x<br />

(t, y)dt = −ey cos t dt = − sen x e<br />

∂y 0<br />

y .<br />

3. Observemos que el campo V = (g, f) es conservativo ya que se cumple la condición ∂g/∂y =<br />

∂f/∂x que, en un simplemente conexo (todo R 2 en este caso) además de necesaria es también<br />

suficiente. La integral de línea depende exclusivamente de los extremos inicial y final de la<br />

curva que son A = ϕ(0) = (1, 2), B = ϕ(π/4) = (2, 3). Podemos proceder de dos formas, o<br />

bien calculando un potencial escalar de V o bien integrando a lo largo del segmento de extremos<br />

A y B.<br />

3 · 1 Cálculo de la integral de línea a partir de un campo h, potencial escalar de V.<br />

∂h<br />

∂y (x, y) = f(x, y) = cos x ey podemos elegir h(x, y) = cos x e y ,<br />

<br />

V = ∇h = h(B) − h(A) = e 3 cos 2 − e 2 cos 1.<br />

3 · 2 Cálculo de la integral de línea a lo largo del segmento [A, B].<br />

Γ<br />

Γ<br />

Sea ψ(t) = (1, 2) + t(1, 1), t ∈ [0, 1] una parametrización del segmento. Entonces:<br />

<br />

Γ<br />

V =<br />

1<br />

0<br />

V(ψ(t)) · ψ ′ (t)dt =<br />

1<br />

Una primitiva de e t (cos t − sen t) es e t cos t, así pues:<br />

<br />

Γ<br />

0<br />

e 2+t (cos(t + 1) − sen(t + 1))dt.<br />

V = e 2+t cos(t + 1)| 1 0 = e 3 cos 2 − e 2 cos 1.<br />

Gabriela Sansigre.


AMPLIACIÓN DE CÁLCULO (Curso 2007/2008) Convocatoria de febrero 12.02.08<br />

NOMBRE . . . . . . . . . . . . . . . . . . . . . . . . . APELLIDOS . . . . . . . . . . . . . . . . . . . . . . . . . . . . . . . . . . . . . . . . . . .<br />

PROBLEMA 3: (3 puntos)<br />

Número de matrícula. . . . . . . . . . . . . . . . . . . . . . . .<br />

1. En R 3 se considera el campo vectorial F(r) = v × r, donde v es un vector fijo. Determinar<br />

todas las funciones f : R → R tales que el campo f (v · r) r es un potencial vector de F.<br />

(0’5 puntos)<br />

2. Sea S la porción del cilindro 2x 2 + (y + z − 1) 2 = 1 limitada entre los planos z = 0 y z = 3,<br />

orientada según la normal exterior al cilindro. Relacionar el flujo del campo F a través de S<br />

con dos integrales curvilíneas, justificando la respuesta. (1 punto)<br />

3. Calcular dicho flujo cuando v = (1, 0, 1), utilizando el resultado del apartado anterior.<br />

(1’5 puntos)<br />

Respuesta: (Se entregará esta hoja y, a lo sumo, una adicional)<br />

1. Impongamos que el rotacional del campo G (r) = f (v · r) r sea F (r) = v × r : aplicando las<br />

propiedades del rotacional, y el hecho de que rot(r) = 0, se tiene<br />

rot (f (v · r) r) = f ′ (v · r) v × r+f (v · r) rot (r) = f ′ (v · r) v × r.<br />

Este rotacional es igual a v × r si y sólo si f ′ (v · r) = 1 para todo vector de posición r, es<br />

decir, si y sólo si f ′ (t) = 1 para todo t real. Por tanto, las funciones pedidas son todas las de<br />

la forma<br />

f (t) = t + c, c ∈ R.<br />

Así pues, los potenciales vectores de F son los campos de la forma<br />

donde c es una constante arbitraria.<br />

G (r) = (v · r+c) r = (v · r) r+cr<br />

2. La superficie cilíndrica S : 2x 2 + (y + z − 1) 2 = 1, limitada por los planos z = 0 y z = 3 no es<br />

cerrada; la cerraremos con las tapas superior e inferior:<br />

S1 : 2x 2 + (y + 2) 2 ≤ 1 en el plano z = 3<br />

S0 : 2x 2 + (y − 1) 2 ≤ 1 en el plano z = 0.<br />

Denotaremos C1, C0 a sus respectivas curvas borde; obsérvese que se trata de elipses con distinto<br />

centro, pero con iguales longitudes de semiejes: 1/ √ 2 y 1.<br />

Sabemos por el apartado anterior que F es solenoidal, pues admite un potencial vector. Por<br />

tanto, aplicando el Teorema de la divergencia de Gauss a la superficie cerrada, se tiene que el<br />

flujo de F a través de la cara exterior de S es<br />

<br />

S<br />

<br />

F ds = F ds+ F ds<br />

S1<br />

S0


donde tanto S1 como S0 deben estar orientadas según la normal que apunta al interior del<br />

cilindro, es decir, S1 con vector normal unitario (0, 0, −1) y S0 con vector normal unitario<br />

(0, 0, 1).<br />

Para transformar dichos flujos en integrales curvilíneas, aplicamos el Teorema de Stokes en<br />

cada una de ellas; puede aplicarse puesto que F =rot(G) y G ∈ C 1 (R 3 ) :<br />

<br />

<br />

S1<br />

S0<br />

F ds =<br />

F ds =<br />

<br />

<br />

C1<br />

C0<br />

G dl donde la curva C1 está orientada negativamente en el plano z = 3 y<br />

G dl donde la curva C2 está orientada positivamente en el plano z = 0.<br />

En efecto, el Teorema de Stokes indica que las orientaciones de la superficie y su curva borde<br />

deben ser coherentes; en el caso de la superficie S1, que está orientada según el vector normal<br />

unitario (0, 0, −1) , la orientación coherente de su curva borde C1 es la orientación negativa.<br />

Por fin, la relación buscada es<br />

<br />

S<br />

<br />

<br />

<br />

F ds = G dl+ G dl = (v · r) r dl+ (v · r) r dl<br />

C1<br />

C0<br />

C1<br />

C0<br />

donde se ha elegido el potencial vector G con c = 0. (Si fuera c = 0, se tendría la expresión<br />

idéntica, puesto que <br />

C cr dl =0 al tratarse de la circulación de un campo conservativo a lo<br />

largo de una curva cerrada).<br />

3. Para v = (1, 0, 1) , el potencial vector es (v · r) r = (x + z) r. Aplicando lo anterior,<br />

<br />

S<br />

<br />

<br />

(1, 0, 1) × r ds = (x + z) r dl+ (x + z) r dl<br />

C1<br />

C0<br />

donde ambas curvas deben estar orientadas convenientemente. La curva C0, de ecuaciones<br />

2x 2 + (y − 1) 2 = 1, z = 0 debe estar parametrizada como<br />

r (t) = (x (t) , y (t) , z (t)) =<br />

<br />

cos t<br />

√2 , 1 + sen t, 0<br />

para que su orientación sea positiva. Su vector derivada es<br />

r ′ (t) = (x ′ (t) , y ′ (t) , z ′ <br />

<br />

sen t<br />

(t)) = − √ , cos t, 0 .<br />

2<br />

Por ello, la circulación a lo largo de C0 es<br />

<br />

C0<br />

(x + z) r dl =<br />

=<br />

2π<br />

0<br />

2π<br />

0<br />

x (t) (r (t) · r ′ (t)) dt=<br />

cos t<br />

√2<br />

t ∈ [0, 2π]<br />

<br />

1<br />

sen t cos t + cos t dt.<br />

2<br />

Dado que las integrales del tipo 2π<br />

0 senp t cos t dt o 2π<br />

0 sen t cosp t dt son nulas (para p ≥ 0),<br />

queda solamente<br />

<br />

C0<br />

(x + z) r dl = 1<br />

2π<br />

√ cos<br />

2 0<br />

2 t = π √ =<br />

2<br />

√ 2<br />

2 π.<br />

Calculemos ahora la integral a lo largo de C1, de ecuaciones 2x2 + (y + 2) 2 = 1, z = 3. Si la<br />

parametrizamos como<br />

<br />

cos t<br />

r (t) = (x (t) , y (t) , z (t)) = √2 , −2 + sen t, 3 t ∈ [0, 2π]


entonces<br />

r ′ (t) = (x ′ (t) , y ′ (t) , z ′ (t)) =<br />

<br />

−<br />

<br />

sen t<br />

√ , cos t, 0<br />

2<br />

pero queda orientada positivamente, y deseamos la orientación contraria; por tanto, cambiamos<br />

de signo la circulación obtenida:<br />

<br />

C1<br />

(x + z) r dl =<br />

2π<br />

− (x (t) + 3) (r (t) · r<br />

0<br />

′ (t)) dt=<br />

=<br />

=<br />

<br />

2π<br />

1<br />

<br />

cos t<br />

− √2 + 3 sen t cos t − 2 cos t<br />

0<br />

2 2<br />

2π<br />

√ cos<br />

2 0<br />

2 t = √ 2π.<br />

Finalmente, se obtiene que el flujo pedido es:<br />

√<br />

2<br />

F ds =<br />

S 2 π + √ 2π = 3√2 2 π.<br />

ALTERNATIVA PARA EL APARTADO 3: calcular las integrales de superficie a través de las<br />

superficies planos S1 y S0, orientadas convenientemente. Por ejemplo, denotando el dominio plano<br />

D0 = {(x, y) ∈ R 2 , 2x 2 + (y − 1) 2 ≤ 1} se tiene que<br />

<br />

S0<br />

F ds =<br />

=<br />

dt =<br />

<br />

<br />

(1, 0, 1) × (x, y, 0) dr = ((1, 0, 1) × (x, y, 0)) · (0, 0, 1) dxdy =<br />

S0<br />

<br />

<br />

<br />

<br />

<br />

D0 <br />

<br />

1<br />

x<br />

0<br />

0<br />

y<br />

0<br />

D0<br />

<br />

1 <br />

<br />

<br />

0 dxdy = ydxdy = 1 ·<br />

<br />

D0<br />

1 <br />

Área (D0) = π √<br />

2<br />

√ =<br />

2 2 π<br />

<br />

donde se ha usado que D0 ydxdy = yG· Área(D0) y el centro geométrico del interior de una<br />

elipse coincide con el centro de la elipse. En este caso la elipse tiene centro (0, 1) y las longitudes de<br />

sus semiejes son 1/ √ 2 y 1, luego el área es π/ √ 2.<br />

De forma análoga, para S1 definimos el dominio plano D1 = {(x, y) ∈ R2 , 2x2 + (y + 2) 2 ≤ 1} :<br />

<br />

S1<br />

<br />

<br />

F ds = (1, 0, 1) × (x, y, 3) dr = ((1, 0, 1) × (x, y, 3)) · (0, 0, −1) dxdy =<br />

=<br />

S1<br />

<br />

<br />

<br />

<br />

− <br />

D1 <br />

<br />

1<br />

x<br />

0<br />

0<br />

y<br />

0<br />

D1<br />

<br />

1 <br />

<br />

<br />

<br />

3 dxdy = − ydxdy = − (−2)<br />

<br />

D1<br />

1 <br />

Área (D1) = 2π<br />

√ =<br />

2 √ 2π<br />

puesto que el centro de la elipse ahora es (0, −2) , aunque su área es la misma. Finalmente, llegamos<br />

al mismo resultado:<br />

√<br />

2<br />

F ds=<br />

2 π + √ 2π = 3√2 2 π.<br />

S1


AMPLIACIÓN DE CÁLCULO (Curso 2007/2008) Convocatoria de junio 17.06.08<br />

NOMBRE . . . . . . . . . . . . . . . . . . . . . . . . . . . . . . . . APELLIDOS . . . . . . . . . . . . . . . . . . . . . . . . . . . . . . . . . . . . . . . . . . . . . . . . . . . . .<br />

PROBLEMA 1 (4 puntos)<br />

Número de matrícula . . . . . . . . . . . . . . . . . . . . . . . . . . . . . . . .<br />

1) (1.5 ptos.) Determinar razonadamente los valores de a para los cuales está definida la función<br />

∞<br />

arctan(ax) − arctan x<br />

I(a) =<br />

dx<br />

x<br />

2) (1 pto.) Estudiar si I(a) es derivable para los valores de a hallados en el apartado anterior.<br />

3) (0.5 ptos.) Calcular I(a)<br />

4) (1 pto.) Calcular razonadamente la integral<br />

∞<br />

(ax2 − 1) log x<br />

(1 + a2x2 )(1 + x2 ) dx<br />

para a > 0, a = 1.<br />

0<br />

0<br />

Respuesta: Se entregará esta hoja y, a lo sumo, una adicional.<br />

1)<br />

Obsérvese en primer lugar que si se separa la integral en suma de integrales en la forma<br />

∞<br />

∞<br />

∞<br />

arctan(ax) − arctan(x) arctan(ax) arctan x<br />

dx =<br />

dx −<br />

dx<br />

x<br />

x<br />

x<br />

0<br />

0<br />

se obtiene (es fácil comprobarlo comparando con 1/x) que la primera diverge para todo a = 0 y la segunda es divergente.<br />

Por ello procediendo de esta forma la única consecuencia que se puede extraer sobre la convergencia de I(a) es que<br />

para a = 0 la integral es divergente (pues es suma de una integral convergente y una divergente). Por ello para estudiar<br />

la convergencia de la integral impropia para el resto de los valores de a se debe proceder de otra forma.<br />

Sea<br />

arctan ax − arctan x<br />

f(x, a) := dx<br />

x<br />

Para todo a ∈ R, la función f(∗, a) (es decir, como función de x) es de clase C∞ (0, ∞), luego está acotada en todos<br />

los compactos de dicho conjunto. Además,<br />

lim f(x, a) = lim<br />

x→0 + x→0 +<br />

arctan ax − arctan x L<br />

x<br />

′ Hospital<br />

= lim<br />

x→0 +<br />

0<br />

a<br />

1+a2x2 − 1<br />

1+x2 1<br />

= a − 1<br />

con lo que para todo a ∈ R, la función f(∗, a) está acotada en el entorno de x = 0. Por ello, la integral sólo es impropia<br />

porque el intervalo de integración es no acotado.<br />

π/2<br />

−π/2<br />

0<br />

a=1<br />

a>1<br />

0


Cuando a = 1 la función subintegral se anula luego la integral es convergente. Puesto que arctan(ax) es una función<br />

creciente si a > 0 y decreciente si a < 0 (ver figura) se tiene que para todo a = 1, la función f(∗, a) conserva el signo<br />

en (0, ∞) (es o siempre negativa o siempre positiva). En concreto, para x > 0, arctan ax − arctan x ≥ 0 si a > 1 y<br />

arctan ax − arctan x ≤ 0 si a < 1. Por ello, para estudiar la convergencia de I podemos tomar valor absoluto (pues<br />

al no cambiar de signo la función subintegral, la convergencia es equivalente a la convergencia absoluta) y utilizar los<br />

criterios de convergencia para funciones no negativas.<br />

Cuando a < 0, el numerador de f(∗, a) tiende a −π cuando x → ∞, lo que parece que f(∗, a) se comporta cuando<br />

x → ∞ ”como −1/x” cuando x → ∞, con lo que la integral parece ser divergente. En efecto<br />

lim<br />

x→∞<br />

<br />

arctan ax−arctan x<br />

x<br />

1/x<br />

<br />

<br />

= π = 0<br />

y como ∞ dx<br />

1 x es divergente, también lo es ∞<br />

f(x, a)dx y por ello I(a) (recuérdese que este criterio se ha podido<br />

1<br />

aplicar porque la función subintegral es no negativa). Cuando a = 0<br />

<br />

<br />

arctan ax−arctan x<br />

x<br />

lim<br />

= π/2 = 0<br />

x→∞ 1/x<br />

con lo que, razonando de forma análoga, la integral diverge.<br />

Cuando a > 0 el numerador de f tiende a cero cuando x → ∞, con lo que intuímos que quizás f(x, a) se comporte<br />

”como 1/x2 ” cuando x → ∞. Veamos,<br />

lim<br />

x→∞<br />

<br />

arctan ax−arctan x<br />

x<br />

1/x 2<br />

<br />

<br />

=<br />

= |a − 1|<br />

<br />

<br />

<br />

lim<br />

x→∞<br />

a<br />

arctan ax − arctan x<br />

1/x<br />

finito<br />

<br />

<br />

<br />

L′ <br />

<br />

Hôpital <br />

= <br />

lim<br />

x→∞<br />

a<br />

1+a 2 x 2 − 1<br />

1+x 2<br />

− 1<br />

x 2<br />

y como ∞ dx<br />

1 x2 es convergente, I(a) es convergente.<br />

Resumiendo, I(a) converge (y además lo hace absolutamente) si y sólo si a > 0.<br />

Obsérvese que, salvo para el caso trivial a = 1, no es posible aplicar el criterio de Dirichlet para asegurar la<br />

convergencia de la integral. En efecto, aunque 1/x es C1 (0, ∞), es estrictamente decreciente y tiende a cero, la función<br />

arctan ax − arctan x no cumple que la primitiva<br />

x<br />

F (x) := (arctan at − arctan t)dt<br />

0<br />

esté acotada. En efecto, ya hemos razonado que para a = 1 la función subintegral conserva el signo (positivo si a > 1<br />

y negativo si a < 1). Por ello F (x) está acotada si y sólo si la integral ∞<br />

(arctan at − arctan t)dt es convergente.<br />

0<br />

Puesto que en este caso la convergencia es equivalente a la convergencia absoluta estudiamos la convergencia de<br />

∞<br />

|arctan at − arctan t| dt que es divergente pues si a < 0,<br />

0<br />

y ∞<br />

1dx es divergente. Si a > 0, con a = 1,<br />

0<br />

|arctan ax − arctan x|<br />

lim<br />

x→∞ 1/x<br />

=<br />

=<br />

|arctan ax − arctan x|<br />

lim<br />

= π = 0<br />

x→∞ 1<br />

<br />

<br />

<br />

<br />

=<br />

<br />

<br />

<br />

lim<br />

<br />

arctan ax − arctan x<br />

<br />

x→∞ 1/x L′ <br />

<br />

Hôpital <br />

= <br />

lim<br />

x<br />

x→∞<br />

2 (a − a2 )x2 + a − 1 <br />

(1 + a2x2 ) (1 + x2 <br />

<br />

<br />

<br />

) =<br />

<br />

<br />

<br />

a − a<br />

<br />

2<br />

a2 <br />

<br />

<br />

=<br />

<br />

<br />

<br />

1 − a<br />

<br />

a finito y no nulo<br />

Como ∞ 1<br />

0 xdx es divergente, la integral ∞<br />

(arctan at − arctan t)dt también es divergente. Como conclusión, salvo en<br />

0<br />

el caso a = 1, la función F (x) no está acotada.<br />

2) Sea [c, d] un intervalo compacto contenido en (0, ∞). Sabemos que para poder garantizar que ∞<br />

f(x, a)dx<br />

0<br />

sea derivable para a ∈ [c, d], se debe cumplir que f(x, a) y ∂f<br />

∂a (x, a) sean continuas en [c, d] × (0, ∞) y que la integral<br />

∞ ∂f<br />

0 ∂a (x, a)dx converja uniformemente en [c, d]. Claramente lo primero se cumple. Estudiemos entonces la convergencia<br />

uniforme de ∞<br />

∞<br />

∂f<br />

1<br />

(x, a)dx =<br />

∂a 1 + a2 dx<br />

x2 Claramente, si a ∈ [c, d] ⊂ (0, ∞) <br />

0<br />

1<br />

1 + a 2 x 2<br />

0<br />

<br />

<br />

<br />

=<br />

1<br />

1 + a2 ≤<br />

x2 1<br />

1 + c 2 x 2


y, como c = 0, ∞ 1<br />

0 1+c2x2 dx es convergente, por lo que por el criterio de Weierstrass, la integral ∞ ∂f<br />

0 ∂a (x, a)dx converge<br />

uniformemente en todo intervalo [c, d] con c > 0. Por consiguiente, I(a) es derivable en (0, ∞) y además, su derivada<br />

vale<br />

I ′ ∞<br />

(a) =<br />

3) Integrando en la expresión de I ′ (a) se tiene<br />

I ′ ∞<br />

(a) =<br />

y por ello<br />

0<br />

0<br />

1<br />

1 + a2 1<br />

dx =<br />

x2 a<br />

1<br />

1 + a2 dx , a > 0<br />

x2 a>0 π<br />

arctan(ax) |x=∞ x=0 =<br />

2a<br />

I(a) = π<br />

π<br />

log |a| + C = log a + C, a > 0<br />

2 2<br />

Imponiendo que I(1) = 0 se obtiene C = 0 luego<br />

a∈(0,d]<br />

I(a) = π<br />

log a, a > 0<br />

2<br />

, a > 0<br />

A continuación haremos algunas observaciones (que no son necesarias para la resolución del problema) sobre la<br />

convergencia puntual y la convergencia uniforme de la integral H(a) := ∞ 1<br />

0 1+a2x2 dx. Hemos visto en el apartado<br />

anterior que H(a) converge uniformemente en todo intervalo de la forma [c, d] con c > 0, de donde que se deduce que<br />

H(a) converge puntualmente en (0, ∞) (en efecto, para todo a ∈ (0, ∞) existen 0 < c < d tales que a ∈ [c, d] y como<br />

H converge uniformemente en [c, d] también lo hará puntualmente y por ello H(a) será convergente). Sin embargo,<br />

ahora veremos que H(a) no converge uniformemente en (0, ∞) y de hecho ni siquiera lo hace en (0, d] para ningún<br />

d > 0. En efecto, de la definición de convergencia uniforme se tiene que H(a) converge uniformemente en (0, d] si<br />

<br />

<br />

T<br />

1<br />

sup <br />

1 + a2 <br />

<br />

<br />

dx − H(a)<br />

x2 <br />

→<br />

T →∞ 0<br />

Veamos<br />

sup<br />

a∈(0,d]<br />

<br />

<br />

T<br />

<br />

<br />

<br />

0<br />

1<br />

1 + a2 <br />

<br />

<br />

dx − H(a)<br />

x2 <br />

<br />

0<br />

<br />

<br />

= sup <br />

1<br />

arctan(ax) |x=T x=0 −<br />

a∈(0,d] a π<br />

<br />

<br />

<br />

2a<br />

=<br />

<br />

<br />

<br />

= sup <br />

1<br />

π <br />

arctan(aT ) − <br />

1<br />

<br />

<br />

a∈(0,d] a 2a<br />

= sup arctan(aT ) −<br />

a∈(0,d] a<br />

π<br />

<br />

<br />

= ∞<br />

2<br />

que, obviamente, no tiende a cero cuando T → ∞. Obsérvese que si en vez de trabajar en (0, d) se trabaja en [c, d],<br />

con c > 0, se tiene<br />

<br />

<br />

T<br />

1<br />

sup <br />

a∈[c,d] 1 + a2 <br />

<br />

1<br />

<br />

<br />

dx − H(a)<br />

x2 = sup arctan(aT ) −<br />

a∈[c,d] a<br />

π<br />

<br />

<br />

=<br />

2<br />

1<br />

<br />

<br />

arctan(cT ) −<br />

c<br />

π<br />

<br />

<br />

→ 0<br />

2 T →∞<br />

0<br />

lo que muestra, utilizando la definición, que hay convergencia uniforme en todo intervalo [c, d], con c > 0 (esto ya lo<br />

sabíamos del apartado 2)<br />

4) Sea J(a) la integral del enunciado. Parece razonable intentar relacionar J(a) con I(a). En vista de que el<br />

intervalo de integración es el mismo y de la expresión de las funciones subintegrales, podemos intentar integrar por<br />

partes en la expresión de I tomando u(x) = arctan(ax) − arctan x y v ′ (x) = 1<br />

x . Entonces<br />

con lo que<br />

u ′ (x) =<br />

a<br />

1 + a2 1<br />

−<br />

x2 1 + x2 = a − 1 + (a − a2 )x2 (1 + a2x2 ) (1 + x2 )<br />

v(x) = log x<br />

= (1 − a)<br />

ax 2 − 1<br />

(1 + a 2 x 2 ) (1 + x 2 )<br />

I(a) = (arctan ax − arctan x) log x | ∞ <br />

∞ 2 ax − 1 log x<br />

0 −(1 − a)<br />

0 (1 + a2x2 ) (1 + x2 dx = (a − 1)J(a)<br />

)


para a > 0, donde se ha usado que<br />

lim (arctan ax − arctan x) log x = lim<br />

x→0 + x→0 +<br />

arctan ax − arctan x L<br />

1<br />

log x<br />

′ Hôpital<br />

=<br />

= lim<br />

x→0 +<br />

(1 − a)<br />

lim (arctan ax − arctan x) log x = lim<br />

x→∞ x→∞<br />

Por tanto, para a > 0, a = 1<br />

ax 2 −1<br />

(1+a2x2 )(1+x2 )<br />

−1<br />

log2 1<br />

x x<br />

= (a − 1) lim<br />

x→0 + x log2 x = 0<br />

arctan ax − arctan x<br />

1<br />

log x<br />

= lim<br />

x→0 +<br />

(1 − a) ax2 − 1 x log 2 x<br />

(1 + a2x2 ) (1 + x2 =<br />

)<br />

L ′ Hôpital<br />

=<br />

(1 − a)<br />

= lim<br />

x→∞<br />

ax2 − 1 x log 2 x<br />

(1 + a2x2 ) (1 + x2 = 0<br />

)<br />

J(a) = 1 π log a<br />

I(a) =<br />

a − 1 2 a − 1


AMPLIACIÓN DE CÁLCULO (Curso 2007/2008) Convocatoria de junio 17.06.08<br />

NOMBRE ......................... APELLIDOS ...........................................<br />

PROBLEMA 2 (3 puntos)<br />

Número de matrícula. .......................<br />

Dada una constante a > 0, se considera el arco Γ de la curva llamada astroide contenido en el<br />

primer cuadrante. Las ecuaciones paramétricas de esa curva son:<br />

x = a cos 3 t<br />

Se pide:<br />

y = a sen 3 t .<br />

(1) Hallar el área del dominio D limitado por el arco Γ y los ejes coordenados. Hallar también la<br />

longitud del arco Γ. (1 punto)<br />

(2) Hallar el centroide (xD, yD) del dominio D. Obviamente, por simetría, se verifica xD =<br />

yD. (1 punto)<br />

(3) Hallar el centroide (x0, y0) del arco Γ. Obviamente, por simetría, se verifica x0 = y0.(1 punto)<br />

Respuesta: Se entregará esta hoja y, a lo sumo, una adicional.<br />

Elevando ambas ecuaciones a potencia 2/3 y sumando miembro a miembro, se obtiene la ecuación<br />

de la astroide en coordenadas cartesianas:<br />

La gráfica de Γ puede verse en la Figura 1.<br />

(1) Se tiene<br />

<br />

Área(D) =<br />

D<br />

a<br />

=<br />

0<br />

π/2<br />

=<br />

0<br />

= 3a 2<br />

π/2<br />

y<br />

0<br />

x 2/3 + y 2/3 = a 2/3 .<br />

a<br />

D<br />

Γ<br />

x<br />

a<br />

Figura 1: Arco de astroide<br />

a<br />

dxdy =<br />

0<br />

<br />

a 2/3 − x 2/3 3/2<br />

2/3<br />

(a −x2/3 3/2<br />

)<br />

dx<br />

0<br />

dx<br />

⎛<br />

⎜<br />

⎝<br />

dy<br />

t = arc sen 1/3 x<br />

a<br />

x = a sen3 t<br />

dx = 3a sen2 t costdt<br />

<br />

a 2/3 − a 2/3 sen 2 t 3/2<br />

3a sen 2 t costdt<br />

0<br />

sen 2 t cos 4 t dt = 3a2<br />

2 B<br />

<br />

3<br />

2<br />

1<br />

<br />

5<br />

,<br />

2<br />

⎞<br />

⎟<br />


Como<br />

= 3a2<br />

2<br />

Por otra parte, la longitud es<br />

se tiene<br />

de donde<br />

(2) Se tiene<br />

de donde<br />

(3) Se tiene<br />

<br />

3 5<br />

Γ Γ<br />

2 2<br />

Γ(4)<br />

π/2<br />

l(Γ) =<br />

0<br />

= 3a2<br />

2<br />

1<br />

2 Γ<br />

<br />

1 3<br />

2 2<br />

6<br />

<br />

x ′ (t) 2 + y ′ (t) 2 dt .<br />

x ′ (t) = −3a cos 2 t sen t<br />

y ′ (t) = 3a sen 2 t cost,<br />

1<br />

2 Γ<br />

<br />

1<br />

2<br />

x ′ (t) 2 + y ′ (t) 2 = 9a 2 (cos 4 t sen 2 t + sen 4 t cos 2 t)<br />

= 9a 2 sen 2 t cos 2 t(cos 2 t + sen 2 t) ,<br />

<br />

π/2<br />

2 π/2<br />

sen t<br />

l(Γ) = 3a sen t costdt = 3a<br />

0<br />

2<br />

<br />

Área(D) · xD =<br />

=<br />

=<br />

=<br />

D<br />

a<br />

0<br />

a<br />

0<br />

π/2<br />

0<br />

xdxdy<br />

= 3a 3<br />

π/2<br />

= 3a3<br />

2 B<br />

= 3a3<br />

2<br />

= 8a3<br />

105 ,<br />

(a2/3−x2/3 ) 3/2<br />

xdx dy<br />

0<br />

x(a 2/3 − x 2/3 ) 3/2 dx<br />

<br />

0<br />

= 3a<br />

2 .<br />

x = a sen 3 t<br />

= 3πa2<br />

32 .<br />

dx = 3a sen 2 t costdt<br />

a sen 3 t(a 2/3 − a 2/3 sen 2 t) 3/2 3a sen 2 t costdt<br />

sen<br />

0<br />

5 t cos 4 t dt<br />

<br />

3, 5<br />

2<br />

<br />

5<br />

Γ(3)Γ<br />

<br />

2<br />

11<br />

Γ<br />

2<br />

8a<br />

xD =<br />

3 256a<br />

=<br />

105 Área(D) 315π .<br />

<br />

l(Γ)x0 =<br />

Γ<br />

xds<br />

2


de donde<br />

π/2<br />

=<br />

= a<br />

a cos<br />

0<br />

3 <br />

t x ′ (t) 2 + y ′ (t) 2 dt<br />

π/2<br />

= 3a 2<br />

= 3a 2<br />

0<br />

π/2<br />

0<br />

<br />

= 3a2<br />

5 ,<br />

cos 3 t 3a sen t costdt<br />

− cos5 t<br />

5<br />

cos 4 t sen t dt<br />

π/2<br />

x0 = 3a2 2a<br />

=<br />

5l(Γ) 5 .<br />

3<br />

0


AMPLIACIÓN DE CÁLCULO (Curso 2007/2008) Convocatoria de junio 17.06.08<br />

NOMBRE . . . . . . . . . . . . . . . . . . . . . . . . . APELLIDOS . . . . . . . . . . . . . . . . . . . . . . . . . . . . . . . . . . . . . . . . . . .<br />

PROBLEMA 3 (3 puntos)<br />

Se considera el campo vectorial definido por:<br />

F(x, y, z) =<br />

donde f(x, y) es una función de clase C 1 en R 2 .<br />

Número de matrícula. . . . . . . . . . . . . . . . . . . . . . . .<br />

(z , x z , f(x, y))<br />

(x 2 + 8y 2 + z 2 ) 3/2<br />

1) Determínese la función f(x, y) para que el campo F sea solenoidal. Para ese valor de f(x, y)<br />

obténgase un potencial vector de F cuya tercera componente sea nula, indicando razonadamente<br />

la mayor región de R 3 en la que F lo admite.<br />

(1 punto)<br />

2) Sea Γ la porción de curva de ecuación cartesiana x 2 + 4y 2 = 1, z = x que está situada en el<br />

primer octante. Sea Σ1 la superficie generada por los segmentos que parten del punto (1, 0, 1)<br />

y se apoyan en Γ. Calcúlese el flujo del campo F, determinado en el primer apartado, a través<br />

de la cara de Σ1 cuya normal tiene tercera componente positiva.<br />

(1 punto)<br />

3) Sea Σ2 una superficie regular y cerrada cualquiera que no pasa por el origen. Calcúlense razonadamente<br />

todos los valores posibles que puede tomar el flujo del campo F, determinado en el<br />

primer apartado, a través de la cara exterior de la mencionada superficie. (1 punto)<br />

Respuesta:<br />

1) El campo F(x, y, z) = (L(x, y, z), M(x, y, z), N(x, y, z)) está definido en R 3 \{(0, 0, 0)}. En ese<br />

conjunto su divergencia es:<br />

∇ · F = ∂L<br />

∂x<br />

+ ∂M<br />

∂y<br />

+ ∂N<br />

∂z<br />

= − 3z(x + 8xy + f(x, y))<br />

(x 2 + 4y 2 + z 2 ) 5/2<br />

Por definición F es solenoidal si su divergencia es nula, lo que ocurre si y solamente si<br />

f(x, y) = −x(1 + 8y) ,<br />

que es la función f(x, y) que se pide. Para esta expresión de f se tiene un campo solenoidal en el<br />

conjunto R 3 \{(0, 0, 0)} que no es estrellado con respecto a ningún punto. En esta situación no se<br />

puede asegurar la existencia de un potencial vector para F en este conjunto. Lo que sí podemos<br />

afirmar es que F admite un potencial vector en cualquier conjunto estrellado que esté contenido en<br />

R 3 \{(0, 0, 0)}.<br />

Para calcularlo suponemos que el potencial vector es un campo de la forma<br />

G(x, y, z) = (P (x, y, z), Q(x, y, z), 0) ,<br />

.


que debe cumplir<br />

⎧<br />

−<br />

⎪⎨<br />

∇ × G = F ⇐⇒<br />

∂Q<br />

∂z =<br />

∂P<br />

∂z =<br />

⎪⎩<br />

∂Q<br />

∂x<br />

z<br />

(x2 + 8y2 + z2 ,<br />

) 3/2<br />

xz<br />

(x2 + 8y2 + z2 ,<br />

) 3/2<br />

− ∂P<br />

∂y =<br />

−x(1 + 8y)<br />

(x2 + 8y2 + z2 .<br />

) 3/2<br />

Integrando en la primera de las igualdades con respecto a z, se obtiene:<br />

Q(x, y, z) =<br />

1<br />

(x 2 + 8y 2 + z 2 ) 1/2 + C1(x, y) ,<br />

donde C1(x, y) es una función arbitraria que solamente puede depender de las variables x e y.<br />

Análogamente, integrando en la segunda de las igualdades con respecto a z, se obtiene:<br />

P (x, y, z) = −<br />

x<br />

(x 2 + 8y 2 + z 2 ) 1/2 + C2(x, y) ,<br />

donde C2(x, y) es otra función arbitraria que solamente puede depender de las variables x e y.<br />

Sustituyendo estas expresiones de P y Q en la tercera igualdad, resulta la siguiente condición que<br />

deben cumplir las funciones C1 y C2:<br />

∂C2(x, y)<br />

∂x<br />

− ∂C1(x, y)<br />

∂y<br />

Una elección posible (la más sencilla) es C1(x, y) = C2(x, y) = 0 con lo que un potencial vector del<br />

campo F es:<br />

<br />

x<br />

G(x, y, z) = −<br />

(x2 + 8y2 + z2 ) 1/2 ,<br />

1<br />

(x2 + 8y2 + z2 ) 1/2<br />

<br />

, 0<br />

y cualquier otro potencial vector de F se diferenciará del dado en un campo de gradientes.<br />

Ahora bien, el campo G está definido y es de clase C 1 en todo el conjunto R 3 \{(0, 0, 0)} y, además,<br />

en todo ese conjunto también cumple ∇ × G = F. Esto significa (por definición de potencial vector)<br />

que el campo G es un potencial vector de F en R 3 \{(0, 0, 0)}. (1 punto)<br />

2) Teniendo en cuenta que la superficie Σ1 no es cerrada y puesto que el campo F que se<br />

considera es solenoidal y admite un potencial vector, se dispone de varias posibilidades a la hora de<br />

calcular su flujo.<br />

Una de ellas consiste en utilizar la propia definición de flujo, para lo que es necesario obtener una<br />

parametrización de la superficie Σ1 que está formada por la el haz de segmentos que unen el punto<br />

(1, 0, 1) con el arco Γ descrito en el enunciado. Unas ecuaciones paramétricas de esta superficie son:<br />

x(u, θ) = 1 + u(cos θ − 1) , y(u, θ) = u<br />

sen θ<br />

2<br />

= 0 .<br />

, z(u, θ) = 1 + u(cos θ − 1) ,<br />

donde u ∈ [0, 1] y θ ∈ [0, π/2], dado que la curva Γ está contenida en el primer octante y se ha<br />

parametrizado mediante las ecuaciones<br />

x(θ) = cos θ , y(θ) =<br />

sen θ<br />

2<br />

Ahora bien, el valor que toma el campo sobre esta superficie es<br />

, z(θ) = cos θ , θ ∈ [0, π/2] .<br />

1<br />

F(x(u, θ), y(u, θ), z(u, θ)) =<br />

(2 + 4(cos θ − 1)u + 4(1 − cos θ)u2 ×<br />

) 3/2<br />

(1 + u (cos θ − 1) , (1 + u (cos θ − 1)) 2 , − (1 + u (cos θ − 1)) (1 + 4 u sen θ) <br />

.


lo que desaconseja utilizar este procedimiento dada la complejidad de las integrales que sería necesario<br />

calcular.<br />

Otra posibilidad pasa por hacer uso del teorema de Stokes. Dado que Σ1 y su curva borde<br />

cumplen las condiciones de regularidad a trozos adecuadas y ambas están contenidas en un abierto<br />

de R3 \{(0, 0, 0)} (que siempre existe) en el que se cumple ∇ × G = F (con G de clase C∞ ), el<br />

teorema de Stokes permite escribir:<br />

<br />

<br />

<br />

F · dσ = ∇ × G · dσ = G · ds ,<br />

Σ1<br />

Σ1<br />

dónde Λ representa el borde de Σ1 orientado de tal forma que su proyección sobre le plano z = 0<br />

se recorra en sentido positivo (lo que se corresponde con el sentido que dicta “la regla del sacacorchos”<br />

aplicada teniendo en cuenta que es la normal con tercera componente positiva la que se debe<br />

considerar para Σ1).<br />

La curva cerrada Λ está formada por el arco Γ (una de cuyas parametrizaciones se ha dado<br />

anteriormente) y los segmentos que unen el punto (1, 0, 1) con los extremos de Γ que son el mismo<br />

(1, 0, 1) y el punto (0, 1/2, 0). Así pues, Λ está formada por la unión yuxtapuesta del arco Γ, recorrido<br />

desde el punto (1, 0, 1) al (0, 1/2, 0), y el segmento S que une (0, 1/2, 0) con (1, 0, 1) recorrido en ese<br />

mismo sentido. Una parametrización de S es:<br />

Por tanto: <br />

Σ1<br />

x(v) = v , y(v) =<br />

<br />

F · dσ =<br />

Σ1<br />

1 − v<br />

2<br />

<br />

∇ × G · dσ =<br />

, z(v) = v , v ∈ [0, 1] .<br />

Λ<br />

G · ds =<br />

Calculamos ahora las dos circulaciones del potencial vector G.<br />

<br />

• G · ds.<br />

Γ<br />

El valor que el potencial vector G toma sobre el arco Γ es:<br />

G(x(θ), y(θ), z(θ)) =<br />

<br />

−<br />

cos θ<br />

√ 2 ,<br />

<br />

Λ<br />

Γ<br />

<br />

G · ds +<br />

<br />

1<br />

√ , 0<br />

2<br />

y la tangente al arco correspondiente a la parametrización considerada es:<br />

Se tiene entonces,<br />

luego:<br />

T(θ) =<br />

<br />

− sen θ ,<br />

cos θ<br />

2<br />

G(x(θ), y(θ), z(θ)) · T(θ) =<br />

, − sen θ<br />

<br />

cos θ<br />

2 √ (1 + 2 sen θ),<br />

2<br />

.<br />

S<br />

G · ds .<br />

<br />

π/2<br />

G · ds = G(x(θ), y(θ), z(θ)) · T(θ) dθ =<br />

Γ<br />

0<br />

1<br />

2 √ π/2<br />

<br />

π/2<br />

cos θ dθ + 2 sen θ cos θ dθ<br />

2 0<br />

0<br />

= 1<br />

2 √ <br />

sen θ|<br />

2<br />

π/2<br />

0 + sen2 <br />

<br />

θ<br />

π/2<br />

<br />

=<br />

0<br />

1<br />

√ .<br />

2<br />

<br />

• G · ds.<br />

S<br />

El valor que el potencial vector G toma sobre el segmento S es:<br />

G(x(v), y(v), z(v)) =<br />

1<br />

(−v , 1 , 0) =<br />

2 − 4 v + 4v2 1<br />

<br />

(2v − 1) 2 (−v , 1 , 0)<br />

+ 1


y la tangente al segmento que se corresponde con la parametrización considerada es:<br />

Se tiene entonces,<br />

luego:<br />

T(v) =<br />

<br />

1 , − 1<br />

2<br />

<br />

, 1 .<br />

1<br />

G(x(v), y(v), z(v)) · T(v) = − <br />

(2v − 1) 2 <br />

2v + 1<br />

+ 1 2<br />

<br />

1<br />

G · ds = G(x(v), y(v), z(v)) · T(v) dv = −<br />

S<br />

0<br />

1<br />

1 2v + 1<br />

<br />

2 0 (2v − 1) 2 + 1 dv<br />

= − 1<br />

1 4v − 2<br />

<br />

4 0 (2v − 1) 2 1 1<br />

dv − <br />

+ 1 0 (2v − 1) 2 + 1 dv<br />

= − 1 <br />

(2v − 1)<br />

4<br />

2 <br />

1<br />

+ 1<br />

−<br />

0<br />

1<br />

ArgSh(1)<br />

dt = − ArgSh(1) = − log<br />

2 −ArgSh(1)<br />

<br />

1 + √ 2 <br />

.<br />

donde la primera de las integrales que aparecen en la segunda línea es inmediata y la segunda se<br />

calcula mediante el cambio de variable: 2v − 1 = Sh(t).<br />

Finalmente, el flujo pedido es:<br />

<br />

Σ1<br />

<br />

F·dσ =<br />

Σ1<br />

<br />

∇×G·dσ =<br />

Λ<br />

G·ds =<br />

<br />

Γ<br />

<br />

G·ds + G·ds =<br />

S<br />

1<br />

√ −ArgSh(1) =<br />

2 1<br />

√ −log<br />

2 <br />

1 + √ 2 <br />

.<br />

(1 punto)<br />

3) Supuesto que la superficie Σ2 no pasa por el origen, solamente existen dos situaciones posibles:<br />

A) El recinto acotado V ⊂ R3 limitado por Σ2 no contiene el origen.<br />

En tal caso, el campo F es solenoidal en V con lo que el teorema de Gauss, según el cual<br />

<br />

<br />

F · dσ = ∇ · F dxdydz<br />

Σ2+<br />

nos permite concluir que el flujo es nulo, siendo Σ + 2 la cara exterior de Σ2.<br />

B) El recinto acotado V ⊂ R 3 limitado por Σ2 sí contiene el origen.<br />

En tal caso y cualquiera que sea la superficie regular Σ2, siempre se puede encontrar una superficie<br />

elipsoidal, S, de ecuación cartesiana<br />

x 2 + 8y 2 + z 2 = ε 2<br />

tal que el elipsoide acotado, E, que limita contiene el origen y, tanto S como E están contenidos en<br />

V . Es decir, se tiene (0, 0, 0) ∈ E ⊂ V ⊂ R3 .<br />

Ahora bien, el conjunto V \E tiene por frontera las superficies Σ2 y S y, además, V \E no contiene<br />

el origen, con lo que el campo F es solenoidal en él. Aplicando de nuevo el teorema de Gauss se obtiene:<br />

<br />

<br />

<br />

<br />

<br />

Σ +<br />

2<br />

F · dσ +<br />

S −<br />

F · dσ =<br />

V \E<br />

V<br />

∇ · F dxdydz = 0 =⇒<br />

Σ +<br />

2<br />

F · dσ =<br />

S +<br />

F · dσ ,<br />

donde, al igual que antes, Σ + 2 es la cara exterior de Σ2, S − es la cara interior de S y S + es la cara<br />

exterior de S.<br />

Así pues, cualquiera que la sea la superficie regular Σ2, calcular el flujo pedido consiste en calcularlo<br />

a través de la cara exterior de la superficie elipsoidal mencionada.


A continuación se detallan tres posibles formas de proceder:<br />

a) Sabemos que<br />

<br />

Σ +<br />

2<br />

(z, xz, −x(1 + 8y))<br />

(x2 + 8y2 + z2 dσ =<br />

3/2<br />

)<br />

<br />

S +<br />

(z, xz, −x(1 + 8y)) 1<br />

dσ = 3/2 ε3 <br />

(x 2 + 8y 2 + z 2 )<br />

S +<br />

(z, xz, −x(1 + 8y)) dσ<br />

pues x 2 + 8y 2 + z 2 = ε 2 sobre S. Ahora, como el campo (z, xz, −x(1 + 8y)) es de clase 1 en R 3 y<br />

además es solenoidal, se puede aplicar Gauss y obtener que el flujo pedido es cero. Obsérvese que,<br />

como era de esperar, el flujo no depende del parámetro ε.<br />

b) Otra forma de razonar, sin aplicar una segunda vez el teorema de Gauss, es utilizar que un<br />

vector normal a la superficie cerrada S es el gradiente de la ecuación h(x, y, z) = 0 que la describe.<br />

Para S tiene la expresión:<br />

N(x, y, z) = (2x , 16y , 2z) ,<br />

y, en este caso, tiene la dirección y sentido de la normal exterior.<br />

Resulta entonces que el campo F es tangente a la superficie elipsoidal en cada punto, ya que:<br />

F(x, y, z) · N(x, y, z) =<br />

=<br />

(z , x z , −x(1 + 8y))<br />

· (2x , 16y , 2z)<br />

(x 2 + 8y 2 + z 2 ) 3/2<br />

1<br />

(x2 + 8y2 + z2 (2xz + 16xyz − 2xz(1 + 8y)) = 0 ,<br />

) 3/2<br />

con lo que también en este caso B) el flujo pedido es cero.<br />

c) Otra posibilidad es proceder utilizando una parametrización P(u, v) = {x(u, v), y(u, v), z(u, v)}<br />

de S; por ejemplo, la dada por las ecuaciones:<br />

x(u, v) = ε cos u sen v , y(u, v) = ε<br />

2 √ sen u sen v , z(u, v) = ε cos v , u ∈ [0, 2π] , v ∈ [0, π] .<br />

2<br />

La normal exterior es:<br />

∂P<br />

∂v<br />

× ∂P<br />

∂u =<br />

ε 2<br />

2 √ 2 cos u sen2 v , ε 2 sin u sin 2 v ,<br />

y tras algunas operaciones se comprueba nuevamente que:<br />

F(x(u, v), y(u, v), z(u, v)) ·<br />

∂P<br />

∂v<br />

ε 2<br />

2 √ 2<br />

<br />

∂P<br />

× = 0 ,<br />

∂u<br />

cos v sen v<br />

lo que de nuevo conduce a la conclusión de que el flujo en este caso B) también es cero y, como era<br />

de esperar, no depende del parámetro ε.<br />

(1 punto)


AMPLIACIÓN DE CÁLCULO (Curso 2007/2008) Convocatoria de septiembre 02.09.08<br />

NOMBRE . . . . . . . . . . . . . . . . . . . . . . . . . APELLIDOS . . . . . . . . . . . . . . . . . . . . . . . . . . . . . . . . . . . . . . . . . . .<br />

PROBLEMA 1 (4 puntos)<br />

Número de matrícula. . . . . . . . . . . . . . . . . . . . . . . .<br />

• 1. (1.25 ptos.) Sea p un número real. Estudiar con todo rigor la convergencia y la convergencia<br />

absoluta de<br />

∞<br />

senx<br />

Hp := dx<br />

xp Para resolver este apartado se pueden utilizar los siguientes resultados:<br />

– (a) ∞<br />

1<br />

– (b) ∞<br />

1<br />

– (c) 1<br />

0<br />

senx<br />

x p dx diverge si p ≤ 0<br />

|senx|<br />

x p dx converge si p > 1<br />

|sen x<br />

1−x|<br />

x(1−x)<br />

dx diverge<br />

0<br />

• 2. (1.5 ptos.) Sea n un entero positivo, sea Mn el recinto acotado del primer cuadrante de R 2<br />

limitado por las curvas de ecuaciones<br />

y considérese la integral<br />

donde β es un número real.<br />

<br />

In :=<br />

xy = 1<br />

xy = 2<br />

y = nx<br />

y = 1<br />

n x<br />

Mn<br />

xβ+2 sen(y<br />

yβ x )dxdy<br />

Se pide expresar In en términos de una integral simple y estudiar para qué valores de β el límite<br />

limn→∞ In existe y es finito.<br />

• 3. (1.25 ptos.) Estudiar razonadamente la convergencia de la integral<br />

donde<br />

<br />

J =<br />

M<br />

xβ+2 sen(y<br />

yβ x )dxdy<br />

M = {(x, y) : 1 ≤ xy ≤ 2 ; x, y ≥ 0}<br />

Respuesta: Se entregará esta hoja y, a lo sumo, una adicional.<br />

1. La integral Hp es impropia porque el intervalo de integración es no acotado y, para ciertos<br />

valores de p, porque la función subintegral no está acotada en el entorno de x = 0. Por ello Hp<br />

converge (absolutamente) si y sólo si convergen (absolutamente) las dos integrales<br />

∞<br />

Ap :=<br />

1<br />

senx<br />

xp dx y Bp<br />

1<br />

:=<br />

0<br />

senx<br />

dx<br />

xp


Por (a) Ap diverge si p ≤ 0. Además Ap converge para p > 0 por el criterio de Abel-Dirichlet. En<br />

efecto, para p > 0 la función 1/x p es C 1 [1, ∞) estrictamente decreciente y tiende a 0 cuando x → ∞,<br />

mientras que la función sen(x) tiene primitiva acotada (es decir, − cos x es una función acotada).<br />

En cuanto a la convergencia absoluta de Ap, por (b) sabemos que Ap converge absolutamente si<br />

p > 1. Por otro lado de (c) se tiene, haciendo el cambio de variable t = x<br />

1−x<br />

es inyectivo en (0, 1))<br />

<br />

1 x sen ∞<br />

1−x<br />

|sent|<br />

dx = · · · = dt<br />

x(1 − x) t<br />

0<br />

0<br />

, es decir, x = t<br />

t+1 (que<br />

donde la última integral sólo es impropia por ser no acotado el intervalo de integración. Como la<br />

primera integral diverge deducimos que Ap no converge absolutamente para p = 1. Para p ≤ 0, Ap<br />

no converge por lo que no puede converger absolutamente. Queda por estudiar el caso p ∈ (0, 1). Si<br />

p ∈ (0, 1) podemos acotar<br />

0 ≤ |senx|<br />

x<br />

≤ |senx|<br />

x p , ∀x > 1<br />

y como ∞ |senx|<br />

1 x dx diverge también lo hace ∞ |senx|<br />

1 xp dx con p ∈ (0, 1). En definitiva Ap converge<br />

⇔ p > 0, y Ap converge absolutamente ⇔ p > 1.<br />

Por otro lado, puesto que senx/xp es no negativa en (0, 1), Bp converge si y sólo si converge<br />

absolutamente. Además podemos aplicar el criterio del cociente<br />

lim<br />

x→0 +<br />

senx<br />

x p<br />

1<br />

x p−1<br />

= lim<br />

x→0 +<br />

senx<br />

x<br />

y por tanto Bp converge (y además lo hace absolutamente) si y sólo si p − 1 < 1, es decir, si y sólo<br />

si p < 2.<br />

En definitiva, Hp converge si y sólo si p ∈ (0, 2) y Hp converge absolutamente si y sólo si p ∈ (1, 2)<br />

2. In es una integral doble en sentido propio. Haciendo el cambio<br />

(que es inyectivo en Mn) o bien<br />

para el que el jacobiano queda<br />

xy = u<br />

y<br />

= v<br />

x<br />

x = u 1/2 v −1/2<br />

y = u 1/2 v 1/2<br />

|Jφ(u, v)| = 1<br />

2 |v|<br />

y teniendo en cuenta que φ −1 (Mn) es el rectángulo [1, 2] × [ 1 , n] se obtiene<br />

n<br />

Ahora<br />

In = 1<br />

<br />

2 [1,2]×[ 1<br />

n ,n]<br />

= 1<br />

2 <br />

u<br />

1<br />

senvdudv = udu<br />

vβ+2 2 1<br />

n<br />

1<br />

n<br />

lim<br />

n→∞ In = 3<br />

∞<br />

4 0<br />

y por ello el límite existe y es finito cuando la integral<br />

∞<br />

senv<br />

dv<br />

vβ+2 0<br />

senv<br />

dv<br />

vβ+2 <br />

senv<br />

dv =<br />

vβ+2 3<br />

n<br />

4 1<br />

n<br />

senv<br />

dv<br />

vβ+2


converge, es decir, usando el apartado 1, cuando<br />

o bien<br />

0 < β + 2 < 2<br />

−2 < β < 0<br />

3. J es una integral doble impropia tanto porque el dominio de integración M está no acotado<br />

como porque la función subintegral puede no estar acotada en el dominio. Por ello J será convergente<br />

si y sólo si el límite<br />

<br />

lim<br />

n→∞<br />

Bn<br />

xβ+2 sen(y<br />

yβ x )dxdy<br />

existe, es finito y es independiente de la sucesión básica de integración Bn que se considere para J.<br />

La función subintegral no conserva el signo, por lo que el límite puede, en principio, depender de la<br />

sucesión básica utilizada. Puesto que en R n con n ≥ 2 la convergencia de una integral es equivalente<br />

a la convergencia absoluta de la misma, se tiene que J converge si y sólo si lo hace la integral<br />

<br />

K :=<br />

M<br />

x β+2<br />

y β<br />

<br />

<br />

sen( y<br />

x )<br />

<br />

<br />

dxdy<br />

Como ahora la función subintegral es no negativa y claramente Mn es una sucesión básica para la<br />

integración en M, se tiene que K es convergente si y sólo si existe y es finito<br />

<br />

x<br />

lim<br />

n→∞<br />

Mn<br />

β+2<br />

yβ <br />

<br />

sen( y<br />

x )<br />

<br />

<br />

dxdy<br />

Haciendo el mismo cambio del apartado anterior se obtiene<br />

<br />

x<br />

lim<br />

n→∞<br />

Mn<br />

β+2<br />

yβ <br />

<br />

sen( y<br />

x )<br />

<br />

<br />

dxdy = 3<br />

8<br />

y por ello, usando el apartado 1, K converge si y sólo si<br />

o bien<br />

En definitiva, J converge si y sólo si −1 < β < 0.<br />

1 < β + 2 < 2<br />

−1 < β < 0<br />

∞<br />

0<br />

|senv|<br />

dv<br />

vβ+2


AMPLIACIÓN DE CÁLCULO (Curso 2007-08) Convocatoria de septiembre 02.09.08<br />

PROBLEMA 2 (3 puntos)<br />

En el plano R 2 se considera la región D del primer cuadrante encerrada por las parábolas de ecuaciones<br />

x 2 = y, x = y 2 ; sea Σ la superficie de revolución que se obtiene al girar la curva frontera de D alrededor<br />

del eje OX.<br />

1. Calcular el centroide de D. (1, 25 puntos)<br />

2. Sea F(x, y, z) = (2xy − x 2 , x − y 2 , 2xz + z). Calcular el flujo del campo F a través de la<br />

superficie Σ orientada según el vector normal saliente. (1, 75 puntos)<br />

Respuesta:<br />

1. Los puntos de intersección de ambas parábolas son (0, 0) y (1, 1), así que la región puede<br />

describirse como:<br />

D = {(x, y) ∈ R 2 / 0 ≤ x ≤ 1, x 2 ≤ y ≤ √ x}<br />

Dado que D es simétrica respecto de la bisectriz del primer cuadrante su centroide cg(D) =<br />

(xg, yg) está situado sobre la bisectriz –es decir, xg = yg– y su área es el doble de la parte D ∗<br />

situada debajo de la bisectriz, esto es:<br />

Por tanto,<br />

A(D) = 2A(D ∗ <br />

) = 2<br />

D ∗ = {(x, y) ∈ R 2 / 0 ≤ x ≤ 1, x 2 ≤ y ≤ x}<br />

D ∗<br />

dx dy =<br />

1 x<br />

Calculemos la componente xg del centroide:<br />

<br />

<br />

1 √ <br />

x<br />

A(D)xg = x dx dy = x dy dx =<br />

D<br />

0<br />

x 2<br />

0<br />

x 2<br />

1<br />

dy dx = 2 (x − x<br />

0<br />

2 ) dx = 1<br />

3 .<br />

1<br />

Se concluye que el centroide de D es cg(D) = 9<br />

(1, 1).<br />

20<br />

0<br />

<br />

x 3/2 − x 3<br />

dx = 2 1<br />

−<br />

5 4<br />

2. Dado que la superficie es cerrada no está de más calcular la divergencia del campo:<br />

div F(x, y, z) = 2y − 2x − 2y + 2x + 1 = 1.<br />

= 3<br />

20 .<br />

Como la divergencia es constante, el teorema de Gauss –que puede aplicarse ya que F es<br />

un campo C∞ y Σ es unión de dos superficies regulares– nos permite calcular la integral de<br />

superficie (con orientación saliente) como el volumen del sólido Ω de frontera Σ:<br />

<br />

<br />

F = div F(x, y, z) dxdydz = dxdydz = V (Ω).<br />

Σ<br />

Ω<br />

Ahora bien, al ser el sólido de revolución, su volumen puede calcularse aplicando el teorema de<br />

Guldin:<br />

El volumen engendrado por un recinto plano que gira alrededor de un eje no secante y situado<br />

en su mismo plano es igual al producto del área del recinto por la longitud de la circunferencia<br />

descrita por su centro de gravedad.<br />

En nuestro caso, si giramos D alrededor de OX el radio de la circunferencia descrita por cg es<br />

la componente yg, luego:<br />

<br />

Σ<br />

F = V (Ω) = 2πygA(D) = 2π 9 1<br />

20 3<br />

Ω<br />

= 3π<br />

10 .


Si no se recuerda el teorema de Guldin, para el cálculo de un volumen de revolución de un arco<br />

y = f(x), x ∈ [a, b] alrededor del eje OX podemos también usar la conocida fórmula<br />

b<br />

V = π f<br />

a<br />

2 .<br />

En nuestro caso, al volumen engendrado por el arco y = √ x hay que restarle el correspondiente a<br />

y = x 2 :<br />

1<br />

V (Ω) = π (x − x<br />

0<br />

4 ) dx = 3π<br />

10 .<br />

Como alternativa al segundo apartado, si no se aplica el teorema de Gauss puede hacerse directamente<br />

la integral de superficie; dado que la curva que gira está formada por dos arcos regulares, la superficie<br />

habrá de parametrizarse a través de dos funciones. Llamemos Σ1 a la superficie generada al girar<br />

el arco y = √ x, x ∈ [0, 1] alrededor del eje OX y Σ2 a la relativa al arco y = x 2 . Entonces una<br />

parametrización de Σ1 se obtiene premultiplicando un punto genérico del arco (t, √ t, 0), t ∈ [0, 1]<br />

por la correspondiente matriz de giro de ángulo θ, θ ∈ [0, 2π]:<br />

σ1(θ, t) =<br />

⎛<br />

⎜<br />

⎝<br />

x(θ, t)<br />

y(θ, t)<br />

z(θ, t)<br />

⎞<br />

⎟<br />

⎠ =<br />

⎛<br />

⎜<br />

⎝<br />

1 0 0<br />

0 cos θ − sen θ<br />

0 sen θ cos θ<br />

El vector normal asociado a esta parametrización es<br />

N1(θ, t) = ∂σ1(θ, t)<br />

∂θ<br />

× ∂σ1(θ, t)<br />

∂t<br />

⎞ ⎛<br />

⎟ ⎜<br />

⎠ ⎝<br />

t<br />

√ t<br />

0<br />

⎞<br />

⎟<br />

⎠ =<br />

⎛<br />

⎜<br />

⎝<br />

t<br />

√ t cos θ<br />

√ t sen θ<br />

= (− 1<br />

2 , √ t cos θ, √ t sen θ)<br />

Si tomamos, por ejemplo, θ = 0, t = 1/2 se tiene N1(0, 1/2) = (−1/2, 1/2, 0) que apunta al exterior<br />

de Σ por lo que orienta adecuadamente la superficie. Análogamente, para Σ2 se tiene<br />

σ2(t, θ) =<br />

⎛<br />

⎜<br />

⎝<br />

x(t, θ)<br />

y(t, θ)<br />

z(t, θ)<br />

⎞<br />

⎟<br />

⎠ =<br />

⎛<br />

⎜<br />

⎝<br />

1 0 0<br />

0 cos θ − sen θ<br />

0 sen θ cos θ<br />

⎞ ⎛<br />

⎟ ⎜<br />

⎠ ⎝<br />

t<br />

t 2<br />

0<br />

⎞<br />

⎟<br />

⎠ =<br />

⎛<br />

⎜<br />

⎝<br />

t<br />

t 2 cos θ<br />

t 2 sen θ<br />

Obsérvese que hemos cambiado el orden de los parámetros para obtener la orientación adecuada;<br />

ahora el vector normal es<br />

La integral de superficie pedida es:<br />

N2(t, θ) = (2t 3 , −t 2 cos θ, −t 2 sen θ)<br />

<br />

Σ<br />

<br />

F =<br />

Σ1<br />

<br />

F +<br />

Hagamos con cierto detalle la integral del campo a través de Σ1:<br />

<br />

Σ1<br />

<br />

F =<br />

[0,2π]×[0,1]<br />

Al sustituir y hacer el producto escalar se obtiene<br />

Σ2<br />

F.<br />

F(σ1(θ, t)) · N1(θ, t) dθ dt.<br />

F(σ1(θ, t)) · N1(θ, t) = (2t √ t cos θ − t 2 , t − t cos 2 θ, (2t + 1) sen θ) · (− 1<br />

2 , √ t cos θ, √ t sen θ)<br />

⎞<br />

⎟<br />

⎠ .<br />

⎞<br />

⎟<br />


e integrando:<br />

<br />

Σ1<br />

<br />

F =<br />

= t2<br />

2 − t√ t cos 3 θ + (2t + 1)t sen 2 θ,<br />

[0,2π]×[0,1]<br />

t 2<br />

2 + (2t + 1)t sen2 θ<br />

De forma totalmente análoga (no nos detenemos en los detalles),<br />

Y, para terminar:<br />

<br />

Σ2<br />

<br />

F =<br />

<br />

Σ<br />

[0,1]×[0,2π]<br />

<br />

F =<br />

Σ1<br />

<br />

dθ dt = 3<br />

2 π.<br />

F(σ2(t, θ)) · N2(t, θ) dt dθ = − 6<br />

5 π.<br />

<br />

F +<br />

Σ2<br />

F = 3 6 3<br />

π − π =<br />

2 5 10 π.<br />

Gabriela Sansigre.


AMPLIACIÓN DE CÁLCULO (Curso 2007/2008) Convocatoria de septiembre 02.09.08<br />

NOMBRE ......................... APELLIDOS ...........................................<br />

PROBLEMA 3 (3 puntos)<br />

Se considera el campo vectorial<br />

F =<br />

definido en R 2 \{0}, siendo r := √ x 2 + y 2 . Se pide:<br />

Número de matrícula. .......................<br />

<br />

yf(r) − x2y xy2<br />

, xf(r) −<br />

r3 r3 <br />

(1) Determinar una función f : R\{0} → R que haga al campo F conservativo en algún<br />

dominio. (1,5 puntos)<br />

(2) Para la función f hallada en el apartado anterior, calcular un potencial escalar de F.<br />

(1,5 puntos)<br />

Respuesta: Se entregará esta hoja y, a lo sumo, una adicional.<br />

(1) Sea<br />

Derivando,<br />

Igualando las derivadas:<br />

P := yf(r) − x2 y<br />

r 3<br />

∂P<br />

∂y = f(r) + y2 f ′ (r)<br />

, Q := xf(r) − xy2<br />

r 3<br />

r − x2r3 − 3rx2y2 r6 ∂Q<br />

∂x = f(r) + x2 f ′ (r)<br />

r − y2r3 − 3rx2y2 r6 y 2 f ′ (r)<br />

r − x2r2 − 3x2y2 r5 = x 2 f ′ (r)<br />

r − y2r2 − 3x2y2 r5 ⇒ y 2 r 2 f ′ (r) − x 2 = x 2 r 2 f ′ (r) − y 2<br />

⇒ r 2 (y 2 − x 2 )f ′ (r) = x 2 − y 2<br />

⇒ f ′ (r) = − 1<br />

r 2<br />

⇒ f(r) = 1<br />

+ k .<br />

r<br />

Podemos elegir k = 0, y tenemos<br />

f(r) = 1<br />

r .<br />

(2) En todo punto de la forma (1, y), el campo F está bien definido. Un potencial escalar de F<br />

será:<br />

<br />

x<br />

U(x, y) = Q(1, y) dy + P(s, y) ds<br />

1<br />

<br />

=<br />

<br />

1<br />

√<br />

1 + y2 −<br />

y2 <br />

x y<br />

dy + √<br />

s2 + y2 −<br />

s2 <br />

y<br />

ds .<br />

(1 + y2 ) 3/2<br />

<br />

=I<br />

<br />

1<br />

.<br />

1 (s2 + y2 ) 3/2<br />

<br />

=J<br />

<br />

.


Se tiene<br />

de donde<br />

ya que<br />

Por otra parte,<br />

de donde<br />

Por tanto,<br />

x<br />

J =<br />

1<br />

√ 1 + y 2 −<br />

<br />

I =<br />

<br />

=<br />

y<br />

√ s 2 + y 2 −<br />

1<br />

x/y<br />

=<br />

=<br />

1/y<br />

y2 (1 + y2 ) 3/2 = 1 + y2 − y2 (1 + y2 =<br />

) 3/2<br />

dy<br />

(1 + y2 ) 3/2<br />

ch η dη<br />

(1 + sh 2 <br />

=<br />

η) 3/2<br />

(cambio y = sh η)<br />

dη<br />

ch 2 η<br />

th 2 η = sh2 η<br />

ch 2 η = sh2 η<br />

1 + sh 2 η<br />

s 2 y<br />

y3 ds<br />

(s2 + y2 x<br />

=<br />

) 3/2<br />

1<br />

= th η =<br />

(s2 + y2 ) 3/2 = y s2 + y2 − s2 (s2 + y2 )<br />

y dz<br />

(1 + z2 <br />

= y<br />

) 3/2<br />

xy<br />

√ x 2 + y 2 −<br />

y<br />

√ 1 + y 2 .<br />

ds<br />

⎡ ⎤<br />

2<br />

⎣1<br />

s<br />

+ ⎦<br />

y<br />

z<br />

√ 1 + z 2<br />

U(x, y) = I + J =<br />

3/2<br />

y2<br />

= .<br />

1 + y2 3/2 =<br />

<br />

1<br />

(1 + y2 ,<br />

) 3/2<br />

y<br />

√ 1 + y 2<br />

y3 (s2 + y2 ,<br />

) 3/2<br />

cambio s<br />

y<br />

x<br />

y<br />

= z<br />

<br />

1<br />

y<br />

z=x/y = y <br />

z=1/y<br />

1 + x2<br />

y2 − y <br />

1 + 1<br />

y2 2<br />

xy<br />

√ x 2 + y 2<br />

= xy<br />

r .


AMPLIACIÓN DE CÁLCULO (Curso 2008/2009) Convocatoria de Febrero 03.02.09<br />

NOMBRE ......................... APELLIDOS ...........................................<br />

PROBLEMA 1 (3 puntos)<br />

Número de matrícula. .......................<br />

Siendo a > 0 una constante, se considera el arco Γ de la curva de ecuación<br />

contenido en el primer cuadrante (Figura 1).<br />

Se pide:<br />

y<br />

x 4 − 2ax 3 + 4a 2 y 2 = 0<br />

Γ<br />

D<br />

0 V<br />

Figura 1: Curva x 4 − 2ax 3 + 4a 2 y 2 = 0 en el primer cuadrante<br />

(1) Hallar el área del dominio D limitado por la curva Γ y el eje OX. (1 punto)<br />

(2) Si (x0, y0) designa el centroide del domino D, hallar la ordenada y0 (no se pide x0 por razón<br />

de brevedad). (1 punto)<br />

(3) Dado el campo vectorial<br />

<br />

F =<br />

x 3 log(1 + y 2 ) + e x sen y + x , ax3y 1 + y2 + ex <br />

cos y + 5xy<br />

calcular la circulación de F a lo largo del arco Γ recorrido desde el punto V hasta el origen O. Para<br />

responder a esta pregunta se sugiere utilizar, de manera adecuada, el teorema de Green. (1 punto)<br />

Respuesta: Se entregará esta hoja y, a lo sumo, una adicional.<br />

(1) Los puntos de corte de Γ con el eje OX se obtienen haciendo y = 0 en la ecuación de la curva,<br />

y son:<br />

O(0, 0) y V (2a, 0) .<br />

Despejando y en la ecuación de la curva, resulta<br />

Como<br />

El área de D será<br />

Área(D) =<br />

2a<br />

haremos en la integral el cambio de variable<br />

t = arc sen<br />

0<br />

x − a<br />

a<br />

y = 1 <br />

(2a − x)x<br />

2a<br />

3 .<br />

1 <br />

(2a − x)x<br />

2a<br />

3 dx = 1<br />

2a<br />

x<br />

2a 0<br />

√ 2ax − x2 dx.<br />

2ax − x 2 = a 2 − (x − a) 2 ,<br />

⇒ x − a = a sen t ⇒ dx = a costdt<br />

1<br />

x<br />

,


de modo que<br />

x = 0 ⇒ t = − π<br />

2<br />

, x = 2a ⇒ t = π<br />

2 ,<br />

Área(D) = 1<br />

π/2<br />

a(1 + sen t)<br />

2a −π/2<br />

√ a2 − a2 sen2 t a costdt<br />

= a2<br />

π/2<br />

(1 + sen t) cos<br />

2 −π/2<br />

2 t dt<br />

= a2<br />

π/2<br />

(cos<br />

2 −π/2<br />

2 t + sen t cos 2 t) dt<br />

= a 2<br />

π/2<br />

cos 2 t dt<br />

0<br />

= πa2<br />

4 .<br />

(2) La ordenada del centroide de D es<br />

Calculamos, pues, la integral<br />

Por tanto,<br />

y0 =<br />

<br />

J : =<br />

=<br />

1<br />

Área(D)<br />

D<br />

2a<br />

0<br />

y dxdy<br />

<br />

D<br />

y dxdy .<br />

<br />

√<br />

1<br />

(2a−x)x3 2a<br />

dx<br />

0<br />

2a<br />

y dy<br />

= 1<br />

8a2 (2a − x)x<br />

0<br />

3 dx<br />

= 1<br />

8a2 2a<br />

(2ax<br />

0<br />

3 − x 4 ) dx<br />

= 1<br />

8a2 <br />

4 x=2a<br />

ax x5<br />

−<br />

2 5 x=0<br />

= 1<br />

8a2 <br />

4 5 2 a<br />

2 − 25a5 <br />

5<br />

= 2a 3<br />

<br />

1 2<br />

−<br />

2 5<br />

= a3<br />

5 .<br />

J<br />

y0 =<br />

Área(D)<br />

= 4a<br />

5π .<br />

(3) Sea C := Γ ∪ OV la curva obtenida cerrando el arco Γ con el segmento OV . Aplicaremos la<br />

fórmula de Green-Riemann al campo F en el dominio D.<br />

Si hacemos F = (P, Q), se tiene<br />

∂Q<br />

∂x<br />

− ∂P<br />

∂y = 3ax2 y<br />

1 + y 2 + ex cos y + 5y − 2x3 y<br />

1 + y 2 − ex cos y<br />

= x2 y(3a − 2x)<br />

1 + y 2<br />

+ 5y .<br />

2


Por la fórmula de Green-Riemann,<br />

<br />

<br />

C<br />

<br />

∂Q ∂P<br />

F · dr = − dxdy<br />

D ∂x ∂y<br />

<br />

x<br />

=<br />

D<br />

2y(3a − 2x)<br />

1 + y2 <br />

dxdy +5<br />

<br />

:=I<br />

y dxdy ,<br />

<br />

D<br />

<br />

=J<br />

<br />

donde la segunda integral coincide con la integral J del apartado (2).<br />

Calculamos la primera:<br />

Sea<br />

Entonces,<br />

de donde<br />

I =<br />

=<br />

2a<br />

0<br />

= 1<br />

2<br />

2a<br />

0<br />

2a<br />

0<br />

x 2 <br />

√<br />

1<br />

(2a−x)x3 2a<br />

(3a − 2x) dx<br />

0<br />

x 2 <br />

1<br />

(3a − 2x)<br />

(3ax 2 − 2x 3 ) log<br />

f(x) := 1 +<br />

2 log(1 + y2 )<br />

(2a − x)x3<br />

4a 2<br />

f ′ (x) = 6ax2 − 4x 3<br />

4a 2<br />

I = 1<br />

2<br />

y<br />

dy<br />

1 + y2 y= 1<br />

2a<br />

y=0<br />

√ (2a−x)x 3<br />

<br />

1 + 1<br />

<br />

(2a − x)x3<br />

4a2 = 1 + 2ax3 − x 4<br />

4a 2<br />

= 1<br />

2a 2 (3ax2 − 2x 3 ) ,<br />

2a<br />

2a<br />

0<br />

2 f ′ (x) log f(x) dx<br />

= a 2 [f(x) log f(x) − f(x)] x=2a<br />

x=0<br />

= 0<br />

ya que f(0) = f(2a) = 1.<br />

Por tanto, <br />

F<br />

· dr = I + 5J = a<br />

C<br />

3 .<br />

Por otra parte, <br />

Parametrizamos el segmento −→<br />

OV ,<br />

de donde<br />

<br />

−→<br />

OV<br />

luego <br />

F<br />

· dr =<br />

Γ<br />

<br />

F<br />

· dr =<br />

C<br />

<br />

F · dr =<br />

<br />

C<br />

<br />

Γ<br />

x(t) ≡ t<br />

y(t) ≡ 0 ⇒<br />

=<br />

F · dr +<br />

<br />

−→<br />

OV<br />

F · dr .<br />

x ′ (t) ≡ 1<br />

y ′ (t) ≡ 0 ,<br />

.<br />

dx<br />

dx.<br />

2a<br />

[P(t, 0)x<br />

0<br />

′ (t) + Q(t, 0)y ′ (t)] dt<br />

<br />

2a 2 2a<br />

t<br />

0<br />

<br />

F · dr −<br />

t dt =<br />

−→<br />

OV<br />

2<br />

0<br />

= 2a 2 ,<br />

F · dr = a 3 − 2a 2 = a 2 (a − 2) .<br />

3


Nota 1. El cálculo de x0, que no se pide en el problema, es también sencillo utilizando el mismo<br />

cambio de variable x − a = a sen t del apartado (1). Sabemos que<br />

Calculamos la integral doble<br />

de donde<br />

<br />

H : =<br />

=<br />

D<br />

2a<br />

0<br />

= 1<br />

2a<br />

= 1<br />

2a<br />

= a3<br />

2<br />

= a3<br />

2<br />

= a3<br />

2<br />

= a3<br />

2<br />

= a3<br />

2<br />

x0 =<br />

xdxdy<br />

1<br />

Área(D)<br />

<br />

√<br />

1<br />

(2a−x)x3 2a<br />

xdx<br />

0<br />

2a<br />

0<br />

π/2<br />

= 5πa3<br />

16 ,<br />

<br />

x 2√ 2ax − x 2 dx<br />

D<br />

dy<br />

xdxdy .<br />

(a + a sen t) 2√ a 2 − a 2 sen 2 t a costdt<br />

−π/2<br />

π/2<br />

(1 + sen t)<br />

−π/2<br />

2 cos 2 t dt<br />

π/2<br />

(cos<br />

−π/2<br />

2 t + 2 sen t cos 2 t + sen 2 t cos 2 t) dt<br />

<br />

π π/2<br />

+ 2 sen<br />

0<br />

2 t cos 2 <br />

t dt<br />

2<br />

<br />

π 3<br />

+ B<br />

2 2<br />

<br />

π π<br />

+<br />

2 8<br />

<br />

3<br />

,<br />

2<br />

x0 = 5πa3 πa2 5a<br />

: =<br />

16 4 4 .<br />

Nota 2. Claramente, la curva completa es simétrica respecto del eje OX, y su gráfica puede verse<br />

en la Figura 2.<br />

y<br />

0<br />

Figura 2: Curva x 4 − 2ax 3 + 4a 2 y 2 = 0 completa<br />

4<br />

x


AMPLIACIÓN DE CÁLCULO (Curso 2008/2009) Convocatoria de febrero 03.02.09<br />

NOMBRE . . . . . . . . . . . . . . . . . . . . . . . . . APELLIDOS . . . . . . . . . . . . . . . . . . . . . . . . . . . . . . . . . . . . . . . . . . .<br />

PROBLEMA 2 (4 puntos)<br />

Nota: Los apartados de este problema son independientes.<br />

Número de matrícula. . . . . . . . . . . . . . . . . . . . . . . .<br />

En R 3 se considera la circunferencia definida por la intersección del plano x = 0 con la superficie<br />

cilíndrica y 2 + (z + 1) 2 = 4.<br />

1. Sea Γ1 el arco de esta circunferencia limitado por los planos z = 1 y z = −1, situado en el<br />

semiespacio y ≥ 0. Sobre Γ1 se considera definida una densidad lineal de masa que en cada<br />

punto es proporcional al cuadrado de la distancia del punto al eje OY . Calcúlese el momento<br />

de inercia de Γ1 con respecto al eje OZ. (1 punto.)<br />

2. Sea Γ2 el arco de la mencionada circunferencia situado en el semiespacio y ≥ 0 y limitado por<br />

los planos z = 0 y z = 1. Sea Σ la superficie de revolución que se obtiene al girar Γ2 alrededor<br />

del eje OZ. Calcúlese la masa de Σ si sobre ella se considera definida una densidad superficial<br />

de masa que en cada punto es proporcional a la distancia al origen del plano tangente a la<br />

superficie en ese punto. (2 puntos.)<br />

3. Calcular la masa del dominio acotado Ω ⊂ R 3 limitado por la superficie Σ definida en el<br />

apartado anterior y por el plano z = 0, sabiendo que la densidad de masa por unidad de<br />

volumen en cada punto de Ω es proporcional a la distancia de ese punto al plano XY .<br />

(1 punto.)<br />

Respuesta: Se entregará esta hoja y, a lo sumo, una adicional.<br />

1. La curva Γ1 está contenida en el plano x = 0, por tanto las distancias de uno de sus puntos<br />

(x, y, z) a los ejes OY y OZ son, respectivamente, dOY (x, y, z) = |z|, dOZ(x, y, z) = |y| . La densidad<br />

lineal de masa, DΓ1, definida sobre Γ1 tiene entonces la expresión<br />

DΓ1(x, y, z) := [dOY (x, y, z)] 2 = k |z| 2 ,<br />

donde k > 0 es la constante de proporcionalidad.<br />

Teniendo estas expresiones en cuenta, resulta que (por definición) el momento de inercia de Γ1<br />

con respecto al eje OZ viene dado por la siguiente integral curvilínea:<br />

<br />

IOZ(Γ1) = d 2 <br />

OZ DΓ1 ds = k |y| 2 |z| 2 <br />

ds = k y 2 z 2 ds .<br />

Γ1<br />

NOTA: No es conveniente aplicar en este problema el teorema de Steiner, dada la ausencia de<br />

simetrías en el arco.<br />

Para calcular la integral en cuestión, sea r(t) = (x(t), y(t), z(t)), t ∈ [a, b], una parametrización<br />

de Γ1, en la que siempre es x(t) = 0 puesto que el arco está contenido en el plano Y Z. Entonces:<br />

<br />

IOZ(Γ1) = k<br />

Γ1<br />

y 2 z 2 ds = k<br />

b<br />

a<br />

Γ1<br />

y(t) 2 z(t) 2 r ′ (t) dt = k<br />

b<br />

a<br />

Γ1<br />

y(t) 2 <br />

2<br />

z(t) [y ′ (t)] 2 + [z ′ (t)] 2 dt .<br />

Resta entonces parametrizar Γ1 que es un arco de la circunferencia de ecuación cartesiana:<br />

definido por las condiciones: y ≥ 0, −1 ≤ z ≤ 1.<br />

x = 0 , y 2 + (z + 1) 2 = 4 ,<br />

1


Para ello existen varias alternativas. Dos de las más razonables se exponen a continuación.<br />

A) Coordenadas polares en el plano Y Z.<br />

Haciendo una traslación, que lleve el punto (0, 0, −1) al origen, y utilizado entonces coordenadas<br />

polares en el plano Y Z, unas ecuaciones paramétricas para el arco Γ1 son:<br />

x(θ) = 0 , y(θ) = 2 cos θ , z(θ) = −1 + 2 sen θ , θ ∈ [0, π/2] ,<br />

donde los valores θ = 0 y θ = π/2 corresponden a los puntos de corte de la circunferencia con los<br />

planos z = −1 y z = 1, respectivamente, y se ha tenido en cuenta que el arco se encuentra en el<br />

semiespacio y ≥ 0.<br />

Se tiene entonces:<br />

y, por tanto:<br />

<br />

IOZ(Γ1) = k<br />

r ′ (θ) = (x ′ (θ), y ′ (θ), z ′ (θ)) = (0 , −2 sen θ , 2 cos θ) =⇒ r ′ (θ) = 2 ,<br />

y 2 z 2 ds = k<br />

Γ1<br />

π/2<br />

π/2<br />

0<br />

y(θ) 2 z(θ) 2 r ′ (θ) dθ = 8k<br />

π/2<br />

π/2<br />

0<br />

cos 2 θ (2 sen θ − 1) 2 dθ<br />

π/2<br />

= 32k cos<br />

0<br />

2 θ sen 2 θ dθ − 32k cos<br />

0<br />

2 θ sen θ dθ + 8k cos<br />

0<br />

2 θ dθ<br />

=<br />

<br />

3 3<br />

3<br />

3 1<br />

k 16 B , − 16 B , 1 + 4 B , = k 2π −<br />

2 2<br />

2 2 2<br />

32<br />

=<br />

<br />

+ 2π<br />

3<br />

<br />

k 4π − 32<br />

<br />

,<br />

3<br />

donde B(p, q) = 2 π/2<br />

0 cos2p−1 θ sen2q−1 θ dθ es la función Beta de Euler. Para calcular sus valores se<br />

ha utilizado su relación con la función Gamma de Euler: B(p, q) = Γ(p)Γ(q)/Γ(p + q) junto con los<br />

valores Γ(n) = (n − 1)Γ(n − 1), Γ(1) = 1 y Γ(1/2) = √ π.<br />

B) Parametrización cartesiana.<br />

Si se utiliza como parámetro la coordenada y, unas ecuaciones paramétricas para el arco Γ1 son:<br />

x(t) = 0 , y(t) = t , z(t) = −1 + √ 4 − t 2 , t ∈ [0, 2] ,<br />

donde, los valores t = 0 y t = 2 corresponden a los valores de la coordenada, y, en los puntos de<br />

corte del arco con los planos z = 1 y z = −1, respectivamente.<br />

Se tiene entonces:<br />

y, por tanto:<br />

<br />

IOZ(Γ1) = k<br />

r ′ (t) = (x ′ (t), y ′ (t), z ′ (t)) =<br />

Γ1<br />

y 2 z 2 ds = k<br />

2<br />

0<br />

<br />

0 , 1 , −<br />

<br />

t<br />

√ =⇒ r<br />

4 − t2 ′ (θ) =<br />

y(t) 2 z(t) 2 r ′ (t) dt = 2k<br />

2<br />

0<br />

2<br />

√ 4 − t 2 ,<br />

t 2 √ 2 1<br />

4 − t2 − 1 √ dt .<br />

4 − t2 Para calcular esta integral, efectuamos el cambio de variable t = 2 sen θ, dt = 2 cos θdθ, t = 0 ⇒ θ = 0,<br />

t = 2 ⇒ θ = π/2. De ello resulta:<br />

2<br />

π/2<br />

IOZ(Γ1) = 2k t<br />

0<br />

2 √ 2 1<br />

4 − t2 − 1 √ dt = 8k<br />

4 − t2 0<br />

<br />

3 3<br />

= k 16 B , − 16 B 1 ,<br />

2 2<br />

3<br />

<br />

1 3<br />

+ 4 B ,<br />

2 2 2<br />

sen 2 θ (2 cos θ − 1) 2 dθ<br />

<br />

= k 4π − 32<br />

<br />

.<br />

3<br />

donde se han vuelto a utilizar las funciones Gamma y Beta de Euler en forma totalmente análoga al<br />

caso anterior. (1 punto).<br />

Nota: También se pude utilizar como parámetro la coordenada z.<br />

2


2. Para obtener la expresión de la densidad definida sobre Σ es necesario determinar la ecuación del<br />

plano a tangente a Σ en cada uno de sus puntos. Si n(x, y, z) representa un vector normal cualquiera<br />

a la superficie en el punto (x, y, z) de la misma, la ecuación cartesiana del plano tangente es:<br />

n(x, y, z) · (X − x, Y − y, Z − z) = 0 ,<br />

donde (X, Y, Z) representan las coordenadas cartesianas de los puntos del plano. La distancia de este<br />

plano al origen y, por tanto, la densidad DΣ(x, y, z) definida sobre Σ, es entonces:<br />

DΣ(x, y, z) = C<br />

n(x, y, z) · (x, y, z)<br />

n(x, y, z)<br />

donde C > 0 es la constante de proporcionalidad.<br />

En el caso que nos ocupa, la superficie de revolución Σ es el casquete (situado en el semiespacio<br />

z ≥ 0) de la esfera de ecuación cartesiana:<br />

x 2 + y 2 + (z + 1) 2 = 4 ,<br />

que se obtendría al hacer girar toda la circunferencia alrededor del eje OZ. Un vector normal al<br />

punto (x, y, z) es entonces<br />

n(x, y, z) = grad <br />

x 2 + y 2 + (z + 1) 2 − 4 <br />

con lo que la expresión de la densidad es:<br />

DΣ(x, y, z) = C<br />

n(x, y, z) · (x, y, z)<br />

n(x, y, z)<br />

2 (x, y, (z + 1)) · (x, y, z)<br />

= C <br />

2 x2 + y2 + (z + 1) 2<br />

,<br />

= 2 (x, y, (z + 1)) ,<br />

= C x2 + y2 + z(z + 1)<br />

<br />

x2 + y2 + (z + 1)<br />

Ahora bien, dado que la densidad hay que evaluarla sobre Σ y sus puntos (x, y, z) verifican la ecuación<br />

cartesiana de la esfera, se tiene: x2 +y2 <br />

+z(z+1) = 3−z y x2 + y2 + (z + 1) 2 = √ 4 = 2 sobre Σ, con<br />

lo que, finalmente, la densidad definida sobre la superficie que debemos considerar es, simplemente:<br />

DΣ(x, y, z) = C<br />

3 − z<br />

2<br />

La masa de Σ viene dada entonces por la integral de superficie:<br />

<br />

M(Σ) =<br />

Σ<br />

DΣ(x, y, z) dσ = C<br />

.<br />

<br />

Σ<br />

3 − z<br />

2<br />

(1 punto).<br />

Para calcularla es necesario parametrizar la superficie que se origina al hacer girar alrededor del<br />

eje OZ el arco de ecuación cartesiana:<br />

x = 0 , z =<br />

<br />

4 − y 2 − 1 , 0 ≤ y ≤ √ 3 ,<br />

donde el valor y = √ 3 corresponde al corte del arco con el plano z = 0.<br />

Una de las posibles parametrizaciones, s(r, θ) = (x(r, θ), y(r, θ), z(r, θ)), de Σ viene dada por:<br />

x(r, θ) = r cos θ , y(r, θ) = r sin θ , z(r, θ) = √ 4 − r 2 − 1 , θ ∈ [0, 2π] , r ∈ [0, √ 3] .<br />

Resulta entonces:<br />

M(Σ) =<br />

<br />

Σ<br />

<br />

DΣ dσ =<br />

[0,2π]×[0, √ 3]<br />

3<br />

dσ .<br />

DΣ(s(r, θ)) n(r, θ) dθ dr<br />

2 .


= C<br />

[0,2π]×[0, √ 3]<br />

<br />

3 − z(r, θ)<br />

n(r, θ) dθ dr ,<br />

2<br />

donde n(r, θ) es el vector normal asociado a la parametrización; es decir:<br />

n(r, θ) =<br />

∂s(r, θ)<br />

∂r<br />

∂s(r, θ)<br />

×<br />

∂θ =<br />

<br />

2 r cos θ<br />

√<br />

4 − r2 , r2 <br />

sen θ<br />

√ , r<br />

4 − r2 =⇒ n(r, θ) =<br />

Por otra parte, la densidad evaluada sobre la superficie tiene la expresión:<br />

DΣ(s(r, θ)) = C<br />

<br />

3 − z(r, θ)<br />

con lo que la integral doble que hay que calcular es<br />

2<br />

= C 4 − √ 4 − r 2<br />

2<br />

<br />

M(Σ) = C<br />

[0,2π]×[0, √ r <br />

√ 4 −<br />

3] 4 − r2 √ 4 − r2 <br />

dr dθ<br />

<br />

= 4C<br />

[0,2π]×[0, √ <br />

r<br />

√ dr dθ −<br />

3] 4 − r2 [0,2π]×[0, √ r dr dθ<br />

3]<br />

⎡<br />

= 2πC ⎣−4 √ 4 − r2 √<br />

√<br />

⎤<br />

3 r2 3<br />

<br />

− ⎦ = 5π C .<br />

0 2 <br />

0<br />

3. Unas ecuaciones que describen el dominio Ω ∈ R 3 son:<br />

Ω =<br />

<br />

(x, y, z) ∈ R 3 : 0 ≤ x 2 + y 2 <br />

≤ 3 , 0 ≤ z ≤ 4 − x2 − y2 <br />

− 1 .<br />

,<br />

2r<br />

√ 4 − r 2 .<br />

(1 punto).<br />

donde el círculo 0 ≤ x 2 + y 2 ≤ 3 es la proyección de Ω sobre el plano XY y z debe variar entre este<br />

plano y Σ.<br />

Puesto que la densidad definida en Ω es proporcional en cada uno de sus puntos a la distancia al<br />

plano XY , es decir: DΩ(x, y, z) = A|z|, su masa viene dada por la integral triple:<br />

<br />

M(Ω) =<br />

Ω<br />

A|z| dx dy dz ,<br />

donde A > 0 es la constante de proporcionalidad.<br />

Para calcular su valor utilizamos coordenadas cilíndricas (r, θ, z):<br />

x = r cos θ , y = r sen θ , z = z , con jacobiano: J = r .<br />

El recinto Ω expresado en estas coordenadas es:<br />

Ω ′ = <br />

(r, θ, z) ∈ R 3 : 0 ≤ θ ≤ 2π , 0 ≤ r ≤ √ 3 , 0 ≤ z ≤ √ 4 − r 2 − 1 <br />

Como Ω está situado en el semiespacio z ≥ 0, se tiene:<br />

<br />

M(Ω) = A<br />

= 2π A<br />

Ω<br />

√ 3<br />

0<br />

<br />

z dx dy dz = A<br />

Ω ′<br />

r z dr dθ dz = A<br />

⎛<br />

r √ 2 4 − r2 − 1 dr = π A ⎝<br />

2<br />

5r2 r4<br />

−<br />

2 4<br />

4<br />

2π<br />

0<br />

<br />

dθ<br />

√ 3<br />

0<br />

2<br />

+<br />

3 (4 − r2 ) 3/2<br />

<br />

r dr<br />

√ 4−r2−1 0<br />

√<br />

⎞<br />

3<br />

<br />

<br />

<br />

0<br />

z dz<br />

⎠ = 7<br />

π A .<br />

12<br />

(1 punto).


AMPLIACIÓN DE CÁLCULO (Curso 2008/2009) Convocatoria de febrero 03.02.09<br />

PROBLEMA 3 (3 puntos)<br />

Sea F un campo vectorial de clase 1 en un abierto Ω de R 3 . Se dice que un campo escalar f : Ω → R<br />

de clase 1 es un factor integrante de F en Ω si rot(f F) = 0. Sea F(x, y, z) = (yz, zx, −xy). Se pide:<br />

1. Calcular un factor integrante f de F que sea solamente función de z, ¿cuál es su dominio de<br />

definición? Calcular asimismo un potencial escalar de f F.<br />

2. Sea Σ la superficie del paraboloide x 2 + 9y 2 = z − 1 contenida en el semiespacio z ≤ 9 y<br />

orientada de forma que en (0, 0, 1) su vector normal es k. Calcular el flujo de rot F a través de<br />

Σ.<br />

3. Sea Γ la curva parametrizada por<br />

ϕ(t) = (2 cos t, 2<br />

π<br />

sen t, 5), t ∈ [0,<br />

3 3 ]<br />

Calcular la circulación de F a lo largo de Γ.<br />

Nota: Los apartados 2. y 3. pueden hacerse directamente, pero se valorará especialmente el hacerlos aplicando 1.<br />

Respuesta:<br />

1. Calculemos en primer lugar el rotacional de F:<br />

<br />

<br />

<br />

<br />

rot F(x, y, z) = <br />

<br />

<br />

i<br />

∂/∂x<br />

yz<br />

j<br />

∂/∂y<br />

zx<br />

<br />

k <br />

<br />

<br />

∂/∂z = (−2x, 2y, 0).<br />

<br />

−xy <br />

Sea f(x, y, z) = h(z) el factor integrante buscado y denotemos G = f F. Recordemos la fórmula<br />

rot G = ∇f × F + f rot F (véase el problema 6.8 (f)). Calculemos el primer sumando:<br />

<br />

<br />

i j k<br />

<br />

∇f × F(x, y, z) = 0 0 h<br />

<br />

<br />

′ <br />

<br />

<br />

<br />

(z) = h<br />

<br />

yz zx −xy <br />

′ (z)(−zx, yz, 0).<br />

Entonces<br />

rot G = 0 ⇐⇒ h ′ (z)(−zx, yz, 0) + h(z)(−2x, 2y, 0) = 0<br />

Suponiendo x e y no nulos e integrando se tiene<br />

⇐⇒ zxh ′ (z) + 2xh(z) = yzh ′ (z) + 2yh(z) = 0.<br />

h ′ (z)<br />

h(z)<br />

= −2<br />

z<br />

c<br />

⇐⇒ h(z) = , c = 0.<br />

z2 La constante c es arbitraria, podemos tomarla igual a 1 por ejemplo, con lo que el factor<br />

integrante queda f(x, y, z) = 1/z 2 y es válido siempre que z sea no nulo; esto es, en cualquier<br />

parte del espacio que no atraviese el plano de ecuación z = 0.<br />

Sea g un potencial escalar de G(x, y, z) = (y/z, x/z, −xy/z 2 ), es decir G = ∇g,<br />

∂g y<br />

xy<br />

= ⇒ g(x, y, z) = + k(y, z)<br />

∂x z z<br />

∂g x x ∂k ∂k<br />

= = + ⇒ = 0<br />

∂y z z ∂y ∂y<br />

∂g<br />

∂k ∂k<br />

= −xy = −xy + ⇒ = 0<br />

∂z z2 z2 ∂z ∂z<br />

Se concluye que la función k es constante; en particular podemos escogerla nula y tomar<br />

g(x, y, z) = xy/z como potencial escalar de G, siempre y cuando estemos en R3 \ {z = 0}.


2. La superficie Σ está contenida en el semiespacio z ≥ 1 que es un conjunto estrellado y F<br />

es de clase 1, así pues podemos aplicar el teorema de Stokes; sea B el borde de Σ orientado<br />

coherentemente; se tiene<br />

<br />

Σ<br />

<br />

rot F =<br />

Ahora bien, la curva B está situada en el plano z = 9, por tanto<br />

Se tiene entonces:<br />

B<br />

F.<br />

G(x, y, z) = 1<br />

F(x, y, z), (x, y, z) ∈ B.<br />

81<br />

<br />

B<br />

<br />

F = 81 G = 81 rot G = 0,<br />

B<br />

Σ<br />

donde se ha aplicado de nuevo el teorema de Stokes, esta vez al campo G.<br />

2. (Razonamientos alternativos, sin utilizar el primer apartado)<br />

(a) Consideremos la superficie D que cierra la superficie Σ en el plano z = 9, esto es<br />

D = {(x, y, z) ∈ R 3 , x 2 + 9y 2 ≤ 8, z = 9}<br />

En virtud del Teorema de Stokes, si un campo de clase 1 es solenoidal en una parte estrellada<br />

de R3 , su integral de superficie depende de un potencial vector y del borde de la superficie, no<br />

de la superficie propiamente dicha. En nuestro caso Σ y D tienen el mismo borde (la elipse<br />

x2 + 9y2 = 8, z = 9), el vector unitario normal a D coherente con la orientación que Σ induce<br />

en su contorno es k, pero resulta que rot F · k = (−2x, 2y, 0) · (0, 0, 1) = 0. Concluimos por<br />

tanto:<br />

<br />

rot F = rot F · k = 0.<br />

Σ<br />

D<br />

El razonamiento anterior puede leerse también en clave del Teorema de Gauss: sea Σ − ∪ D la<br />

superficie frontera del paraboloide sólido P (comprendido entre la superficie del paraboloide y el<br />

plano z = 9), orientada por el vector normal exterior –obsérvese que Σ y Σ − tienen orientación<br />

opuesta–, entonces<br />

<br />

Σ − ∪D<br />

<br />

rot F =<br />

P<br />

<br />

div(rot F) = 0 ⇒<br />

Σ −<br />

<br />

rot F = − rot F = 0.<br />

D<br />

(b) También puede realizarse directamente la integral de superficie, sin tomar en consideración los<br />

teorema de Stokes o Gauss, en tal caso:<br />

Consideremos la parametrización dada por la ecuación implícita H(x, y, z) = x 2 +9y 2 −z+1 = 0<br />

entre z = 1 y z = 9. El vector normal es<br />

∇H<br />

∂H/∂z<br />

= (−2x, −18y, 1),<br />

este vector, al tener la tercera componente positiva, orienta el paraboloide como se nos indica<br />

en el enunciado. El flujo pedido es:<br />

<br />

Σ<br />

<br />

rot F =<br />

x 2 +9y 2 ≤8<br />

<br />

(−2x, 2y, 0) · (−2x, −18y, 1) dx dy =<br />

x 2 +9y 2 ≤8<br />

(4x 2 − 36y 2 ) dx dy


Hacemos cambio de variables a coordenadas elípticas:<br />

y obtenemos:<br />

x = ρ cos t, y = 1<br />

3 ρ sen t, (t, ρ) ∈ [0, 2π] × [0, √ 8] J = 1<br />

3 ρ<br />

<br />

Σ<br />

rot F = 4<br />

<br />

3 [0,2π]×[0, √ ρ<br />

8]<br />

3 (cos 2 t − sen 2 t)dρ dt = 0.<br />

3. Busquemos una representación implícita de la curva Γ (es decir, su traza) eliminando el<br />

parámetro t,<br />

x 2 + 9y 2 = 4, z = 5.<br />

Notamos que Γ está sobre la intersección de Σ con el plano z = 5, por tanto<br />

G(x, y, z) = 1<br />

F(x, y, z), (x, y, z) ∈ Γ.<br />

25<br />

Podemos calcular la integral de línea con ayuda del potencial g obtenido en el primer apartado,<br />

veamos cuáles son los extremos de la curva:<br />

Entonces:<br />

<br />

Γ<br />

ϕ(0) = (2, 0, 5), ϕ(π/3) = (1, √ 3/3, 5)<br />

<br />

F = 25 G = 25 ∇g = 25(g(1,<br />

Γ<br />

Γ<br />

√ √<br />

3<br />

3/3, 5) − g(2, 0, 5)) = 5<br />

3 .<br />

3. (Cálculo directo de la integral de línea)<br />

<br />

Γ<br />

π/3<br />

F = F(ϕ(t)) · ϕ<br />

0<br />

′ (t)dt = 20<br />

π/3<br />

(cos<br />

3 0<br />

2 t − sen 2 t)dt = 20<br />

π/3<br />

cos 2tdt<br />

3 0<br />

= 20<br />

<br />

sen 2tπ/3<br />

<br />

=<br />

3 2<br />

10<br />

√<br />

3<br />

sen(π/3) = 5<br />

3 3 .<br />

0<br />

Gabriela Sansigre.


AMPLIACIÓN DE CÁLCULO (Curso 2008/2009) Convocatoria de Junio 09.06.09<br />

NOMBRE ......................... APELLIDOS ...........................................<br />

PROBLEMA 1 (3 puntos)<br />

Número de matrícula. .......................<br />

Este problema tiene por objeto utilizar el teorema de Green para calcular una conocida integral<br />

impropia, que puede verse más abajo, al final de este enunciado.<br />

Se considera el campo vectorial F de clase C1 definido en R2 por y<br />

⎧<br />

<br />

−xy<br />

⎪⎨<br />

e sen x<br />

, 0<br />

F(x, y) := x<br />

⎪⎩ (1, 0)<br />

si x = 0<br />

si x = 0 .<br />

R<br />

Γ4<br />

Γ3<br />

D<br />

Γ<br />

Γ2<br />

Dado R > 0 arbitrario, sea la curva cerrada Γ = Γ1 ∪ Γ2 ∪ Γ3 ∪ Γ4 de la<br />

Figura. Se pide:<br />

0 Γ1 R x<br />

(1) (0,5 puntos) Mediante una aplicación del teorema de Green, calcular una función f(x, y) tal<br />

que<br />

R R<br />

F · dr = dy f(x, y) dx .<br />

<br />

Γ<br />

0<br />

0<br />

<br />

=:I1(R,y)<br />

<br />

(2) (0,5 puntos) Calcular la integral I1(R, y).<br />

(3) (0,25 puntos) Integrando I1(R, y) respecto de la variable y en el intervalo [0, R], obtener dos<br />

funciones g(R) y h(R, y) tales que<br />

<br />

<br />

Γ<br />

R<br />

F · dr = g(R) − h(R, y) dy .<br />

(4) (0,5 puntos) Hallar la circulación del campo F a lo largo de cada uno de los segmentos Γ1,<br />

Γ2, Γ3 y Γ4, todos ellos con la orientación indicada en la Figura. En algunos casos se obtendrá un<br />

valor numérico y en otros una integral en [0, R] que no es posible calcular en términos de funciones<br />

elementales.<br />

(5) (0,25 puntos) Utilizando, si se desea, la desigualdad |(sen x)/x| < 1 para x > 0, obtener una<br />

función ξ(R) que verifique<br />

<br />

<br />

<br />

<br />

Γ3<br />

<br />

<br />

F · dr<br />

<br />

0<br />

≤ ξ(R) y tal que lím ξ(R) = 0 .<br />

R→+∞<br />

(6) (0,5 puntos) Encontrar una función η(R) que verifique<br />

<br />

<br />

<br />

R <br />

<br />

<br />

h(R, y) dy<br />

0 <br />

≤ η(R) y tal que lím η(R) = 0 .<br />

R→+∞<br />

(7) (0,5 puntos) Como consecuencia inmediata de todo lo anterior, probar la igualdad<br />

∞<br />

0<br />

sen x<br />

x<br />

dx = π<br />

2 .<br />

Este problema está tomado de R. L. Robertson, An improper application of Green’s theorem,<br />

College Math. J. 38 (2007), 142–145.<br />

Respuesta: Se entregará esta hoja y, a lo sumo, una hoja adicional.


(1) Haciendo F = (P, Q), tenemos por los teoremas de Green y de Fubini:<br />

<br />

<br />

Γ<br />

<br />

F · dr = −<br />

<br />

=<br />

=<br />

D<br />

R<br />

0<br />

D<br />

∂P<br />

∂y dxdy<br />

e −xy sen xdxdy<br />

R<br />

dy e −xy sen xdx.<br />

(2) Integrando dos veces por partes:<br />

<br />

u = e−xy ⇒ du = −ye−xy dx<br />

dv = sen xdx ⇒ v = − cosx<br />

I1(R, y) = <br />

−e −xy cosx <br />

x=R R<br />

− y e −xy cosxdx<br />

de donde<br />

(3)<br />

(4) Es claro que<br />

<br />

<br />

Γ<br />

0<br />

x=0<br />

= 1 − e −Ry cosR − y<br />

0<br />

R<br />

e −xy cos xdx<br />

0<br />

<br />

=:I2(R,y)<br />

<br />

(dv = cosxdx ⇒ v = sen x)<br />

R<br />

+ y e −xy sen xdx<br />

I2(R, y) = <br />

e −xy sen x x=R<br />

x=0<br />

= e −Ry sen R + yI1(R, y) ,<br />

I1(R, y) = 1 − e −Ry cosR − y[e −Ry sen R + yI1(R, y)]<br />

<br />

<br />

Γ<br />

⇒ I1(R, y) = 1 − e−Ry cosR − ye −Ry sen R<br />

1 + y 2<br />

R 1 − e<br />

F · dr =<br />

0<br />

−Ry cosR − ye−Ry sen R<br />

1 + y2 dy<br />

R<br />

= arc tg R −<br />

dy .<br />

<br />

F · dr =<br />

Γ1<br />

<br />

F · dr +<br />

Γ2<br />

0<br />

0<br />

e −Ry (cosR + y sen R)<br />

1 + y 2<br />

<br />

F · dr +<br />

Parametrizando cada intervalo del modo obvio, se tiene<br />

<br />

<br />

<br />

<br />

Γ1<br />

Γ2<br />

Γ3<br />

Γ4<br />

<br />

F · dr =<br />

<br />

F · dr =<br />

F · dr =<br />

Γ1<br />

Γ2<br />

0<br />

R<br />

R<br />

P(x, 0) dx =<br />

0<br />

Γ3<br />

<br />

F · dr +<br />

sen x<br />

x dx,<br />

Γ4<br />

.<br />

<br />

F · dr .<br />

P(R, y) dx = 0 ya que x es constante,<br />

e −Rx sen x<br />

x<br />

F · dr = 0 por ser x constante.<br />

R e<br />

dx = −<br />

0<br />

−Rx sen x<br />

dx,<br />

x<br />

2


(5) Obtenemos fácilmente<br />

resulta<br />

<br />

<br />

<br />

<br />

Γ3<br />

<br />

<br />

F · dr<br />

=<br />

<br />

<br />

R<br />

e<br />

<br />

0<br />

−Rx <br />

sen x <br />

<br />

dx<br />

x <br />

R<br />

≤ e<br />

0<br />

−Rx<br />

<br />

sen<br />

x<br />

<br />

<br />

x<br />

dx<br />

R<br />

≤ e −Rx dx<br />

=<br />

= 1 − e−R2<br />

0<br />

<br />

− 1<br />

R e−Rx<br />

R<br />

x=R<br />

x=0<br />

−−−−→<br />

R→+∞<br />

(6) Teniendo en cuenta que |cosR + y sen R| ≤ y + 1 para todo y ≥ 0, y que<br />

<br />

<br />

R<br />

<br />

<br />

0<br />

e −Ry (cosR + y sen R)<br />

1 + y 2<br />

1<br />

≤ 1 para todo y,<br />

1 + y2 <br />

<br />

<br />

dy<br />

≤<br />

R<br />

e −Ry (y + 1) dy<br />

=<br />

= 1 − e−R2<br />

0<br />

<br />

− 1<br />

R e−Ry (y + 1)<br />

0 .<br />

y=R<br />

y=0<br />

(1 + R)<br />

+<br />

R<br />

1 − e−R2<br />

R2 + 1<br />

R<br />

e<br />

R 0<br />

−Ry dy<br />

−−−−→<br />

R→+∞<br />

(7) Sustituyendo los valores obtenidos en (4) en la fórmula del apartado (3), resulta:<br />

R<br />

0<br />

sen x<br />

x<br />

R<br />

dx = arc tg R +<br />

0<br />

e −Rx sen x<br />

x<br />

R<br />

dx −<br />

0<br />

e −Ry (cos R + y sen R)<br />

1 + y 2<br />

Tomando límites para R → +∞ en ambos miembros y, teniendo en cuenta los resultados de (5), se<br />

obtiene ∞ sen x<br />

π<br />

dx = lím (arc tg R) =<br />

0 x R→+∞ 2 .<br />

3<br />

0 .<br />

dy .


AMPLIACIÓN DE CÁLCULO (Curso 2008/2009) Convocatoria de junio 09.06.09<br />

NOMBRE . . . . . . . . . . . . . . . . . . . . . . . . . APELLIDOS . . . . . . . . . . . . . . . . . . . . . . . . . . . . . . . . . . . . . . . . . . .<br />

PROBLEMA 2 (4 puntos)<br />

Sea Σ la porción de la superficie de ecuación<br />

que pertenece al primer octante de R 3 y verifica que z ≤ 1.<br />

Número de matrícula. . . . . . . . . . . . . . . . . . . . . . . .<br />

x 2 + y 2 3/2 = (1 − z)y (1)<br />

1. (1 pto.) Sea A el sólido limitado por Σ y por los 3 planos coordenados. Sobre A hay una<br />

distribución de masa de forma que la densidad en cada punto de A es proporcional a la distancia<br />

del punto al plano xy. Se pide determinar el momento de inercia de A respecto del eje z.<br />

2. (1.5 ptos.) Calcular el flujo φ del campo F(x, y, z) = (y, −x, 0) a través de Σ orientada de<br />

forma que el vector normal en cada punto tenga su primera componente positiva.<br />

3. (1.5 ptos.) Sea Γ la curva intersección de Σ con los planos coordenados. Calcular el campo<br />

escalar g que cumple que g(0, 0, z) = z 2 /2 y que el flujo φ del apartado 2 es igual a la circulación<br />

del campo H(x, y, z) = (x, y, g(x, y, z)) sobre la curva Γ orientada adecuadamente. Calcular<br />

explícitamente dicha circulación para comprobar el resultado.<br />

Respuesta: Se entregará esta hoja y, a lo sumo, una adicional.<br />

Obsérvese que los puntos de la forma (0, 0, z) son soluciones de la ecuación, lo que quiere decir<br />

que todo el eje z es solución de la ecuación. Para estudiar la forma de la superficie Σ podemos pasar<br />

a cilíndricas. Se obtiene<br />

es decir, ρ = 0 (que corresponde al eje z) ó<br />

ρ 3 = (1 − z)ρsenθ<br />

ρ = √ 1 − z √ senθ (2)<br />

que corresponde a la superficie Σ. La variación de los parámetros es θ ∈ [0, π/2] y z ∈ [0, 1].<br />

Para ver qué forma tiene esta superficie cortamos por planos z igual a constante, z = C, 0 ≤ C ≤ 1<br />

obteniendo que (2) define una curva que une los puntos (0, 0, C) y (0, √ 1 − C, C) cuya ”amplitud”<br />

va decreciendo cuando aumenta z y se reduce al punto (0, 0, 1) cuando z = 1.<br />

Estudiemos también la curva Γ intersección de Σ con los planos coordenados. La intersección Γ1<br />

con el plano XY la podemos obtener o bien de (2) o bien de (1) haciendo z = 0. Así obtenemos la<br />

curva con ecuación polar ρ = √ senθ, θ ∈ [0, π/2] que une los puntos (0, 0, 0) y (0, 1, 0). Haciendo<br />

x = 0 en la (2) se obtiene que la ecuación de su intersección con el plano Y Z es y 3 = (1 − z)y, es<br />

decir, la curva y 2 = 1 − z con z ∈ [0, 1] a la que denominamos Γ2 y la curva Γ3 definida por y = 0,<br />

es decir, la porción del eje z con z ∈ [0, 1]. En cuanto a su intersección con el plano Y Z se obtiene,<br />

haciendo y = 0, que debe ser x = 0 con lo que de nuevo volvemos a obtener la curva Γ3.


Apdo. 1. La densidad es ρ(x, y, z) = Kz por lo que<br />

<br />

I = K z x 2 + y 2 dxdydz<br />

z<br />

1<br />

0<br />

0<br />

y<br />

A<br />

Procedimiento 1. Fijando z e integrando en x, y. Fijando primero la z e integrando en x y en y<br />

se tiene<br />

1 <br />

I = K zdz<br />

2 2<br />

x + y dxdy<br />

0<br />

donde ΠC es la intersección de A con el plano z = C. Haciendo un cambio a polares (x, y) = Ψ(ρ, θ)<br />

tenemos<br />

1 <br />

I = K zdz ρ 2 ρdρdθ<br />

0<br />

Πz<br />

Ψ −1 (Πz)<br />

Para obtener Ψ −1 (Πz) entramos con el cambio a polares en la ecuación de Σ y obtenemos (2) con lo<br />

que, para cada θ ∈ [0, π/2], ρ varía entre 0 y ρ = √ 1 − z √ senθ. Por tanto<br />

1 π/2<br />

I = K zdz<br />

0<br />

0<br />

0<br />

<br />

dθ<br />

√ 1−z √ senθ<br />

0<br />

ρ 3 dρ =<br />

= 1<br />

4 K<br />

1 π/2<br />

zdz (1 − z)<br />

0 0<br />

2 sen 2 θdθ = 1<br />

4 K<br />

1<br />

0<br />

= K<br />

π/2<br />

1 − cos 2θ<br />

β(2, 3)<br />

dθ =<br />

4 2<br />

K 1 π Kπ<br />

=<br />

4 12 4 192<br />

1<br />

x<br />

0<br />

z(1 − z) 2 <br />

dz<br />

π/2<br />

sen<br />

0<br />

2 <br />

θdθ =<br />

Procedimiento 2. Fijando x, y e integrando en z. La superficie Σ (salvo los puntos para los que<br />

y = 0, que tienen contenido nulo en R 2 ) se puede escribir en forma explícita mediante<br />

donde<br />

z = h(x, y) = 1 − (x2 + y 2 ) 3/2<br />

B =<br />

y<br />

<br />

(x, y) : x 2 + y 2 <br />

3/2<br />

≤ y y y = 0<br />

(x, y) ∈ B (3)<br />

(excluimos los puntos con y = 0 puesto que en ellos h no está definida, aunque a efectos de integración<br />

es irrelevante incluirlos o no)<br />

Por tanto<br />

<br />

I = K<br />

= K<br />

2<br />

<br />

B<br />

z x 2 + y 2 <br />

dxdydz = K<br />

A<br />

2 2<br />

x + y<br />

<br />

1 − (x2 + y 2 ) 3/2<br />

y<br />

B<br />

2<br />

dxdy<br />

dxdy x 2 + y 2 1− (x2 +y 2 ) 3/2<br />

y<br />

0<br />

zdz =


Obsérvese que la integral doble que se obtiene es una integral en sentido propio, pues (x2 +y 2 ) 3/2<br />

está acotada en B. Haciendo un cambio a polares se obtiene<br />

I = K<br />

2<br />

π/2<br />

0<br />

<br />

dθ<br />

√ senθ<br />

ρ<br />

0<br />

3<br />

<br />

1 − ρ2<br />

2 dρ<br />

senθ<br />

Para poder calcular la primera integral hacemos un cambio ρ = w(u) = √ senθu, y se tiene<br />

I = K<br />

π/2 1 √ 3 1 2<br />

dθ senθu − u<br />

2 0 0<br />

2 √ senθdu =<br />

= K<br />

π/2<br />

sen<br />

2<br />

2 1<br />

θdθ u 3 1 − u 2 <br />

2<br />

du = K<br />

1<br />

π<br />

u<br />

2 4<br />

3 1 − u 22 du<br />

0<br />

0<br />

Para calcular esta última integral se puede desarrollar (1 − u 2 ) 3 o, de forma más elegante, relacionarla<br />

con una β. Para ello hacemos el cambio u = h(t) = √ t y se obtiene<br />

1<br />

con lo que<br />

0<br />

u 3 1 − u 2 2 du = 1<br />

2<br />

1<br />

0<br />

t 3/2 (1 − t) 2 t −1/2 dt = 1<br />

2<br />

I = K<br />

2<br />

π<br />

4<br />

1<br />

24<br />

= Kπ<br />

192<br />

1<br />

0<br />

0<br />

t (1 − t) 2 dt = 1 1<br />

β(2, 3) =<br />

2 24<br />

Apdo. 2.<br />

Primer procedimiento. En primer lugar tenemos en cuenta que F es de clase 1 y solenoidal en R3 .<br />

La superficie Σ no es cerrada por lo que no se puede asegurar que el flujo valga cero, pero sí se puede<br />

”cerrar” la superficie con otras superficies más cómodas y aplicar Gauss, con lo que pasaríamos a<br />

integrar sobre dichas superficies. Sea Σ1 la porción de A que pertenece al plano xy (orientada con<br />

la normal n = (0, 0, 1)) y sea Σ2 la porción de A que pertenece al plano yz (orientada con la normal<br />

n = (1, 0, 0)). Entonces podemos escribir<br />

<br />

<br />

φ − F · ds− (y, −x, 0) · ds = divFdxdydz = 0<br />

luego<br />

φ =<br />

=<br />

=<br />

Σ1<br />

<br />

<br />

<br />

Σ1<br />

Σ1<br />

Σ2<br />

Σ2<br />

<br />

(y, −x, 0) · ds+ (y, −x, 0) · ds =<br />

Σ2 <br />

(y, −x, 0) · (0, 0, 1)ds+ (y, −x, 0) · (1, 0, 0)ds =<br />

yds<br />

donde se ha utilizado que (y, −x, 0) · (0, 0, 1) = 0. Claramente esta integral es más cómoda que<br />

calcular el flujo integrando sobre Σ.<br />

Ya hemos visto que la ecuación de la intersección de Σ con el plano x = 0 es y = 0 (que<br />

corresponde al eje z) y y 2 = 1 − z, que es la curva buscada. Por ello Σ2 se puede parametrizar en la<br />

forma<br />

x = 0<br />

y = y (y, z) ∈ T = (y, z) : 0 ≤ z ≤ 1 − y 2 , y ∈ [0, 1] <br />

z = z<br />

Σ2<br />

A<br />

y


y por tanto<br />

<br />

φ =<br />

Σ2<br />

<br />

yds =<br />

T<br />

1<br />

ydydz =<br />

0<br />

y(1 − y 2 )dy = 1<br />

4<br />

Segundo procedimiento. Si no nos damos cuenta de lo anterior, debemos calcular el flujo integrando<br />

directamente sobre Σ. Para ello en primer lugar necesitamos parametrizar la superficie.<br />

Primera forma de parametrizar. Usando (2) tenemos, utilizando que x = ρ cos θ, y = ρsenθ, la<br />

parametrización (x, y, z) = Φ(θ, z) , (θ, z) ∈ [0, π/2] × [0, 1] dada por<br />

Ahora<br />

x = √ 1 − z √ senθ cos θ (4)<br />

y = √ 1 − z √ senθsenθ = √ 1 − zsen 3/2 θ (θ, z) ∈ [0, π/2] × [0, 1]<br />

z = z<br />

N(θ, z) = ± ∂Φ<br />

∂θ<br />

× ∂Φ<br />

∂z<br />

<br />

<br />

i j k<br />

√ 1−3sen<br />

= ± 1 − z<br />

<br />

<br />

2θ 2 √ √ 3 1 − z<br />

senθ<br />

√ <br />

<br />

<br />

senθ cos θ 0 <br />

2 <br />

<br />

1 <br />

− √ senθ cos θ<br />

2 √ 1−z<br />

− sen3/2 θ<br />

2 √ 1−z<br />

Nótese que como la tercera componente del campo F es idénticamente nula no hace falta calcular la<br />

tercera componente de N. Así<br />

0<br />

N = ±<br />

0<br />

<br />

3√<br />

√ √ 1 − 3sen<br />

1 − z senθ cos θ, − 1 − z<br />

2<br />

2θ 2 √ senθ<br />

La orientación del enunciado para Σ hace que tomemos el signo + y por ello se tiene<br />

<br />

φ = (y, −x, 0) · ds =<br />

=<br />

Σ<br />

√1 √ √ <br />

3/2 3√<br />

√ √ 1 − 3sen<br />

− zsen θ, − 1 − z senθ cos θ, 0 · 1 − z senθ cos θ, − 1 − z<br />

[0,π/2]×[0,1]<br />

2<br />

2θ 2 √ =<br />

, N<br />

senθ<br />

<br />

3<br />

[0,π/2]×[0,1] 2 (1 − z) sen2θ cos θ + 1<br />

2 (1 − z)(1 − 3sen2 <br />

θ) cos θ dθdz operando<br />

=<br />

=<br />

1 <br />

(1 − z)dz<br />

π/2<br />

cos θ<br />

2 dθ<br />

<br />

= 1<br />

4<br />

Segunda forma de parametrizar. También podemos utilizar la expresión explícita (3) de la superficie,<br />

con lo que se tiene la parametrización (x, y, z) = Θ(x, y) , (x, y) ∈ B dada por<br />

x = x<br />

y = y (x, y) ∈ B<br />

z = h(x, y) = 1 − (x2 + y 2 ) 3/2<br />

Entonces<br />

y<br />

N(θ, z) = ± ∂Θ<br />

∂x<br />

<br />

= ±<br />

× ∂Θ<br />

∂y<br />

<br />

= ±<br />

3 (x 2 + y 2 ) 1/2<br />

y<br />

, N3<br />

<br />

(x, y) : x 2 + y 2 <br />

3/2<br />

≤ y y y = 0<br />

<br />

<br />

(x, y), −∂h (x, y), 1<br />

∂y<br />

− ∂h<br />

=<br />

∂x<br />

, (x2 + y2 ) 1/2 (2y2 − x2 )<br />

y2 <br />

, 1


donde elegimos el signo + que es el correspondiente a la orientación definida en el enunciado. Así<br />

φ =<br />

<br />

<br />

<br />

3 (x<br />

(y, −x, 0) · ds = (y, −x, 0) ·<br />

Σ<br />

B<br />

2 + y2 ) 1/2<br />

,<br />

y<br />

(x2 + y2 ) 1/2 (2y2 − x2 )<br />

y2 <br />

, 1 dxdy operando<br />

=<br />

=<br />

<br />

<br />

<br />

x2 + y2 x +<br />

B<br />

x3<br />

y2 <br />

dxdy polares<br />

=<br />

=<br />

π/2 <br />

dθ<br />

0<br />

√ senθ<br />

ρ<br />

0<br />

3<br />

<br />

cos θ + cos3 θ<br />

sen2 π/2 <br />

dρ = cos θ +<br />

θ<br />

0<br />

cos3 θ<br />

sen2 =<br />

2 sen θ<br />

dθ =<br />

θ 4 1<br />

π/2<br />

<br />

π/2<br />

2 3 1<br />

cos θsen θ + cos θ dθ = cos θdθ =<br />

4<br />

4<br />

1<br />

4<br />

0<br />

0<br />

Apdo. 3. Obsérvese que el borde de Σ es precisamente la curva Γ. Puesto que se pide relacionar<br />

el flujo de F con la circulación de H sobre su borde Γ, si H fuese un potencial vector de F de clase<br />

uno en R3 se tendría, usando el teorema de Stokes<br />

<br />

H · dr = F · ds<br />

Γ<br />

donde la orientación de Γ es la coherente con la de Σ.<br />

Ya sabemos que F admite potencial vector de clase 1 en R 3 , pues F es de clase 1 y solenoidal<br />

en el conjunto estrellado R 3 . En definitiva, buscamos encontrar un potencial vector H de F de la<br />

forma H(x, y, x) = (x, y, g(x, y, z)) y que cumpla que g(0, 0, z) = z 2 /2. Imponiendo que rotH = F se<br />

obtiene<br />

∂g<br />

∂y<br />

= y ; − ∂g<br />

∂x<br />

Σ<br />

= −x ; 0 = 0<br />

De la primera igualdad se deduce que g(x, y, z) = y 2 /2 + α(x, z) y entrando en la segunda obtenemos<br />

∂α<br />

∂x = x luego α(x, z) = x2 /2 + γ(z), es decir, g(x, y, z) = x 2 /2 + y 2 /2 + γ(z). Imponiendo g(0, 0, z) =<br />

z 2 /2 se obtiene finalmente g(x, y, z) = x 2 /2+y 2 /2+z 2 /2, luego H(x, y, x) = (x, y, x 2 /2+y 2 /2+z 2 /2).<br />

Para calcular la circulación sobre Γ, que debemos orientar de forma coherente con Σ, expresamos<br />

Γ en la forma Γ = Γ1 ∪ (−Γ2) ∪ (−Γ3)<br />

Γ1 : intersección de Σ con el plano XY orientada en sentido antihorario (si se mira desde arriba)<br />

Γ2 : intersección de Σ con el plano Y Z (salvo eje z) orientada en sentido horario<br />

(si se mira desde la parte positiva del eje x)<br />

Γ3 : intersección de Σ con el eje z orientada hacia arriba<br />

Entonces <br />

H · dr =<br />

Γ<br />

Γ1<br />

<br />

H · dr −<br />

Γ2<br />

<br />

H · dr −<br />

Γ3<br />

H · dr<br />

Parametrización de Γ1: se puede obtener de la parametrización de Σ (4) haciendo z = 0<br />

x = α(θ) = √ senθ cos θ (5)<br />

y = β(θ) = sen 3/2 θ θ ∈ [0, π/2]<br />

z = γ(θ) = 0<br />

Como γ ′ (θ) = 0 no es necesario escribir la tercera componente de H. Por ello<br />

<br />

H · dr<br />

Γ1<br />

=<br />

<br />

<br />

π/2 √ <br />

3/2 1 − 3sen<br />

(x, y, H3) · dr = senθ cos θ, sen θ, H3 ·<br />

Γ1<br />

0<br />

2θ 2 √ senθ , 3√ <br />

senθ cos θ<br />

, 0 dθ<br />

2<br />

operando<br />

=<br />

=<br />

π/2<br />

cos θ 1<br />

dθ =<br />

2 2<br />

0


Parametrización de Γ2: Ya hemos obtenido que la ecuación cartesiana de Γ2 como curva en el<br />

plano x = 0 es y 2 = 1 − z, y ∈ [0, 1]. Por ello podemos parametrizar en la forma<br />

Por ello<br />

<br />

H · dr =<br />

Γ2<br />

=<br />

<br />

Γ2<br />

1<br />

Parametrización de Γ3:<br />

y por tanto<br />

0<br />

x = 0<br />

y = y y ∈ [0, 1]<br />

z = 1 − y 2<br />

(x, y, x 2 /2 + y 2 /2 + z 2 x=0 en Γ2<br />

/2) · dr =<br />

(y − y 3 − y 1 − y 2 2 )dy = 1<br />

2<br />

<br />

Γ3<br />

x = 0<br />

1 1<br />

− −<br />

4 6<br />

1<br />

(0, y, y 2 /2 + 1 − y 22 /2) · (0, 1, −2y)dy =<br />

0<br />

= 1<br />

12<br />

y = 0 z ∈ [0, 1]<br />

z = z<br />

1<br />

H · dr= (0, 0, z 2 /2) · (0, 0, 1)dz = 1<br />

6<br />

En definitiva <br />

H · dr=<br />

Γ<br />

1 1 1<br />

− −<br />

2 12 6<br />

0<br />

como debíamos obtener.<br />

Obsérvese que podríamos haber simplificado un poco las operaciones dándonos cuenta de que H<br />

se puede escribir en la forma<br />

= 1<br />

4<br />

H = (x, y, x 2 /2 + y 2 /2 + z 2 /2) = (x, y, z 2 /2) + (0, 0, x 2 /2 + y 2 /2)<br />

El primer campo es conservativo en R3 (pues es irrotacional en R3 que es un abierto estrellado) con<br />

lo que su circulación a lo largo de una curva cerrada es nula. Por ello<br />

<br />

H · dr = (0, 0, x 2 /2 + y 2 /2) · dr<br />

y ahora <br />

Γ1 (0, 0, x2 /2 + y 2 /2) · dr = <br />

<br />

Γ<br />

<br />

H · dr = −<br />

=<br />

1<br />

0<br />

Γ2<br />

Γ<br />

Γ<br />

Γ3 (0, 0, x2 /2 + y 2 /2) · dr = 0 por lo que<br />

(0, 0, x 2 /2 + y 2 1<br />

/2) · dr = − (0, 0, y 2 /2) · (0, 1, −2y)dy =<br />

y 3 dy = 1<br />

4<br />

0


AMPLIACIÓN DE CÁLCULO (Curso 2008/2009) Convocatoria de Junio 09.06.09<br />

PROBLEMA 3 (3 puntos)<br />

Se considera el cono Ωh de base x 2 + y 2 ≤ 1, z = 0 y vértice (0, 0, h) .<br />

1. Calcular mediante una integral el área de su superficie lateral cónica Σh.<br />

2. Sea F (x, y, z) = y 2 z − z 4 , −x 2 z, 2z − 8y 2 el campo de velocidades de un fluido. Calcular el flujo que<br />

sale por la base de Ωh.<br />

3. ¿Cuál debe ser la altura h del cono Ωh para que el flujo de F que entra a través de la superficie cónica<br />

Σh sea π?<br />

Respuesta: Se entregará esta hoja y, a lo sumo, una adicional.<br />

1. La superficie cónica está formada por segmentos que parten de la circunferencia (cos t, sen t, 0) y<br />

terminan en el vértice (0, 0, h) , luego sus ecuaciones paramétricas son<br />

⎫<br />

x = a cos t ⎪⎬<br />

y = asen t t ∈ [0, 2π], a ∈ [0, 1]<br />

⎪⎭ z = h − ah<br />

de donde eliminando el parámetro a = 1 − z<br />

h también puede extraerse la ecuación de la superficie<br />

cónica:<br />

x 2 + y 2 <br />

= 1 − z<br />

2 .<br />

h<br />

Para calcular su área, usaremos la parametrización obtenida:<br />

por tanto<br />

área (Σh) =<br />

φ (a, t) = (a cos t, asen t, h − ha)<br />

∂φ<br />

∂t<br />

= (−asen t, a cos t, 0)<br />

∂φ<br />

∂a <br />

<br />

<br />

∂φ ∂φ <br />

× <br />

∂t ∂a <br />

=<br />

=<br />

(cos t, sen t, −h)<br />

a1<br />

+ h2 2π <br />

1 <br />

<br />

∂φ ∂φ <br />

× <br />

0 0 ∂t ∂a dadt = 1 + h2 2π 1 <br />

dt ada = π<br />

0<br />

0<br />

<br />

1 + h2 2. La base es B = {(x, y, 0) x 2 + y 2 ≤ 1} orientada con normal (0, 0, −1), que es la normal saliente del<br />

sólido Ωh por su base B. En los puntos de B el campo toma el valor F (x, y, 0) = 0, 0, −8y 2 ; entonces<br />

el flujo pedido es<br />

<br />

F ds =<br />

B<br />

x 2 +y 2 ≤1<br />

<br />

0, 0, −8y 2<br />

<br />

◦(0, 0, −1) dxdy =<br />

x2 +y2 8y<br />

≤1<br />

2 2π<br />

dxdy = 8 sen<br />

0<br />

2 1<br />

tdt r<br />

0<br />

3 <br />

dr = 2π.<br />

3. El campo es de clase C 1 (Ωh) y div (F) = 2 luego por el Teorema de Gauss el flujo a través de la<br />

superficie exterior de Ωh es<br />

<br />

B<br />

<br />

F ds −<br />

Σh<br />

<br />

F ds =<br />

Ωh<br />

divF dxdydz = 2vol (Ωh) = 2 π<br />

3 h.<br />

donde la integral de superficie <br />

Σh F ds está orientada hacia el interior de Ωh tal y como se nos pide.<br />

Entonces <br />

F ds = F ds −<br />

Σh<br />

B<br />

2π 2π<br />

h=2π −<br />

3 3 h.<br />

Por tanto, la altura pedida h debe ser tal que<br />

π = 2π − 2π<br />

h ⇐⇒ h=3<br />

3 2 .


AMPLIACIÓN DE CÁLCULO (Curso 2008/2009) Convocatoria de septiembre 15.09.09<br />

PROBLEMA 1 (3 puntos)<br />

Estudiar la convergencia de las siguientes integrales impropias y calcularlas explícitamente en caso<br />

∞<br />

sen t π<br />

de ser convergentes. Se supone conocido el resultado dt =<br />

0 t 2 .<br />

1.<br />

∞ 1 − cos t<br />

(0,5 puntos)<br />

0 t2 dt,<br />

2.<br />

1 dt<br />

(0,5 puntos) <br />

0 t log(1/t) ,<br />

<br />

3. (1 punto)<br />

{(x,y)∈R2 e<br />

/ 0≤x


2.<br />

1 dt<br />

.<br />

0 t log(1/t)<br />

La integral es impropia en ambos límites, vamos a transformarla mediante un<br />

cambio de variable y razonaremos conforme al resultado:<br />

Cambio de variable: Sea f : (a, b) → R una función continua; sea ϕ : (α, β) → R de clase 1,<br />

ϕ ′ = 0 y tal que ϕ((α, β)) = (a, b). Entonces se da igualdad<br />

b β<br />

f(t)dt = f(ϕ(x))|ϕ<br />

a<br />

α<br />

′ (x)|dx.<br />

Es decir, ambas integrales convergen o divergen simultáneamente.<br />

Hagamos el cambio ϕ(x) = e −x que es una función monótona decreciente que transforma biyectivamente<br />

la semirrecta (0, ∞) en el intervalo (0, 1). Se tiene:<br />

f(t) =<br />

1<br />

<br />

t log(1/t) =⇒ f(ϕ(x)) = f(e−x ) = ex/2<br />

√ , f(ϕ(x))|ϕ<br />

x ′ (x)| = ex/2<br />

√ e<br />

x −x = x −1/2 e −x/2 .<br />

Al hacer ahora el cambio x = 2y, dx = 2dy la integral queda:<br />

3.<br />

1 dt<br />

<br />

0 t log(1/t)<br />

∞<br />

= 2 (2y)<br />

0<br />

−1/2 e −y dy = √ ∞<br />

2 y<br />

0<br />

−1/2 e −y dy = √ 2Γ(1/2) = √ 2π.<br />

Para los apartados 3. y 4. utilizaremos los siguientes resultados:<br />

Sea S ⊂ R n y f una función definida en S. Entonces<br />

<br />

<br />

f es convergente ⇐⇒ |f| es convergente.<br />

S<br />

S<br />

<br />

Si f es no negativa en S y existe una sucesión básica {Mk}k∈N tal que existe el lím<br />

<br />

<br />

k→∞<br />

entonces f es integrable en S y además f = lím<br />

S k→∞<br />

f.<br />

<br />

{(x,y)∈R 2 / 0≤x


y, como lím e<br />

k→∞ −k2<br />

= 0, la integral es convergente y vale 1/2.<br />

4.<br />

<br />

{(x,y)∈R 2 / 0


AMPLIACIÓN DE CÁLCULO (Curso 2008/2009) Convocatoria de Septiembre 15.09.09<br />

NOMBRE . . . . . . . . . . . . . . . . . . . . . . . . . APELLIDOS . . . . . . . . . . . . . . . . . . . . . . . . . . . . . . . . . . . . . . . . . . .<br />

PROBLEMA 2 (4 puntos)<br />

Número de matrícula. . . . . . . . . . . . . . . . . . . . . . . .<br />

Sea a un vector no nulo de R 3 , sea α ∈ R y denotamos r = (x, y, z), r = r.<br />

1. (0.75 ptos.) Calcular la divergencia del campo<br />

H(r) =<br />

a · r α<br />

r −<br />

r3 r a<br />

y estudiar para qué valores de α es solenoidal en su dominio de definición.<br />

2. (1 pto.) Para los valores de α calculados en el apartado 1 estudiar si el campo H admite un<br />

potencial vector de la forma F(r) = f(r)a × r donde f : (0, ∞) → R es una función de clase 1. En<br />

caso afirmativo calcularlo.<br />

3. (1.25 ptos.) Sea<br />

G(x, y, z) =<br />

x + y<br />

(x2 + y2 + z2 (x, y, z) +<br />

3/2<br />

)<br />

1<br />

(x2 + y2 + z2 (1, 1, 0)<br />

1/2<br />

)<br />

y sea S la porción de la superficie esférica de centro el origen y radio 2 que está situada por encima<br />

del plano x = z, orientada con la normal ”saliente”. Calcular el flujo de G a través de S.<br />

4. (1 pto.) Sea<br />

T(x, y, z) =<br />

Calcular el flujo de T a través de S.<br />

Nota: a × (b × c) = (a · c)b − (a · b)c<br />

x + y<br />

(x2 + y2 + z2 1<br />

2 (x, y, z) +<br />

) (x2 + y2 + z2 (1, 1, 0)<br />

)<br />

Respuesta: Se entregará esta hoja y, a lo sumo, una adicional.<br />

1. El campo H está definido en Ω := R3 − {0} y además es de clase infinito en dicho conjunto.<br />

Utilizando que div(UV) = gradU · V + UdivF, que grad(U/V ) = (V gradU − UgradV )/V 2 grad(r) = r/r para r = 0 y la regla de la cadena se tiene<br />

, que<br />

divH(r) =<br />

<br />

a · r<br />

· r · r<br />

div( r) − αdiv(1 a) = grad(a ) · r+a div(r) − α grad(<br />

r3 r r3 r3 1 1<br />

) · a +<br />

r r diva<br />

<br />

=<br />

= grad(a · r)r3 − a · r grad r3 r6 a · r 1<br />

· r + 3 + α<br />

r3 r2 r<br />

· a =<br />

r<br />

= r3 2 r<br />

a − a · r 3r r<br />

r6 a · r 1<br />

· r + 3 + α<br />

r3 r2 r<br />

· r<br />

a = (1 + α)a<br />

r r3 donde también se ha utilizado que grad(a · r) = a.<br />

Puesto que a·r no se anula salvo en los puntos del plano que pasa por el origen y es perpendicular<br />

a a, para que divH(r) sea nula en Ω debe ser 1 + α = 0, es decir, α = −1.<br />

2. Considerando el caso α = −1, sea<br />

H(r) =<br />

a · r 1<br />

r +<br />

r3 r a


Obsérvese que aunque H es solenoidal en su dominio Ω, no es posible asegurar a priori que H admita<br />

potencial vector en Ω, pues Ω no es estrellado.<br />

Calcularemos rot F para ver si se cumple rot F = H para alguna función f. Veamos<br />

rot F(r) = grad(f(r)) × (a × r) + f(r)rot(a × r)<br />

Usando la regla de la cadena obtenemos grad(f(r)) = f ′ (r) r<br />

r y como a×r = (a2z − a3y, a3x − a1z, a1y − a2x)<br />

calculando se obtiene rot(a × r) = 2a. Por ello<br />

rot F(r) = f ′ (r) r<br />

× (a × r) + 2f(r)a<br />

r<br />

Utilizando la fórmula para el doble producto vectorial a × (b × c) = (a · c)b − (a · b)c obtenemos<br />

finalmente<br />

rot F(r) = f ′ (r)<br />

r<br />

Por tanto rot F = H en Ω si y solo si se cumple<br />

2 ′ f<br />

r a − (a · r) r + 2f(r)a = (rf (r) + 2f(r)) a − ′ (r)<br />

(a · r) r<br />

r<br />

rf ′ (r) + 2f(r) = 1<br />

r<br />

y − f ′ (r)<br />

r<br />

1<br />

= para todo r > 0<br />

r3 es decir, debemos estudiar si existe alguna f tal que las dos ecuaciones diferenciales ordinarias<br />

tf ′ (t) + 2f(t) = 1<br />

t<br />

y − f ′ (t)<br />

t<br />

1<br />

= para todo t > 0<br />

t3 tienen solución. De la segunda ecuación tenemos f ′ (t) = − 1<br />

t2 por lo que integrando directamente se<br />

obtiene que sus soluciones son<br />

f(t) = 1<br />

+ C<br />

t<br />

donde C es una constante arbitraria. Entrando en la primera ecuación se obtiene<br />

−t 1<br />

<br />

1<br />

+ 2<br />

t2 t<br />

<br />

+ C<br />

= 1<br />

t<br />

que se verifica si y sólo si C = 0. En definitiva, existe una única solución que es<br />

F(r) = 1<br />

a × r<br />

t<br />

Obsérvese que hemos obtenido un potencial vector de H en Ω con lo que en este caso concreto<br />

hemos demostrado que sí existe potencial vector en Ω a pesar de que no se cumplen las condiciones<br />

suficientes (campo solenoidal en un conjunto abierto estrellado).<br />

3. Del enunciado se deduce que G corresponde al campo H del apartado 1 para α = −1 cuando<br />

a = (1, 1, 0). Por lo tanto, el campo es solenoidal en Ω y además, como resultado del aparatado<br />

anterior, el campo F(r) = 1a<br />

× r es un potencial vector de G cuando a = (1, 1, 0).<br />

r<br />

El cálculo del flujo pedido se puede enfocar:<br />

a) A través de la definición calculando la integral de superficie.<br />

b) Aplicando el teorema de Stokes,calculando la circulación de F sobre la curva frontera Γ de S,<br />

es decir,<br />

<br />

φ := Gds = Fdr<br />

S<br />

Γ<br />

c) Aplicando el teorema de Gauss. Puesto que G es solenoidal en Ω, si Σ es cualquier superficie<br />

que no pase por el origen (pues ahí no está definido el campo) que se apoye sobre Γ (y esté orientada


de forma coherente con la misma) y tal que el reciento limitado por S ∪ Σ no contenga al origen, la<br />

aplicación del teorema de Gauss permite afirmar que<br />

<br />

φ = Gds<br />

La opción (a) presenta la dificultad de que no es fácil obtener una parametrización simple de la<br />

superficie. Por ejemplo, un primer intento de parametrizar la superficie podría ser<br />

Σ<br />

x = 2 cos θsenϕ ; y = 2senθsenϕ , z = 2 cos ϕ ; (θ, ϕ) ∈ A<br />

pero el dominio A en el que varían θ y ϕ no es fácil de calcular. En efecto, A será de la forma<br />

A = [0, 2π] × [0, ϕ(θ)] donde ϕ(θ) se obtiene imponiendo x = z, es decir, cos θsenϕ(θ) = cos ϕ(θ).<br />

Como se ve, es complicado obtener ϕ(θ) a partir de esta ecuación.<br />

Una parametrización más sencilla, aunque no tan evidente, es considerar que ϕ sea el ángulo que<br />

el radio vector forma con el eje y, en cuyo caso<br />

x = 2 cos θsenϕ ; y = 2 cos ϕ, z = 2senθsenϕ ; (θ, ϕ) ∈ A (1)<br />

Dibujando la superficie es fácil ver que ahora el dominio en que varían los parámetros es A =<br />

[ π 5π , ] × [0, π]. Con esta parametrización el flujo se podría calcular sin mucha dificultad utilizando la<br />

4 4<br />

definición.<br />

Estudiemos el camino (c). En nuestro caso la aplicación de este resultado no es muy útil, pues no<br />

hay superficies ”fáciles” Σ que cumplan las condiciones anteriormente mencionadas. En un primer<br />

momento nos podríamos ver tentados de tomar como superficie Σ la porción del plano x = z contenida<br />

dentro de la esfera. Sin embargo dicha superficie pasa por el origen, donde G no está definido, y por<br />

ello el teorema de Gauss no podría aplicarse.<br />

Procedamos por el camino (b). Consideremos la curva frontera Γ de la superficie S, es decir, la<br />

circunferencia de ecuación<br />

x 2 + y 2 + z 2 = 4<br />

x = z<br />

orientada de forma coherente con S, es decir, de forma que su proyección sobre el plano xy se recorre<br />

en sentido positivo. Como el campo F es de clase 1 en Ω y la superficie S está contenida en Ω, se<br />

puede aplicar el teorema de Stokes obteniéndose<br />

<br />

φ := Gds =<br />

S<br />

Γ<br />

<br />

1<br />

Fdr = a × r dr<br />

Γ r<br />

Puesto que Γ está contenida en la esfera y en ella la distancia al origen es 2 se puede simplificar la<br />

expresión anterior<br />

<br />

<br />

1<br />

1<br />

φ = a × r dr = a × r dr<br />

Γ r 2 Γ<br />

Ahora tenemos dos posibles caminos. (b.1) Calcular la circulación anterior utilizando la definición o<br />

(b.2) aplicar el teorema de Stokes y calcular dicha circulación mediante una integral de superficie.<br />

Camino (b.2). Sea Π la porción del plano x = z contenida dentro de la esfera orientada de forma<br />

coherente con la curva. Puesto que a × r es de clase 1 en R3 podemos aplicar Stokes y escribir,<br />

teniendo en cuenta que el vector normal unitario a Π es n = (−1, 0, 1)/ √ 2,<br />

φ = 1<br />

<br />

a × r dr =<br />

2 Γ<br />

1<br />

<br />

rot(a × r) ds =<br />

2 Π<br />

1<br />

<br />

2a ds = a · n ds =<br />

2 Π<br />

Π<br />

<br />

(−1, 0, 1)<br />

= a · √ ds =<br />

Π 2<br />

1<br />

<br />

√ (a3 − a1)ds =<br />

2 Π<br />

1<br />

<br />

√ (a3 − a1) ds =<br />

2 Π<br />

= 1<br />

1<br />

√ (a3 − a1) Área(Π) = √ (a3 − a1)π2<br />

2 2 2 = −2 √ 2π


donde se ha usado que Π es un círculo de radio 2.<br />

Camino (b.1). Parametricemos Γ, para lo cual escribimos Γ en la forma<br />

2x 2 + y 2 = 4<br />

x = z<br />

donde la primera ecuación define la curva proyección de Γ sobre el plano xy. Parametrizamos dicha<br />

proyección (x/ √ 2) 2 + (y/2) 2 = 1 en la forma<br />

x = √ 2 cos t ; y = 2 sint , t ∈ [0, 2π]<br />

(obsérvese que la orientación es la correcta) y por ello una posible parametrización de Γ es<br />

x = √ 2 cos t ; y = 2 sint ; z = √ 2 cos t , t ∈ [0, 2π]<br />

Aplicando la definición se tiene<br />

φ = 1<br />

<br />

a × r dr =<br />

2 Γ<br />

1<br />

=<br />

<br />

(z, −z, y − x) dr =<br />

2 Γ<br />

1<br />

2π √ √ √ <br />

2 cos t, − 2 cos t, 2 sint − 2 cos t ·<br />

2<br />

= 1<br />

2<br />

0<br />

2π<br />

0<br />

<br />

−2 √ <br />

2 dt = −2 √ 2π<br />

4. El campo T se puede escribir en la forma<br />

T(r) =<br />

a · r 1<br />

r+ a<br />

r4 r2 <br />

− √ 2sin t, 2 cos t, − √ <br />

2sin t dt =<br />

con a = (1, 1, 0). Este campo no es solenoidal (se puede calcular su divergencia para comprobarlo)<br />

con lo que en principio la forma más inmediata de calcular el flujo pedido<br />

<br />

a · r 1<br />

α := Tds = r+ a ds<br />

r4 r2 S<br />

S<br />

es calculando la integral de superficie aplicando la definición. Nótese que, como ya se vio en el<br />

apartado 3, el cálculo de dicha integral tiene el inconveniente de que no es fácil parametrizar la<br />

superficie. Además, si aplicásemos el teorema de Gauss tendríamos que calcular un flujo sobre una<br />

superficie en principio más difícil que el casquete esférico (como ya se vio en el punto (c) del apartado<br />

anterior, no se puede utilizar el plano Π) y además una integral triple, por lo que no parece el método<br />

más aconsejable.<br />

Como resultado de los razonamientos anteriores, intentaremos relacionar el flujo α con el flujo φ<br />

calculado en el apartado anterior, pues la superficie es la misma y el campo, si bien es distinto, tiene<br />

una forma muy parecida. Si no lo conseguimos entonces calcularemos α aplicando la definición.<br />

Puesto que sobre la esfera el vector normal unitario es r = 2 tenemos que<br />

<br />

α :=<br />

S<br />

1<br />

r<br />

a · r<br />

1<br />

r +<br />

r3 r a<br />

<br />

ds = 1<br />

2<br />

<br />

S<br />

a · r<br />

1<br />

r +<br />

r3 r a<br />

<br />

ds = 1<br />

2 φ = −√ 2π<br />

Si no nos damos cuenta de esta circunstancia y no somos capaces de relacionar α con φ aplicamos<br />

la definición usando la parametrización (1). Como el vector unitario es n = r/R = r/2 se tiene<br />

<br />

a · r 1<br />

α = r + a ·<br />

S r4 r2 r<br />

<br />

1 a · r 1<br />

ds = + a · r ds =<br />

2 2 S r2 r2 1<br />

23 <br />

(a · r + a · r) ds =<br />

S<br />

= 1<br />

<br />

a · rds<br />

4<br />

a=(1,1,0)<br />

= 1<br />

<br />

xds + yds<br />

4<br />

S<br />

S<br />

S


Como S es simétrica respecto del plano y = 0 y y es una función impar en y s sigue que <br />

yds = 0<br />

S<br />

con lo que, usando (1),<br />

α = 1<br />

<br />

xds =<br />

4 S<br />

1<br />

4<br />

= − √ 2π<br />

5π<br />

4<br />

π<br />

4<br />

π<br />

0<br />

5π π<br />

4<br />

(2 cos θsenϕ) (4senϕ) dθdϕ = 2 cos θdθ sen<br />

π<br />

0<br />

4<br />

2 ϕdϕ =


AMPLIACIÓN DE CÁLCULO (Curso 2008/2009) Convocatoria de septiembre 15.09.09<br />

NOMBRE . . . . . . . . . . . . . . . . . . . . . . . . . APELLIDOS . . . . . . . . . . . . . . . . . . . . . . . . . . . . . . . . . . . . . . . . . . .<br />

Número de matrícula. . . . . . . . . . . . . . . . . . . . . . . .<br />

Problema 3 (3 puntos): Se considera el sólido Ω interior al cilindro (x − a) 2 + y 2 = a 2 (a > 0)<br />

limitado por la superficie cónica z 2 = x 2 + y 2 , z ≥ 0, y el plano XOY.<br />

1. Calcular el volumen de Ω. (1 punto)<br />

2. Hallar el área de su superficie lateral. (Sugerencia: relacionarla con una integral curvilínea).<br />

(1 punto)<br />

3. Sea F (x, y, z) = <br />

0, 0, − √ x2 + y2 <br />

el campo de velocidades de un fluido. Calcular el flujo que<br />

entra por la base superior de Ω. (1 punto)<br />

Respuesta: (Se entregará esta hoja y, a lo sumo, una adicional)<br />

1. El sólido es<br />

Ω = {(x, y, z) : (x − a) 2 + y 2 ≤ a 2<br />

0 ≤ z ≤<br />

<br />

x 2 + y 2 }<br />

luego es el espacio proyectable sobre el disco plano D : (x − a) 2 + y2 ≤ a2 . Así pues:<br />

<br />

vol (Ω) =<br />

<br />

dxdydz =<br />

⎛<br />

<br />

⎝<br />

Ω<br />

D<br />

√ x2 +y2 ⎞<br />

<br />

dz⎠<br />

dxdy =<br />

0<br />

D<br />

<br />

x 2 + y 2 dxdy.<br />

Para calcular esta integral doble, podemos cambiar a coordenadas polares en D, donde se<br />

verifica que<br />

(x − a) 2 + y 2 ≤ a 2 ⇔<br />

x 2 + y 2 ≤ 2ax ⇔<br />

0 ≤ ρ ≤ 2a cos θ<br />

así que, para cada ángulo θ con cos θ ≥ 0 (es decir, para θ ∈ [−π/2, π/2]) se tiene que ρ ∈<br />

[0, 2a cos θ]. Por tanto, la integral queda<br />

vol (Ω) =<br />

π/2 2a cos θ<br />

ρ<br />

−π/2 0<br />

2 dρdθ = 1<br />

π/2<br />

(2a cos θ)<br />

3 −π/2<br />

3 dθ = 16a3<br />

π/2<br />

cos<br />

3 0<br />

3 θdθ<br />

π/2<br />

cos θ<br />

0<br />

<br />

1 − sin 2 θ <br />

dθ = 32a3<br />

9 .<br />

= 16a3<br />

3<br />

2. La superficie lateral Σ de Ω es la superficie cilíndrica comprendida entre las gráficas de las<br />

funciones h (x, y) = √ x 2 + y 2 y g (x, y) = 0, definidas sobre la curva borde C del dominio D.<br />

Por ello, puede relacionarse el área de Σ con una integral curvilínea sobre C :<br />

<br />

<br />

area (Σ) = (h − g) dl = x2 + y2dl. C<br />

Por otro lado, la curva C es la circunferencia plana (x − a) 2 +y 2 = a 2 luego viene parametrizada<br />

como<br />

(x (t) , y (t)) = (a + a cos t, a sin t) t ∈ [0, 2π]<br />

C


y su vector derivada es (x ′ (t) , y ′ (t)) = (−a sin t, a cos t) que tiene módulo a. Por todo ello,<br />

area (Σ) =<br />

<br />

x<br />

C<br />

2 + y2 2π <br />

dl = a<br />

=<br />

2π <br />

a<br />

0<br />

=<br />

0<br />

a 2√ 2π √ 2π<br />

2<br />

2 1 + cos tdt = 2a<br />

x2 (t) + y2 (t)dt =<br />

(a + a cos t) 2 + a2 sin2 2π<br />

tdt = a<br />

0<br />

√ 2a2 + 2a2 cos tdt =<br />

<br />

<br />

<br />

<br />

0<br />

0<br />

cos<br />

<br />

<br />

dt = 4a 2<br />

π<br />

cos<br />

0<br />

3. El fluido F es adivergente y de clase C1 en el sólido Ω, luego<br />

podemos aplicar el Teorema de Gauss, con lo que:<br />

t<br />

2<br />

<br />

<br />

<br />

F ds =<br />

Base superior<br />

F ds +<br />

Base inferior<br />

<br />

t<br />

dt = 8a<br />

2<br />

2 .<br />

<br />

Ω div (F) dxdydz = 0; y<br />

donde el primer miembro es el flujo pedido, y las superficies del segundo miembro están orientadas<br />

hacia el exterior de Ω. Obsérvese que Σ es una superficie cilíndrica recta, y F es un<br />

campo de dirección vertical, luego el flujo de F a través de Σ es nulo (en otras palabras, F<br />

pertenece al plano tangente de Σ en cada punto, o bien F es ortogonal al vector normal a Σ en<br />

cada punto). Por todas esas razones, <br />

Σ Fds = 0.<br />

Basta, pues, calcular el flujo de F en la base inferior de Ω, que es el dominio plano D, con<br />

vector normal (0, 0, −1) se considera el flujo que sale de Ω. Dicha integral de superficie queda<br />

<br />

D<br />

<br />

F ds =<br />

D<br />

Σ<br />

F ds<br />

<br />

0, 0, − x2 + y2 <br />

<br />

◦ (0, 0, −1) dxdy = x<br />

D<br />

2 + y2 dxdy<br />

que resulta ser la integral doble ya calculada en el apartado 1. Finalmente,<br />

<br />

Base superior<br />

F ds = vol (Ω) = 32a3<br />

9 .

Hooray! Your file is uploaded and ready to be published.

Saved successfully!

Ooh no, something went wrong!